Sei sulla pagina 1di 86

Cap tulo 5 Tpicos de Algebra Linear.

I o
Contedo u
5.1 5.2 Propriedades Bsicas de Determinantes e Inversas de Matrizes . . . . . . a Noes Bsicas sobre o Espectro de uma Matriz . . . . . . . . . . . . . . . co a 5.2.1 Autovalores e Polinmios Caracter o sticos de Matrizes . . . . . . . . . . . . . 5.2.2 Autovetores de Matrizes . . . . . . . . . . . . . . . . . . . . . . . . . . . . . 5.2.3 O Trao de uma Matriz . . . . . . . . . . . . . . . . . . . . . . . . . . . . . . c 5.2.3.1 Algumas Relaes entre Determinantes e Traos de Matrizes . . . . . co c Polinmios de Matrizes . . . . . . . . . . . . . . . . . . . . . . . . . . . . . . o 5.3.1 O Teorema de Hamilton-Cayley . . . . . . . . . . . . . . . . . . . . . . . . . Matrizes Diagonalizveis e o Teorema Espectral . . . . . . . . . . . . . . . a 5.4.1 Diagonalizao Simultnea de Matrizes . . . . . . . . . . . . . . . . . . . . . ca a Matrizes Auto-adjuntas, Normais e Unitrias . . . . . . . . . . . . . . . . . a 5.5.1 Matrizes Positivas . . . . . . . . . . . . . . . . . . . . . . . . . . . . . . . . . 5.5.2 O Teorema de Inrcia de Sylvester. Superf e cies Quadrticas . . . . . . . . . a Matrizes Triangulares . . . . . . . . . . . . . . . . . . . . . . . . . . . . . . . O Teorema de Decomposio de Jordan e a Forma Cannica de Matrizes ca o 5.7.1 Resultados Preparatrios . . . . . . . . . . . . . . . . . . . . . . . . . . . . . o 5.7.2 O Teorema da Decomposio de Jordan . . . . . . . . . . . . . . . . . . . . . ca 5.7.3 Matrizes Nilpotentes e sua Representao Cannica . . . . . . . . . . . . . . ca o 5.7.4 A Forma Cannica de Matrizes . . . . . . . . . . . . . . . . . . . . . . . . . o Algumas Representaes Especiais de Matrizes . . . . . . . . . . . . . . . . co 5.8.1 A Decomposio Polar de Matrizes . . . . . . . . . . . . . . . . . . . . . . . ca 5.8.2 A Decomposio em Valores Singulares . . . . . . . . . . . . . . . . . . . . . ca 5.8.3 O Teorema da Triangularizao de Schur . . . . . . . . . . . . . . . . . . . . ca 5.8.4 A Decomposio QR e a Decomposio de Iwasawa (KAN) . . . . . . . . ca ca A Pseudo-Inversa de Moore-Penrose. Optimizao Linear . . . . . . . . . ca 5.9.1 Outras Propriedades da Pseudo-Inversa de Moore-Penrose . . . . . . . . . . 5.9.1.1 A Regularizao de Tikhonov. Existncia . . . . . . . . . . . . . . . . ca e 5.9.1.2 A Pseudo-Inversa de Moore-Penrose e o Teorema Espectral . . . . . . 5.9.2 A Pseudo-Inversa de Moore-Penrose e Problemas de Optimizao Linear . . ca 5.9.3 Existncia e Decomposio em Valores Singulares . . . . . . . . . . . . . . . e ca Propriedades Especiais de Determinantes . . . . . . . . . . . . . . . . . . . 5.10.1 Expanso do Polinmio Caracter a o stico . . . . . . . . . . . . . . . . . . . . . 5.10.2 A Desigualdade de Hadamard . . . . . . . . . . . . . . . . . . . . . . . . . . Exerc cios Adicionais . . . . . . . . . . . . . . . . . . . . . . . . . . . . . . . . . . . . . . . . . . . . . . . . . . . . . . . . . . . . . . . . . . . . . . . . . . . . . . . . . . . . . . . . . . . . . . . . . . . . . . . . . . . . . . . . . . . . . . . . . . . . . . . . . . . . . . . . . . . . . . . . . . . . . . . . . . . . . . . . . . . . . . . . . . . . . . . . . . . . . . . . . . . . . . . . . . . . . . . . . . . . . . . . . . . . . . . . . . . . . . . . . . . . . . . . . . . . . . . . . . . . . . . . . . . . . . . . . . . . . . . . . . . . . . . . . . . . . . . . . . . . . . . . . . . . . . . . . . . . . . . . . 187 . 194 . . 194 . . 197 . . 200 . . 201 . 202 . . 204 . 208 . . 220 . 223 . . 228 . . 230 . 235 . 237 . . 238 . . 242 . . 244 . . 248 . 249 . . 249 . . 251 . . 251 . . 253 . 256 . . 258 . . 260 . . 263 . . 264 . . 265 . 266 . . 266 . . 267 . 270

5.3 5.4 5.5

5.6 5.7

5.8

5.9

5.10

5.11

principal objetivo deste cap tulo apresentar a demonstraao do Teorema Espectral para matrizes diagonae c lizveis, em particular, para matrizes auto-adjuntas (resultado de grande relevncia para a Mecnica Quntica) a a a a e a demonstraao do Teorema de Decomposiao de Jordan. Sempre trabalharemos no contexto de espaos c c c vetoriais de dimenso nita Cn sobre o corpo dos complexos. A leitura deste cap a tulo pressupe que alguns o conceitos bsicos de Algebra Linear, tais como o conceito de matriz, de determinante de uma matriz, suas propriedades a e mtodos de clculo, sejam familiares ao leitor, mas uma breve reviso apresentada na Seao 5.1. Na Seao 5.2, e a a e c c pgina 194 apresentamos a noao de espectro e a de polinmio caracter a c o stico de de uma matriz. Na Seao 5.5, pgina c a 223, introduzimos as nooes de matrizes auto-adjuntas, normais e unitrias, de importncia, por exemplo, na Mecnica c a a a Quntica. Na Seao 5.8 pgina 249, apresentamos algumas representaoes de matrizes de interesse em diversos contextos a c a c (por exemplo, na teoria de grupos). Este cap tulo ser continuado no Cap a tulo 6, pgina 272, onde outros aspectos de lgebras de matrizes sero explorados. a a a 186

JCABarata. Curso de F sica-Matemtica a

Verso de 4 de abril de 2009. a

Cap tulo 5

187/1628

5.1

Propriedades Bsicas de Determinantes e Inversas de Maa trizes

A presente seao desenvolve a teoria bsica de determinantes de matrizes. Sua leitura pode, provavelmente, ser dispensada c a por aqueles que julgam dispor desses conhecimentos bsicos, mas a notaao que aqui introduzimos ser empregada alhures. a c a Propriedades mais avanadas de determinantes sero estudadas na Seao 5.10, pgina 266. c a c a Alguma notao ca

O conjunto de todas as matrizes mn (m linhas e n colunas) com entradas complexas ser denotado por Mat (C, m, n). a O conjunto de todas as matrizes quadradas n n com entradas complexas ser denotado simplesmente por Mat (C, n). a Dado um conjunto de n n meros complexos 1 , . . . , n , denotaremos por diag (1 , . . . , n ) a matriz A Mat (C, n) u cujos elementos Aij so denidos da seguinte forma: a Aij = i , se i = j 0, se i = j .

Denotaremos por a, b a matriz a b cujos elementos de matriz so todos nulos. Denotaremos por l a matriz a identidade l l. Por vezes, quando no houver perigo de confuso, poderemos omitir os sub- a a ndices e escrever a, b simplesmente como e l simplesmente como . Vamos tambm empregar as seguintes denioes. Para m, n N, sejam Im, m+n Mat (C, m, m + n) e Jm+n, n e c Mat (C, m + n, n) dadas por Im, m+n := cujas transpostas so dadas por a (Im, m+n )T :=

A mais popular dentre as matrizes diagonais a matriz identidade, que denotaremos por neste texto: e 1 0 . . := diag (1, . . . , 1) = . . . . . . . . 0 1

Uma tal matriz dita ser diagonal pois apenas os elementos de sua diagonal principal so eventualmente no-nulos. Na e a a representaao usual c 1 0 . . . .. . A = . . . . 0 n

m m, n

Jm+n, n :=

n m, n

(5.1)

m n, m

(Jm+n, n )T :=

n n, m .

(5.2)

As seguintes identidades uteis sero usadas mais adiante e sua demonstraao (fcil) deixada como exerc ao leitor: a c a e cio Im, m+n (Im, m+n )T (Jm+n, n )T Jm+n, n = =

m , n ,

(5.3) (5.4)

Para cada A Mat (C, m, n) podemos associar uma matriz quadrada A Mat (C, m + n) dada por A := (Im, m+n )T A(Jm+n, n )T = Obtemos das relaoes (5.3)-(5.4) que c A = Im, m+n A Jm+n, n . (5.6)

n, n

m, m m, n

(5.5)

JCABarata. Curso de F sica-Matemtica a

Verso de 4 de abril de 2009. a

Cap tulo 5

188/1628

Denotaremos por x1 , . . . , xn seja

Sejam x1 , . . . , xn vetores, representados na base cannica por vetores-coluna o a x1 . xa = . . . xa n x1 1 . = . . xn 1 . . . .

a matriz n n constru de forma que sua a-sima coluna seja o vetor-coluna xa , ou da e x1 , . . . , xn .. . (5.7)

x1 n

xn n

tambm evidente que e e

Considerando os vetores da base cannica o 1 0 e1 = 0 , . . . 0

e2

0 1 = 0 , . . . 0

...,

en

0 0 . = . , . 0 1

(5.8)

e1 , . . . , en .

(5.9)

A notaao acima util por permitir a seguinte observaao. Seja B uma matriz qualquer. Ento, c e c a B x1 , . . . , xn = Bx1 , . . . , Bxn . (5.10)

e Essa relaao provada observando-se a regra de multiplicaao de matrizes: a a-sima coluna de B x1 , . . . , xn c e c e B11 xa + + B1n xa 1 n . . . Bn1 xa 1
a

(5.11)

+ +

Bnn xa n

que vem a ser as componentes de Bx , representado como vetor-coluna na base cannica. o Ainda sobre essa notaao, vale a seguinte identidade util, cuja demonstraao (elementar) deixamos como exerc c c cio: se D = diag (d1 , . . . , dn ) uma matriz diagonal, ento e a x1 , . . . , xn D = d1 x1 , . . . , dn xn . (5.12)

Se v1 , . . . , vk so vetores em um espao vetorial V , denotamos por [v1 , . . . , vk ] o subespao gerado pelos vetores a c c v1 , . . . , vk , ou seja, a coleao de todos os vetores que so combinaes lineares dos vetores v1 , . . . , vk : c a co [v1 , . . . , vk ] = 1 v1 + + k vk , 1 , . . . , k C .

Seja V um espao vetorial dotado de um produto escalar , . Dizemos que dois vetores u e v so perpendiculares c a (em relaao ao produto escalar , ) se u, v = 0. c

Denotamos por [v1 , . . . , vk ] o subespao de todos os vetores perpendiculares a todos os vetores de [v1 , . . . , vk ]: c [v1 , . . . , vk ] = w V| w, (1 v1 + + k vk ) = 0 para todos 1 , . . . , k C .

Propriedades bsicas de determinantes de matrizes a colunas. O determinante de A, det(A), foi denido em (3.6) como det(A) := det (a1 , . . . , an ) , (5.13) Seja A Mat (C, n) da forma A = a1 , . . . , an para o conjunto de vetores a1 , . . . , an que representam suas

JCABarata. Curso de F sica-Matemtica a

Verso de 4 de abril de 2009. a

Cap tulo 5

189/1628

onde det a forma alternante maximal em n dimenses, normalizada de sorte que det (e1 , . . . , en ) = 1. Com isso, e o c vale det() = 1. Assim, se Sn denota o conjunto de todas as bijeoes de {1, . . . , n} em si mesmo (o chamado grupo de permutaoes de n elementos), tem-se det (ej(1) , . . . , ej(n) ) = sinal(j) para todo j Sn e, portanto, vale a expresso c a (3.7): det(A) = A1j(1) Anj(n) sinal(j) (5.14)
jSn

freq entemente denominada frmula de Leibniz para o determinante de uma matriz. u o O teorema a seguir re ne todas as propriedades fundamentais do determinante de matrizes. u Teorema 5.1 Para toda matriz A Mat (C, n) valem: 1. det(A) = n det(A) para todo C. 2. det(A) = det AT . Conseqentemente, o determinante de uma matriz troca de sinal quando da permuta de duas u de suas colunas ou linhas. 3. det(AB) = det(A) det(B) = det(BA) para qualquer B Mat (C, n). 4. det(A) = det(SAS 1 ) para qualquer S Mat (C, n), inversvel. 5. Se det(A) = 0 ento A no tem inversa. a a 6. Se det(A) = 0 ento A tem inversa e vale a chamada regra de Laplace2 : a A1 = 1 Cof(A)T , det(A) (5.15)

onde Cof(A) Mat (C, n), denominada matriz dos cofatores de A, a matriz cujos elementos so e a Cof(A)jk = det (a1 , . . . , ak1 , ej , ak+1 , . . . , an ) = det a1 , . . . , ak1 , ej , ak+1 , . . . , an . (5.16)

Em palavras, Cof(A)jk o determinante da matriz obtida substituindo a k-sima coluna de A pelo vetor ej . No e e prximo item veremos outra caracterizaao da matriz dos cofatores Cof(A). o c Conjuntamente com o item 5, conclumos que A tem inversa se e somente se det(A) = 0. 7. Os elementos de matriz de Cof(A) so dados por a Cof(A)ij = (1)i+j Men(A)ij , onde Men(A), chamada de matriz dos menores de A, a matriz de Mat (C, n) denida de sorte que cada elemento e Men(A)ij seja o determinante da matriz (n 1) (n 1) obtida eliminando-se a i-sima linha e a j-sima coluna e e de A. Se n = 1, convenciona-se denir Men(A) = 1. Assim, para det(A) = 0, a regra de Laplace escreve-se A1
ij

1 (1)i+j Cof(A)ji = Men(A)ji . det(A) det(A)

(5.17)

8. Para qualquer k {1, . . . , n} valem a expanso em linhas do determinante a


n n

det(A) =
j=1

Akj Cof(A)kj =
j=1

(1)j+k Akj Men(A)kj

(5.18)

e a expanso em colunas do determinante a


n n

det(A) =
j=1
1 Gottfried

Ajk Cof(A)jk =
j=1

(1)j+k Ajk Men(A)jk .

(5.19)

2 Pierre-Simon

Wilhelm von Leibniz (16461716). Laplace (17491827).

JCABarata. Curso de F sica-Matemtica a

Verso de 4 de abril de 2009. a

Cap tulo 5

190/1628

Em (5.147), pgina 267, apresentaremos outra frmula expl a o cita para o cmputo da inversa de matrizes baseada no o Teorema de Hamilton-Cayley (Teorema 5.3, pgina 204). a Demonstrao do Teorema 5.1. Prova de 1. Pela frmula de Leibniz (5.14), ca o det(A) =
jSn

(A1j(1) ) (Anj(n) ) sinal(j) = n det(A) .

Prova de 2. Observemos frmula de Leibniz (5.14). Usando o fato elementar que um produto de n meros complexos no o u a depende da ordem dos fatores, podemos escrever A1j(1) Anj(n) = Al(1)j(l(1)) Al(n)j(l(n)) para qualquer l Sn . Em particular, escolhendo l = j 1 obtemos A1j(1) Anj(n) = Aj 1 (1)1 Aj 1 (n)n . Assim, pela frmula de Leibniz (5.14), e o usando o fato que sinal(j) = sinal(j 1 ) para todo j Sn (justique!), vale det(A) =
jSn

Aj 1 (1)1 Aj 1 (n)n sinal(j 1 ) =

j 1 Sn

Aj 1 (1)1 Aj 1 (n)n sinal(j 1 ) =


jSn

Aj(1)1 Aj(n)n sinal(j) = det(AT ) .

Quando da permuta de duas linhas ou colunas de A seu determinante troca de sinal devido ` alternncia da forma a a det . A igualdade det(A) = det AT ensina que isso tambm ocorre quando da permuta de linhas. e E. 5.1 Exerccio. Justique todas as passagens de acima. Prova de 3. Sejam A = a1 , . . . , an e B = b1 , . . . , bn . Temos que AB = Ab1 , . . . , Abn (vide (5.10)). Agora,
n n n

(Abj )i =
k=1

Aik (bj )k =
k=1

(ak )i (bj )k ,

ou seja,

Abj =
k=1

(bj )k ak .

Assim, det(AB) = = det (Ab1 , . . . , Abn )


n n

det
n n

(b1 )k1 ak1 , . . . ,


k1 =1

(bn )kn akn


kn =1

multi-linearidade

k1 =1

kn =1

(b1 )k1 (bn )kn det (ak1 , . . . , akn )

=
kSn

(b1 )k(1) (bn )k(n) det ak(1) , . . . , ak(n) (b1 )k(1) (bn )k(n) sinal(k) det (a1 , . . . , an )

=
kSn

=
kSn

(b1 )k(1) (bn )k(n) sinal(k) det(A)

det(B) det(A) .

JCABarata. Curso de F sica-Matemtica a

Verso de 4 de abril de 2009. a

Cap tulo 5

191/1628

Acima, na passagem da terceira para a quarta linha usamos o fato que det (ak1 , . . . , akn ) anula-se a menos que a k1 , . . . , kn sejam distintos, o que somente ocorre se forem da forma k(1), . . . , k(n), respectivamente, para algum k Sn . Na passagem da quarta para a quinta linha usamos que det ak(1) , . . . , ak(n) = sinal(k) det (a1 , . . . , an ), pois det uma forma alternante. e Estabelecemos, portanto, que det(AB) = det(A) det(B) = det(BA). Prova de 4. Do item 3 segue que, para quaisquer A, S Mat (C, n), com S invers vel, vale det(A) = det((AS 1 )S) = 1 det(SAS ). a a amos 1 = det() = det(AA1 ) = Prova de 5. Se det(A) = 0 ento A no pode ter inversa, pois se existisse A1 ter 1 det(A) det(A ) = 0, absurdo. Prova de 6. E bastante claro que podemos escrever
n

ak =
j=1

Ajk ej .

(5.20)

Logo, para qualquer k {1, . . . , n} vale


n

det(A) = det (a1 , . . . , ak1 , ak , ak+1 , . . . , an ) =


j=1

Ajk det (a1 , . . . , ak1 , ej , ak+1 , . . . , an ) .

Note que ej ocorre na k-sima posiao. Provamos assim que e c


n

det(A) =
j=1

Ajk Cof(A)jk ,
n

(5.21)

onde a matriz Cof(A) foi denida em (5.16). Mostremos agora que para l = k a expresso a
j=1

Ajl Cof(A)jk nula. De e

fato,
n n

Ajl Cof(A)jk =
j=1 j=1

Ajl det (a1 , . . . , ak1 , ej , ak+1 , . . . , an )

(5.20)

det (a1 , . . . , ak1 , al , ak+1 , . . . , an ) = 0 ,

pois em det (a1 , . . . , ak1 , al , ak+1 , . . . , an ) o vetor al aparece na l-sima e na k-sima posiao o que faz det anular-se, e e c por ser uma forma alternante. Provamos, assim, que
n

Ajl Cof(A)jk = kl det(A) .


j=1

(5.22)

Vamos supor que det(A) = 0. Dena-se a matriz G = det(A)1 Cof(A)T , cujos elementos de matriz so Gkj = a det(A)1 Cof(A)jk . Ento, (5.22) diz-nos que a
n

Gkj Ajl = kl ,
j=1

ou seja,

GA = .

Isso signica que A invers com A1 = G. e vel Prova de 7. Observemos primeiramente que, supondo provisoriamente k > 1, det (a1 , . . . , ak1 , ej , ak+1 , . . . , an ) = det (a1 Aj1 ej , . . . , ak1 , ej , ak+1 , . . . , an )

JCABarata. Curso de F sica-Matemtica a

Verso de 4 de abril de 2009. a

Cap tulo 5

192/1628

devido ` linearidade e ao fato que det (ej , . . . , ak1 , ej , ak+1 , . . . , an ) = 0, pelo fato de det ser alternante. Agora, a a j-sima linha do vetor-coluna a1 Aj1 ej nula. Repetindo esse argumento podemos anular j-sima linha de todas e e e

e as colunas da matriz a1 , . . . , ak1 , ej , ak+1 , . . . , an , exceto a k-sima coluna, sem alterar seu determinante. Um pouco de meditaao nos convence que a matriz resultante obtida da matriz A anulando-se a k-sima coluna e a j-sima c e e e linha, exceto no cruzamento das duas, onde o elemento de matriz vale 1 (elemento jk). O determinante dessa matriz e Cof(A)jk . Pelo item 2 e pela propriedade de alternncia, sabemos que o determinante de uma matriz troca de sinal quando a permutamos a posiao de duas colunas ou duas linhas quaisquer. Com esse tipo de operaao podemos transportar o 1 c c do elemento jk at a posiao nn da matriz, ao preo de realizar n k transposioes de colunas vizinhas e n j de linhas e c c c vizinhas, as quais alteram o determinante por fatores (1)nk e (1)nj , respectivamente. Temos com isso que 0 . . A[jk] . , Cof(A)jk = (1)k+j det(A[jk] ) com A[jk] := det 0 0 0 1

onde A[jk] a matriz de Mat (C, n 1) obtida eliminando a j-sima linha e a k-sima coluna da matriz A. Pela frmula e e e o de Leibniz (5.14), sinal(l) . A[jk] A[jk] det A[jk] =
lSn 1l(1) nl(n)

Como A[jk]

nl(n)

= l(n), n (justique!), segue que det A[jk] =


l S
n1

A[jk]
1l (1)

A[jk] A[jk]

(n1)l (n1)

sinal(l )

=
l S
n1

A[jk]

1l (1)

(n1)l (n1)

sinal(l )

det A[jk]

= Men(A)jk .

(Justique por que a soma no lado direito da primeira linha acima sobre Sn1 e no mais sobre Sn ). Provamos, e a portanto, que Cof(A)jk = (1)k+j Men(A)jk . A relaao (5.17) imediata por (5.15). c e Prova de 8. Eq. (5.19) imediata por (5.21) e pelo item 7. Eq. (5.18) segue facilmente de (5.19) usando o item 2. e

Menores e cofatores de uma matriz. Propriedades adicionais E. 5.2 Exerccio. Seja Mat (C, n), = diag + 1, 1, +1, . . . , (1)n+1 , a matriz diagonal cujos elementos so a alternadamente +1 e 1, ou seja, ij = (1)i+1 ij . Mostre que Cof(A) = Men(A)1 para toda matriz A Mat (C, n). Para uma matriz M Mat (C, n), a transformaao de similaridade M M 1 denominada chessboard c e transformation, pois com ela os sinais so trocados em M como alternam-se as cores das casas em um tabuleiro de a xadrez. c E. 5.3 Exerccio. Usando a regra de Laplace (5.15), mostre que para toda matriz A Mat (C, n) valem as relaoes Men(A1 ) = Men(A)1 , Cof(A1 ) = Cof(A)1 ,

JCABarata. Curso de F sica-Matemtica a

Verso de 4 de abril de 2009. a

Cap tulo 5

193/1628

Cof(A) = Men(A1 ) ,

Men(A) = Cof(A1 ) .

Se A Mat (C, n) invers e vel, segue da regra de Laplace (5.15) que det(A1 ) = det(Cof(A)) = det(A)n1 . Do Exerc E. 5.3, conclui-se tambm que cio e det(Men(A)) = det(A)n1 . E. 5.4 Exerccio. Mostre que para toda matriz A Mat (C, n), n 2, vale Cof Cof(A) Do Exerc E. 5.3, obtm-se tambm cio e e Men Men(A) Assim, para toda matriz A Mat (C, n) vale Cof Cof(A) = Men Men(A) . = det(A) = det(A)
n2

1 det(A)n

det(Cof(A)) e, portanto, (5.23)

(5.24)

A. A.

n2

Portanto, se det(A) = 1 e n 2, vale Cof Cof(A) = Men Men(A) = A. Um resultado util

Mais abaixo usaremos o seguinte fato:

Proposio 5.1 Seja M Mat (C, n) uma matriz da seguinte forma ca k, nk A , M = B C

onde A uma matriz k k, B uma matriz (n k) k e C uma matriz (n k) (n k). Ento, e e e a det(M ) = det(A) det(C) .

Prova. O primeiro ingrediente da prova a constataao que e c A k, nk k, nk k A = M = B C B nk nk, k = A

k, nk
C

k, nk nk

nk, k

k
B

k, nk nk

k nk, k

k, nk
C

E. 5.5 Exerccio. Verique! Com isso, temos pela regra do determinante de um produto de matrizes que A k, nk k k, nk k det det det(M ) = det B nk, k nk nk nk, k

k, nk
C

JCABarata. Curso de F sica-Matemtica a

Verso de 4 de abril de 2009. a

Cap tulo 5

194/1628

Isso completa a prova.

Agora, pelas regras (5.18)-(5.19) de clculo de determinantes, fcil constatar (faa-o!) que a e a c k, nk k k, nk k A = det(A), = det(C) e det det det C B nk, k nk nk, k

k, nk nk

= 1.

5.2

Noes Bsicas sobre o Espectro de uma Matriz co a

Seja A Mat (C, n) uma matriz n n com entradas complexas. No estudo das propriedades de A de grande e importncia saber para quais n meros complexos a matriz A invers e para quais no . Essa questo conduz a u e vel a e a a `s seguintes importantes denioes: c Denio. O espectro de A Mat (C, n), denotado por (A), denido como sendo o conjunto de todos os C ca e para os quais a matriz A no tem inversa. Assim, um n mero complexo dito ser um elemento do espectro de a u e a A Mat (C, n) se a matriz A no possuir uma inversa. e Denio. O conjunto resolvente de A Mat (C, n), denotado por (A), denido como sendo o conjunto de todos os ca C para os quais a matriz A tem inversa. Assim, um n mero complexo dito ser um elemento do conjunto u e resolvente de A Mat (C, n) se a matriz A possuir uma inversa. E evidente que (A) e (A) so conjuntos complementares, ou seja, (A) (A) = mas (A) (A) = C. a

O espectro de uma matriz

e a vel a Um fato importante que A no-invers se e somente se det( A) = 0 (vide Teorema 5.1, pgina 189). e Assim, um n mero complexo um elemento do espectro de uma matriz A se e somente se for tal que det( A) = 0. u e Essa observaao conduz-nos ao importante conceito de polinmio caracter c o stico de uma matriz.

5.2.1

Autovalores e Polinmios Caracter o sticos de Matrizes

O polinmio caracter o stico de uma matriz

a dene, um polinmio de grau n na varivel z, com coecientes complexos, os quais dependem dos elementos de matriz o Aij de A. Isso se constata facilmente pelos mtodos usuais de clculo de determinantes (por exemplo, as expanses em e a o linha ou coluna de (5.18) e (5.19)), Esse polinmio denominado polinmio caracterstico de A e desempenha um papel muito importante no estudo de o e o propriedades de matrizes. O leitor poder encontrar na Seao 5.10.1, pgina 266, uma expresso mais expl a c a a cita para o polinmio caracter o stico em termos dos elementos de matriz Aij de A (vide (5.146), pgina 267), mas por ora no a a precisaremos de maiores detalhes sobre esse polinmio. o Como todo polinmio complexo de grau n, pA possui n ra o zes, no necessariamente distintas no plano complexo a (Teorema Fundamental da Algebra). As ra do polinmio caracter zes o stico pA so denominadas autovalores da matriz A. a Assim, o espectro de uma matriz A coincide com o conjunto de seus autovalores. O estudo de autovalores de matrizes e de grande importncia na Algebra Linear e em suas aplicaoes ` Teoria das Equaoes Diferenciais, ` Geometria, ` Teoria a c a c a a dos Sistemas Dinmicos e ` F a a sica, especialmente a F ` sica Quntica. a

Seja A Mat (C, n) uma matriz cujos elementos de matriz so Aij . Para a z A11 A12 A21 z A22 pA (z) := det(z A) = det . . . . . . An1

z C a expresso a A1n A2n . .. . . . z Ann

(5.25)

JCABarata. Curso de F sica-Matemtica a

Verso de 4 de abril de 2009. a

Cap tulo 5

195/1628

Seja A Mat (C, n) uma matriz e sejam 1 , . . . , r , 1 r n, seus autovalores distintos, cada qual com multiplicidade a1 , . . . , ar , respectivamente, ou seja, cada i uma raiz de ordem ai N do polinmio caracter e o stico de A:
r

pA (z) = det(z A) =

i=1

(z i )ai .

A quantidade ai um n mero inteiro positivo e denominado multiplicidade algbrica do autovalor i . e u e e Note-se que como o n mero de ra u zes de pA (contando as multiplicidades) exatamente igual a seu grau, segue e facilmente que a seguinte relaao vlida: c e a
r

ai = n ,
i=1

(5.26)

ou seja, a soma das multiplicidades algbricas dos autovalores de uma matriz A Mat (C, n) n. e e Uma conseqncia elementar disso a seguinte proposiao util: ue e c Proposio 5.2 Seja A Mat (C, n) uma matriz e sejam 1 , . . . , r , 1 r n, seus autovalores distintos, cada qual ca com multiplicidade algbrica a1 , . . . , ar , respectivamente. Ento e a
r

det(A) =
k=1

(k )ak .

(5.27)

Prova. Por deniao, o polinmio caracter c o stico de A pA (z) = det(z A) = k=1 (z k )ak . Tomando z = 0 e usando e r n (k )ak . Porm, det(A) = (1)n det(A) e a proposiao est demonstrada. e c a (5.26), teremos det(A) = (1) k=1
r

Essa proposiao diz que o determinante de uma matriz o produto de seus autovalores distintos elevados ` sua c e a multiplicidade algbrica. e Matrizes similares. Transformaoes de similaridade c

Duas matrizes A Mat (C, n) e B Mat (C, n) so ditas matrizes similares se existir uma matriz invers a vel P Mat (C, n) tal que P 1 AP = B. Para uma matriz invers P Mat (C, n) xa, a transformaao que leva cada matriz A Mat (C, n) ` matriz vel c a P 1 AP denominada transformaao de similaridade. e c

Sabemos que o determinante invariante por transformaoes de similaridade, pois para toda matriz A vale det(A) = e c det(P 1 AP ). O determinante no o unico objeto associado a uma matriz que invariante por transformaoes de similaridade. O a e e c polinmio caracter o stico e, portanto, o conjunto de seus autovalores (incluindo as multiplicidades algbricas), tambm o e e . Isso o conte do da seguinte armaao. e e u c Proposio 5.3 Sejam A e B Mat (C, n) duas matrizes similares, ou seja, tais que existe P Mat (C, n), inversvel, ca com B = P 1 AP . Ento, os polinmios caractersticos de A e de B coincidem: pA = pB . a o Conseqentemente, se A e B Mat (C, n) so similares, seus autovalores so iguais (e, portanto, seus espectros: u a a (A) = (B)), incluindo suas multiplicidades algbricas. e e Prova. O polinmio caracter o stico de A pA (z) = det(z A) e o de B pB (z) = det(z B). Logo, e

pA (z) = det(z A) = det(P 1 (z A)P ) = det(z P 1 AP ) = det(z B) = pB (z) ,

(5.28)

para todo z C. Acima usamos o fato que para P invers vel e para qualquer matriz M vale det(P 1 M P ) = det(P 1 ) det(M ) det(P ) = det(P 1 P ) det(M ) = det() det(M ) = det(M ). Comentrios sobre matrizes invers a veis e sobre matrizes no-invers a veis

JCABarata. Curso de F sica-Matemtica a

Verso de 4 de abril de 2009. a

Cap tulo 5

196/1628

Proposio 5.4 Seja A Mat (C, n) uma matriz arbitrria e B Mat (C, n) uma matriz inversvel. Ento, existem ca a a constantes M1 e M2 (dependentes de A e de B) com 0 < M1 M2 tais que a matriz A + B inversvel para todo e C com 0 < || < M1 e para todo C com || > M2 . Prova. Como B tem inversa, podemos escrever A + B = + AB 1 B. Assim, A + B ser invers se e somente a vel se + AB 1 o for. Seja C AB 1 e sejam {1 , . . . , n } C as n ra (no necessariamente distintas) do polinmio caracter zes a o stico pC da matriz C. Se todos as ra zes forem nulas, tomemos M1 = M2 > 0, arbitrrios. De outra forma, denamos M1 a como sendo o menor valor de |k | dentre as ra no-nulas de pC : M1 := min{|k |, k = 0} e denimos M2 como sendo zes a a o maior valor de |k | para todos os ks: M2 := max{|k |, k = 1, . . . , n}. Ento, o conjunto { C| 0 < || < M1 } e o conjunto { C| || > M2 } no contm ra do polinmio caracter a e zes o stico de C e, portanto, det( C) = 0, provando que para esses valores de a matriz C = + AB 1 invers e vel. Uma conseqncia evidente da Proposiao 5.4 a seguinte armaao: ue c e c Corolrio 5.1 Seja B Mat (C, n) uma matriz inversvel e A Mat (C, n) uma matriz arbitrria. Ento existem a a a constantes 0 < N1 N2 (dependentes de A e de B) tais que para toda C com || < N1 ou com || > N2 a matriz B + A tambm inversvel. e e Prova. Para = 0 a armaao evidente. Para = 0 a armaao segue Proposiao 5.4 escrevendo-se B + A = c e c c 1 A + B e tomando-se = 1/, N1 = 1/M2 e N2 = 1/M1 . O interesse pelo Corolrio 5.1 devido ao fato de este armar que se B Mat (C, n) uma matriz invers ento a e vel a toda matriz prxima o suciente da mesma tambm invers o e e vel. O estudante mais avanado h de reconhecer que essa c a armaao ensina-nos que o conjunto da matrizes invers c veis em Mat (C, n) um conjunto aberto (em uma topologia e mtrica adequada). Essa armaao ser generalizada (a saber, para lgebras de Banach com unidade) no Corolrio 32.4, e c a a a pgina 1527. a A Proposiao 5.4 arma tambm que sempre poss c e e vel encontrar uma matriz invers prxima a uma matriz vel o no-invers a vel. De fato, se A Mat (C, n) no tem inversa a Proposiao 5.4 garante que a matriz A + , por exemplo, a c ser invers para todo C com || pequeno o suciente, mas no-nulo. a vel a

Uma forma geomtrica de compreender as armaoes de acima lembrar que conjunto Mat (C, n) um espao e c e e c vetorial n2 -dimensional complexo e as matrizes invers veis so um subconjunto (n2 1)-dimensional do mesmo, pois so a a caracterizados pela condiao de terem determinante nulo, uma condiao polinomial sobre os n2 coecientes das matrizes c c que dene, portanto, uma unio nita de superf a cies algbricas (n2 1)-dimensionais fechadas em Mat (C, n). Desse e ponto de vista geomtrico, ca claro que o conjunto das matrizes invers e veis aberto (por ser o complementar das e superf cies fechadas mencionadas acima) e ca claro que sempre poss encontrar uma matriz invers prxima a e vel vel o uma matriz no-invers a vel, pois estas ultimas residem em superf cies algbricas de dimenso menor que a dimenso de e a a Mat (C, n). Uma propriedade dos polinmios caracter o sticos

A seguinte proposiao, a qual contm uma armaao em nada evidente, uma conseqncia da Proposiao 5.3, pgina c e c e ue c a 195, e da Proposiao 5.4, pgina 196: c a Proposio 5.5 Sejam A, B Mat (C, n). Ento, o polinmio caracterstico de AB igual ao polinmio caracterstico ca a o e o de BA, ou seja, pAB = pBA . Conseqentemente, se A, B Mat (C, n) ento as matrizes AB e BA tm os mesmos autovalores (e, portanto, os u a e mesmos espectros: (AB) = (BA)), com as mesmas multiplicidades algbricas. e O estudante mais avanado poder interesar-se em encontrar na Proposiao 32.19, pgina 1528, uma verso dos c a c a a resultados da Proposiao 5.5 para o caso de lgebras de Banach com unidade. c a Prova da Proposio 5.5. Se A ou B so invers ca a veis (ou ambas), ento AB e BA so similares, pois no primeiro caso a a teremos AB = A(BA)A1 e no segundo teremos AB = B 1 (BA)B. Nesses casos a armaao segue da Proposiao 5.3, c c

JCABarata. Curso de F sica-Matemtica a

Verso de 4 de abril de 2009. a

Cap tulo 5

197/1628

pgina 195. O unico caso que resta considerar aquele no qual nem A nem B so invers a e a veis. Nesse caso, porm, temos e e vel pela Proposiao 5.4, pgina 196, que existe M > 0 tal que a matriz A + invers para todo C pertencente c a ao aberto 0 < || < M . Assim, para tais valores de valer, pelo racioc a nio acima p(A+)B = pB(A+) . Agora, os coecientes de p(A+)B e de pB(A+) so polinmios em e, portanto, so funoes cont a o a c nuas de . Logo, a igualdade p(A+)B = pB(A+) permanece vlida no limite 0, fornecendo pAB = pBA , como desejvamos demonstrar. a a A Proposiao 5.5 pode ser generalizada para matrizes no-quadradas, como indicado no exerc que segue: c a cio E. 5.6 Exerccio. Sejam A Mat (C, m, n) e B Mat (C, n, m), de sorte que AB Mat (C, m) e BA Mat (C, n). Mostre que xn pAB (x) = xm pBA (x). Sugesto: Considere as matrizes (m + n) (m + n) denidas por a A := (Vide (5.5), pgina 187). Mostre que a A B =

n, n
AB

m, m m, n

B :=

B n, n m, m n, m

n, m

m, n n, n

e que

B A =

BA n, m m, n m, m

ca Em seguida, prove que pA B (x) = xn pAB (x) e que pB A (x) = xm pBA (x). Pela Proposio 5.5, tem-se pA B (x) = pB A (x), de onde segue que xn pAB (x) = xm pBA (x). Segue disso que o conjunto de autovalores no-nulos de AB coincide com o conjunto de autovalores no-nulos de BA: a a (AB) \ {0} = (BA) \ {0} e, portanto, (AB) e (BA) podem no ter em comum apenas o elemento 0. a

5.2.2

Autovetores de Matrizes

Comentrio sobre matrizes bijetoras a

Em parte do que segue estaremos implicitamente usando a seguinte proposiao: c

Proposio 5.6 Uma matriz A Mat (C, n) bijetora (ou seja, inversvel) se e somente se Av = 0 valer apenas ca e e para v = 0. Prova. Se A bijetora, ento existe A1 . Logo, aplicando-se A1 ` esquerda na igualdade Av = 0, obtm-se v = 0. e a a e Vamos agora provar a rec proca: vamos supor que Av = 0 vale apenas para v = 0 e provar que A injetora e sobrejetora e e, portanto, bijetora. Prova-se que A injetora por absurdo. Se A no injetora, ento, existem vetores x e y com x = y mas com Ax = Ay. e a e a Como A linear, isso implica A(x y) = 0. Pela hiptese que Av = 0 vale apenas para v = 0, segue que x = y, uma e o contradiao. c Para provarmos que A sobrejetora procedemos da seguinte forma. Seja {b1 , . . . , bn } uma base em Cn . Vamos e primeiramente mostrar que {Ab1 , . . . , Abn } um conjunto linearmente independente de vetores em Cn (e, portanto, e uma base em Cn ). Suponhamos que assim no o seja e que existam n meros complexos 1 , . . . , n , no todos nulos, tais a u a que 1 Ab1 + + n Abn = 0. Pela linearidade de A, segue que A (1 b1 + + n bn ) = 0. Novamente, pela hiptese o que Av = 0 vale apenas para v = 0, segue que 1 b1 + + n bn = 0. Isso, porm, diz que os vetores {b1 , . . . , bn } so e a linearmente dependentes, o que absurdo. e Logo, {Ab1 , . . . , Abn } um conjunto de n vetores linearmente independente em Cn e, portanto, uma base nesse e e espao. Assim, qualquer x Cn pode ser escrito como uma combinaao linear tal como x = 1 Ab1 + + n Abn = c c A (1 b1 + + n bn ). Isso mostra que x est na imagem de A. Como x arbitrrio, segue que A sobrejetora. a e a e Um corolrio evidente o seguinte: a e Corolrio 5.2 Se uma matriz A Mat (C, n) no bijetora (ou seja, se no possui inversa), ento existe um vetor a a e a a no-nulo v tal que Av = 0. a

JCABarata. Curso de F sica-Matemtica a

Verso de 4 de abril de 2009. a

Cap tulo 5

198/1628

Seja 0 um autovalor de uma matriz A. Ento, 0 A no tem inversa. Logo, como V = Cn um espao vetorial a a e c de dimenso nita, existe pelo Corolrio 5.2 acima pelo menos um vetor no-nulo v tal que (0 A)v = 0, ou seja, a a a Av = 0 v. Chegamos a mais uma importante deniao: c Denio. Um vetor no-nulo v dito ser um autovetor de uma matriz A se houver 0 C tal que ca a e Av = 0 v . Note-se que se um tal 0 satisfaz a relaao acima para algum v = 0 ento 0 A no tem inversa. 0 ento um c a a e a elemento do espectro de A, ou seja, um autovalor. 0 dito ser o autovalor associado ao autovetor v. e Uma observaao importante a seguinte. Sejam v1 e v2 dois autovetores aos quais est associado o mesmo autovalor, c e a ou seja, Av1 = 0 v1 e Av2 = 0 v2 . Ento, para quaisquer n meros complexos c1 e c2 o vetor v = c1 v1 + c2 v2 tambm a u e satisfaz Av = 0 v. De fato, Av = A(c1 v1 + c2 v2 ) = c1 Av1 + c2 Av2 = c1 0 v1 + c2 0 v2 = 0 (c1 v1 + c2 v2 ) = 0 v . A concluso a que se chega que, para cada autovalor i de uma matriz A, a coleao formada pelo vetor nulo e todos a e c os autovetores de A com autovalor i um subespao vetorial. Vamos denotar esse subespao por E(i ) ou simplesmente e c c Ei . Se i e j so autovalores distintos de A ento os subespaos de autovetores E(i ) e E(j ) tm em comum apenas a a c e o vetor nulo, ou seja, E(i ) E(j ) = {0}. Isso fcil de provar, pois se w tal que Aw = i w e Aw = j w ento, e a e a subtraindo-se uma relaao da outra ter c amos 0 = (i j )w, que implica w = 0, j que i = j . a Essas consideraoes nos levam a mais um conceito importante: o de multiplicidade geomtrica de um autovalor. c e A multiplicidade geomtrica de um autovalor e

Autovetores

Alm do conceito de multiplicidade algbrica de um autovalor, h tambm o conceito de multiplicidade geomtrica e e a e e de um autovalor, do qual trataremos agora. Como antes seja A Mat (C, n) uma matriz e sejam 1 , . . . , r , 1 r n, seus autovalores distintos, cada qual com multiplicidade algbrica a1 , . . . , ar , respectivamente. e Acima introduzimos os subespaos Ei = E(i ), denidos como sendo os subespaos gerados por todos os autovetores c c que tm i como autovalor. A multiplicidade geomtrica de um autovalor i denida como sendo a dimenso do e e e a subespao Ei , ou seja, como sendo o n mero mximo de autovetores linearmente independentes com autovalor i . c u a E importante advertir de imediato o leitor do fato que a multiplicidade algbrica e multiplicidade geomtrica de e e autovalores nem sempre coincidem. Isso bem ilustrado no seguinte exemplo simples. Seja e A = Seu polinmio caracter o stico e pa () = det( A) = det 1 0 = 2 . 0 0 1 0 .

Assim, seu ( nico) autovalor 0 com multiplicidade algbrica 2. Quais os seus autovetores? So aqueles vetores que u e e a a 0 1 a b satisfazem Av = 0. Denotando v como um vetor coluna v = , a relaao Av = 0 signica c = = 0. b 0 0 b 0 a Logo, b = 0 e todos os autovetores so da forma v = a , a C. E evidente que o subespao gerado pelos autovetores c 0 com autovalor zero tem dimenso 1. Assim, a multiplicidade algbrica do autovalor zero 2 mas a sua multiplicidade a e e geomtrica 1. e e A multiplicidade algbrica e a multiplicidade geomtrica e e

Apesar de a multiplicidade algbrica e a multiplicidade geomtrica de um autovalor nem sempre coincidirem, h uma e e a relaao de ordem entre eles. A saber, poss mostrar que a multiplicidade geomtrica de um autovalor sempre menor c e vel e e ou igual ` sua multiplicidade algbrica. a e

JCABarata. Curso de F sica-Matemtica a

Verso de 4 de abril de 2009. a

Cap tulo 5

199/1628

Isso segue das seguintes consideraoes. Seja 0 um autovalor de A Mat (C, n) e E(0 ) o subespao gerado pelos c c autovetores com autovalor 0 , e cuja dimenso denotaremos por d. Vamos escolher uma base v1 , . . . , vd , vd+1 , . . . , vn a onde os primeiros d vetores so elementos de E(0 ). Nessa base a matriz A tem a forma a D A3

d, nd
A4

d vezes (n d) d. Alguns segundos (minutos?) de meditaao, usando pela Proposiao 5.1 da pgina 193, nos levam a concluir c c a que o polinmio caracter o stico de A dado por e

onde D uma matriz d d diagonal D = diag 0 , . . . , 0 , A4 uma matriz (n d) (n d) e A3 uma matriz e e e det( A) = ( 0 )d det( A4 ) .

Isso mostra que a multiplicidade algbrica de 0 pelo menos igual a d, sua multiplicidade geomtrica. e e e E. 5.7 Exerccio. Realize a meditao sugerida acima. ca Matrizes simples

O que foi exposto acima leva-nos naturalmente ao conceito de matriz simples que, como veremos mais adiante, est a intimamente ligado ao problema da diagonalizabilidade de matrizes. e Denio. Uma matriz A Mat (C, n) dita ser uma matriz simples se cada autovalor de A tiver uma multiplicidade ca algbrica igual ` sua multiplicidade geomtrica. e a e Deixamos para o leitor provar o seguinte fato: toda matriz diagonal simples. e E. 5.8 Exerccio. Prove isso. Adiante faremos uso da seguinte proposiao. c Proposio 5.7 Se A Mat (C, n) uma matriz simples e P Mat (C, n) inversvel ento P 1 AP tambm ca e e a e e simples. Prova. J vimos na Proposiao 5.3, pgina 195, que A e P 1 AP tm o mesmo polinmio caracter a c a e o stico e, portanto, os mesmos autovalores, incluindo suas multiplicidades algbricas. Seja 0 um desses autovalores com multiplicidade algbrica e e d e sejam v1 , . . . , vd um conjunto de d autovetores linearmente independentes de A. Os vetores P 1 v1 , . . . , P 1 vd so autovetores de P 1 AP com autovalor 0 . De fato, P 1 AP P 1 vi = P 1 Avi = 0 P 1 vi . Fora isso os d vetores a P 1 v1 , . . . , P 1 vd so tambm linearmente independentes. Para ver isso, suponha houvesse constantes c1 , . . . , cd tais a e que c1 P 1 v1 + + cd P 1 vd = 0 . Multiplicando-se ` esquerda por P ter a amos c1 v1 + + cd vd = 0. Como v1 , . . . , vd so linearmente independentes as a constantes ci tm que ser todas nulas, provando que os vetores P 1 v1 , . . . , P 1 vd so tambm linearmente independentes. e a e Isso prova que a multiplicidade geomtrica do autovalor 0 pelo menos igual a d. Como ela no pode ser maior que e e a d (pgina 198), conclui-se que igual a d provando a proposiao. a e c A seguinte proposiao elementar por vezes util para vericar se uma matriz simples. c e e Proposio 5.8 Se todos os n autovalores de uma matriz A Mat (C, n) forem distintos ento A simples. ca a e Prova. Se os autovalores de A so 1 , . . . , n , todos distintos, ento cada um tem multiplicidade algbrica igual a 1. a a e Forosamente, sua multiplicidade geomtrica tambm igual a 1, j que a multiplicidade geomtrica no pode ser maior c e e e a e a que a algbrica. e

JCABarata. Curso de F sica-Matemtica a

Verso de 4 de abril de 2009. a

Cap tulo 5

200/1628

Ressaltemos que a rec proca da proposiao acima no verdadeira: uma matriz pode ser simples e possuir autovalores c a e com multiplicidade algbrica maior que 1. e

5.2.3

O Trao de uma Matriz c

Seja A Mat (C, n), cujos elementos de matriz so Aij , i, j = 1, . . . n. Sejam 1 , . . . , n seus n autovalores (no a a necessariamente distintos e repetidos conforme sua multiplicidade). Denimos o trao de A como sendo a soma de seus n autovalores: c
n

O trao de uma matriz c

Tr(A) :=
a=1

a .

Uma concluso que se tira dessa deniao que se duas matrizes so similares, ento ambas tm o mesmo trao, ou a c e a a e c seja, para qualquer matriz invers P e qualquer matriz A vale vel Tr P 1 AP = Tr(A) . (5.29)

A razo reside na observaao feita acima que duas matrizes similares tm o mesmo conjunto de autovalores e, portanto, a c e o mesmo trao. c Temos a seguinte e importante proposiao: c Proposio 5.9 O trao de uma matriz A Mat (C, n) igual a soma dos elementos de sua diagonal principal, ou ca c e seja,
n n

Tr(A) :=
a=1

a =
a=1

Aaa .

(5.30)

Prova. A demonstraao consistir em se calcular o coeciente de n1 no polinmio caracter c a o stico p() de A de dois modos diferentes. O polinmio caracter o stico pA () de A dado por (5.25). As tcnicas de clculo de determinantes e e a n (e.g., (5.18) e (5.19)) dizem-nos que o coeciente de n1 i=1 Aii . Por exemplo, para o caso n = 2 e p() = det A11 A21 A12 A22 = 2 (A11 + A22 ) + A11 A22 A12 A21 .

E. 5.9 Exerccio. Convena-se da veracidade da armativa acima para o caso de n arbitrrio. Sugesto: use a expanso c a a a em cofatores (5.18)-(5.19) ou leia a Seo 5.10.1, pgina 266. ca a Por outro lado, os autovalores de A, 1 , . . . , n , so por deniao as ra do polinmio caracter a c zes o stico. Logo, p() = ( 1 )( 2 ) ( n ) . Expandindo-se essa expresso, conclui-se que o coeciente de n1 a e (1 + + n ) = Tr(A) . E. 5.10 Exerccio. Certo? Do exposto acima, conclui-se que o coeciente de n1 no polinmio caracter o stico de A e
n

i=1

Aii = (1 + + n ) = Tr(A) ,

JCABarata. Curso de F sica-Matemtica a

Verso de 4 de abril de 2009. a

Cap tulo 5

201/1628

o que termina a prova. Essa proposiao leva a duas outras propriedades igualmente importantes: a linearidade do trao e a chamada propric c edade cclica do trao. c Proposio 5.10 (A Linearidade do Trao) Sejam A, B Mat (C, n) e , C. Ento ca c a Tr(A + B) = Tr(A) + Tr(B) .

Prova. A prova imediata por (5.30). e E curioso notar que a linearidade do trao vista acima evidente por (5.30), mas no nem um pouco evidente pela c e a e deniao do trao de uma matriz como soma de seus autovalores, pois os autovalores individuais de A + B no so c c a a em geral combinaoes lineares dos autovalores de A e de B, especialmente no caso em que A e B no comutam. c a Proposio 5.11 (A Propriedade C ca clica do Trao) Sejam A, B Mat (C, n). Ento c a Tr(AB) = Tr(BA) .

Prova. Pelo que vimos acima, tem-se


n n

Tr(AB) =
i=1

(AB)ii =
i=1

n j=1

Na segunda e quarta igualdades usamos a regra de produto de matrizes. Na terceira igualdade apenas trocamos a ordem das somas. A propriedade c clica expressa na Proposiao 5.11 pode ser provada diretamente da deniao do trao de uma matriz c c c como soma de seus autovalores (incluindo multiplicidades algbricas) se recordarmos a Proposiao 5.5, pgina 196, que e c a arma que AB e BA tm os mesmos auto-valores com as mesmas multiplicidades algbricas. e e

Aij Bji =

n j=1

Bji Aij
i=1

=
j=1

(BA)jj = Tr(BA) .

5.2.3.1

Algumas Relaes entre Determinantes e Traos de Matrizes co c

Proposio 5.12 Seja A() Mat (C, n) uma matriz que depende de forma diferencivel de uma varivel (que pode ca a a ser real ou complexa) em um certo domnio e suponhamos que A() seja invertvel para todos os valores de no domnio considerado. Ento, vale a d d 1 det A() = Tr A()1 A() . (5.31) d det A() d

Prova. Por (5.14), tem-se d det A() d =


Sn n

sinal()

d A1(1) () An(n) () + + d

Sn

sinal()A1(1) ()

d An(n) () d

=
k=1

det Bk () ,

JCABarata. Curso de F sica-Matemtica a

Verso de 4 de abril de 2009. a

Cap tulo 5

202/1628

onde Bk () a matriz obtida substituindo a k-sima linha da matrix A() pela linha e e Usando a expanso em linha do determinante, expresso (5.18), temos a a
n

d d Ak1 ()

d d Akn ()

det Bk () Logo, d det A() d

=
j=1

d Akj () d
n n

Cof A()

(5.17) kj

det A()
j=1

d Akj () d

A1

jk

= det A()
k=1 j=1

d Akj () d

A1

jk

= det A() Tr A()1

d A() d

A expresso (5.31) util at mesmo no contexto da Geometria Riemanniana. Para uma aplicaao naquele contexto, a e e c vide expresso (30.94), pgina 1428. Uma das suas conseqncias o seguinte resultado, tambm muito util: a a ue e e Proposio 5.13 Seja A Mat (C, n). Ento, vale que ca a det eA = eTr(A) . (5.32)

a Nota para o estudante: A noao de exponencial de uma matriz ser apresentada em (6.19), pgina 278. E fcil ver de c a a A A (6.19) que Ae = e A para qualquer matriz A Mat (C, n). Da Proposiao 6.6, pgina 280, segue facilmente que eA c a e invert e que sua inversa eA tambm para qualquer A Mat (C, n). vel e e
d A Prova da Proposio 5.13. Seja A() = eA . Ento d eA = AeA = eA A (por (6.19)) e, portanto, eA ca a d = A. d e d Dessa forma, (5.31) ca d ln det A() = Tr(A). Integrando-se em entre 0 e 1 e lembrando que A(1) = eA e que e amos provar. A(0) = , teremos ln det eA = Tr(A), que o que quer 1

Uma segunda demonstraao da Proposiao 5.13 ser encontrada na Proposiao 6.7, pgina 281. c c a c a

5.3

Polinmios de Matrizes o

Seja p um polinmio de grau m: p(x) = am xm + + a1 x + a0 com x C, aj C e am = 0. Para uma matriz o A Mat (C, n) denimos o polinmio matricial p(A) por o p(A) = am Am + + a1 A + a0 . Obviamente p(A) tambm uma matriz n n com entradas complexas. e e
r

Polinmios de matrizes o

Se as ra do polinmio p forem 1 , . . . , r , com multiplicidades m1 , . . . , mr , respectivamente, ento zes o a p(x) = am


j=1

(x j )mj ,

a para todo x C. E fcil provar, ento, que a


r

p(A) = am
j=1

(A j )mj .

E. 5.11 Exerccio. Justique isso.

JCABarata. Curso de F sica-Matemtica a

Verso de 4 de abril de 2009. a

Cap tulo 5

203/1628

e o a E. 5.12 Exerccio. Mostre que se D = diag (d1 , . . . , dn ) e q um polinmio ento q(D) = diag (q(d1 ), . . . , q(dn )) .

E. 5.13 Exerccio. Suponha que A = P 1 DP , onde D = diag (d1 , . . . , dn ). Se q um polinmio mostre que e o q(A) = P 1 q(D)P = P 1 diag (q(d1 ), . . . , q(dn )) P .

O polinmio m o nimo

Vamos mostrar que para cada matriz A Mat (C, n) sempre existe pelo menos um polinmio p com a propriedade o que p(A) = . Para tal notemos primeiramente que Mat (C, n) um espao vetorial complexo de dimenso n2 . De fato toda a e c a matriz A Mat (C, n), cujos elementos de matriz so Aij C pode ser trivialmente escrita na forma a
n n

A =
a=1 b=1

Aab E ab

onde E ab Mat (C, n) so matrizes cujos elementos de matriz so (E ab )ij = i,a j,b , ou seja, todos os elementos de a a matriz de E ab so nulos, exceto o elemento a, b, que vale 1. a E. 5.14 Exerccio. Certo? Assim, vemos que as matrizes {E ab , a = 1, . . . , n, b = 1, . . . , n} formam uma base em Mat (C, n), mostrando que Mat (C, n) um espao vetorial de dimenso n2 . Isto posto, temos que concluir que qualquer conjunto de mais de n2 e c a matrizes no-nulas em Mat (C, n) linearmente dependente. a e a o Se uma das matrizes Ak , k = 1, . . . , n2 , for nula, digamos Aq = , ento o polinmio p(x) = xq tem a propriedade que p(A) = 0, que o que desejamos provar. Se, por outro lado, as matrizes Ak , k = 1, . . . , n2 , so todas no-nulas, e a a 2 n2 2 ento o conjunto {, A, A , . . . , A } linearmente dependente, pois possui n + 1 elementos. Portanto, existem a e constantes c0 , . . . , cn2 , nem todas nulas, tais que c0 + c1 A + c2 A2 + + cn2 An = . Como o lado esquerdo um polinmio em A, ca provada nossa armaao que toda matriz possui um polinmio que a e o c o anula. Chegamos `s seguintes denioes: a c o e o o Denio Polinmio Mnico.. Um polinmio p : R C de grau n dito ser um polinmio mnico se for da forma ca o o p(x) = xn + an1 xn1 + + a1 x + a0 , ou seja, se o coeciente do monmio de maior grau (no caso, xn ) for igual a 1. Note-se que polinmios mnicos nunca o o o so identicamente nulos. a Denio Polinmio M ca o nimo de uma Matriz.. Dada uma matriz A Mat (C, n), o polinmio mnimo de A o o e polinmio mnico de menor grau que anulado em A, ou seja, o polinmio no-nulo de menor grau da forma o o e e o a M (x) = xm + am1 xm1 + + a1 x + a0 para o qual M (A) = . As consideraoes acima mostram que um tal polinmio sempre existe e que tem grau no mximo igual a n2 . Essa c o a , no entanto, uma estimativa exagerada para o grau do polinmio m e o nimo de uma matriz A Mat (C, n) pois, como veremos abaixo, o polinmio m o nimo de uma matriz A Mat (C, n) tem, na verdade, grau menor ou igual a n. Isso e
2

JCABarata. Curso de F sica-Matemtica a

Verso de 4 de abril de 2009. a

Cap tulo 5

204/1628

um corolrio de um teorema conhecido como Teorema de Hamilton-Cayley , que demonstraremos abaixo (Teorema 5.3, a pgina 204). a Finalizamos com um teorema bsico que garante a unicidade do polinmio m a o nimo e estabelece sua relaao com c outros polinmios que anulam A. o Teorema 5.2 O polinmio mnimo M de uma matriz A Mat (C, n) unico. Fora isso se P um polinmio noo e e o a identicamente nulo que tambm se anula em A, ou seja, P (A) = , ento P divisvel por M , ou seja, existe um e a e polinmio F tal que P (x) = F (x)M (x) para todo x C. o Demonstrao. Dada uma matriz A Mat (C, n), o polinmio m ca o nimo de A o polinmio de menor grau da forma e o M (x) = xm + am1 xm1 + + a1 x + a0 para o qual M (A) = . Vamos supor que haja outro polinmio N da forma o N (x) = xm + bm1 xm1 + + b1 x + b0 amos o polinmio o para o qual N (A) = . Subtraindo um do outro ter (M N )(x) = (am1 bm1 )xm1 + + (a1 b1 )x + (a0 b0 ) , o que tem grau menor ou igual a m 1 e para o qual vale (M N )(A) = M (A) N (A) = = . Como, por hiptese, no h polinmios no-nulos com grau menor que o de M que anulam A, isso uma contradiao, a menos que M = N . a a o a e c Isso prova a unicidade. e Seja P um polinmio no identicamente nulo para o qual valha P (A) = . Se p o grau de P , deve-se ter p m, o a onde m o grau do polinmio m e o nimo de A. Logo, pelos bem conhecidos fatos sobre diviso de polinmios, podemos a o encontrar dois polinmios F e R, cujos graus so, respectivamente p m e r com 0 r < m, tais que o a P (x) = F (x)M (x) + R(x) , para todo x C. Ora, isso diz que P (A) = F (A)M (A) + R(A) .

e e Como P (A) = e M (A) = , isso implica R(A) = . Como, porm, o grau de R menor que m, tem-se que R deve ser identicamente nulo. Isso completa a prova.

5.3.1

O Teorema de Hamilton-Cayley

Vamos aqui demonstrar um teorema sobre matrizes que ser usado mais adiante de vrias formas, em particular no a a Teorema Espectral, o chamado Teorema de Hamilton3 -Cayley4 . Esse teorema fornece tambm, como veremos, um e mtodo eciente para o clculo da inversa de matrizes. Cayley e Hamilton demonstraram casos particulares do teorema e a para matrizes 2 2, 3 3 (Cayley) e 4 4 (Hamilton). A primeira demonstraao geral devida a Frobenius5 . Cayley, c e Hamilton e Sylvester6 esto entre os fundadores modernos da teoria das matrizes7 . a Teorema 5.3 (Teorema de Hamilton-Cayley) Seja A Mat (C, n) e seja pA (x) = det(x A) o polinmio caraco terstico de A (e que tem grau n). Ento, pA (A) = . a

3 Sir

William Rowan Hamilton (18051865). Cayley (18211895). 5 Ferdinand Georg Frobenius (18491917) 6 James Joseph Sylvester (18141897). 7 Muitos certamente se surpreendero em saber que Cayley e Sylvester eram originalmente advogados. a
4 Arthur

JCABarata. Curso de F sica-Matemtica a

Verso de 4 de abril de 2009. a

Cap tulo 5

205/1628

onde 1 , , n so os autovalores de A. Mas a prpria relaao Ay = 0 indica que um dos autovalores igual a zero. a o c e Logo pA (A)y = 0. Mais genericamente, se y = 0 e {y, Ay} no for um conjunto de vetores linearmente independentes, a ento Ay e y so proporcionais, ou seja, existe um autovalor, digamos, n tal que Ay = n y. Nesse caso tambm tem-se a a e
n1

Prova. Desejamos mostrar que para todo vetor y Cn vale pA (A)y = 0. Se y = 0 isso trivial. Se y = 0 mas com e Ay = 0 ento a pA (A)y = (1)n 1 n y ,

pA (A)y =
i=1

(A i ) (A n )y = 0 ,

Seja ento y daqui por diante um vetor xado, no-nulo e tal que {y, Ay} um conjunto de dois vetores no-nulos e a a e a linearmente independentes. Como o espao Cn tem dimenso n, nem todos os conjuntos de vetores da forma c a {y, Ay, A2 y, . . . , Aj y} so formados por vetores no-nulos linearmente independentes. Por exemplo, se j n, o conjunto {y, Ay, A2 y, . . . , Aj y} a a no pode ser formado por vetores no-nulos linearmente independentes pois seu n mero excede a dimenso do espao. a a u a c Seja k o maior n mero tal que {y, Ay, A2 y, . . . Ak1 y} um conjunto de vetores no-nulos e linearmente indepenu e a dentes. E claro que 1 < k n. E claro tambm, pela deniao de k, que e c Ak y = hk y + hk1 Ay + + h1 Ak1 y , para constantes h1 , . . . , hk . Vamos denominar z1 = Ak1 y, z2 = Ak2 y, . . . , zk = y, ou seja, zj = Akj y, j = 1, . . . , k, todos no-nulos por a hiptese. Caso k < n, escolhamos ainda vetores zk+1 , . . . , zn de modo que o conjunto {z1 , . . . , zn } forme uma base em o Cn . Coloquemo-nos agora a seguinte questo: qual a forma da matriz A nessa base? No subespao gerado pelos vetores a e c {z1 , . . . , zk } tem-se o seguinte: para i = 2, . . . , k vale Azi = zi1 . Alm disso, por (5.33), Az1 = h1 z1 + h2 z2 + + hk zk . e Isso mostra que o subespao gerado pelos vetores {z1 , . . . , zk } invariante pela aao de A e o operador linear A, no c e c mesmo subespao, tem a forma c h1 1 0 ... 0 0 .. h2 . 0 0 0 1 . . . .. .. .. . . . . . . . . . . (5.34) hk2 0 0 . . . 1 0 hk1 0 0 . . . 0 1 hk 0 0 ... 0 0 E. 5.15 Exerccio. Justique isso. Se designarmos por P o operador que realiza essa mudana de base, o operador linear A na base {z1 , . . . , zn } tem, c portanto, a forma A = P 1 AP , onde A1 k, nk , A = A2 A3 onde A1 a matriz k k denida em (5.34), A2 uma matriz (n k) k e A3 uma matriz (n k) (n k). No nos e e e a ser necessrio especicar os elementos das matrizes A2 e A3 . a a Outros segundos (minutos?) de meditaao, usando a Proposiao 5.1 da pgina 193, nos levam a concluir que o c c a polinmio caracter o stico pA pode ser escrito como pA (x) = det(x A ) = det(x A1 ) det(x A3 ) . (5.33)

pois (A n )y = Ay n y = 0.

JCABarata. Curso de F sica-Matemtica a

Verso de 4 de abril de 2009. a

Cap tulo 5

206/1628

O estudante deve recordar-se que as matrizes A e A , por serem similares, tm o mesmo polinmio caracter e o stico (Proposiao 5.3, pgina 195). c a Vamos denominar qk (x) = det(x A1 ) e rk (x) = det(x A3 ). Claramente, pA (x) = qk (x)rk (x). No ser necessrio, a a a no que segue, calcular rk , mas precisaremos calcular qk . Como esse pequeno resultado tem interesse independente, vamos formul-lo como um lema, para futura referncia. a e Lema 5.1 Para h1 , . . . , hk C, tem-se x h1 1 0 h2 x 1 . .. . . qk (x) := det . hk2 0 0 hk1 0 0 hk 0 0

... .. . .. . .. . ... ...

0 0 .. .

0 0 . . .

= xk (h1 xk1 + + hk1 x + hk ) . 1 0 x 1 0 x

(5.35)

Prova. A prova feita por induao. Para k = 2 vale e c q2 (x) = det Para k > 2, tem-se, pelas bem conhecidas x h1 1 .. h2 . x . .. qk (x) = x det . . . hk2 0 hk1 0 . . . x h1 h2 1 x = x2 h1 x h2 .

regras de clculo de determinantes, a x h1 0 0 h2 0 0 .. + 1 det . . . . hk2 x 1 hk 0 x (k1)(k1) 1 0

1 x 0 0

0 .. .. . . 0 .. . x 0

...

0 1 0 (k1)(k1)

x = xqk1 (x) + (1)k1+1 (hk ) det 0 0 = xqk1 (x) + (1)k+1 hk (1)k2 = xqk1 (x) hk . E. 5.16 Exerccio. Complete os detalhes. Assim, se pela hiptese indutiva qk1 da forma o e

... 0 0 . 0 1 . . 0 . .. .. .. . . . . . .. . 1 0 0 0 . . . x 1 (k2)(k2)

(5.36)

qk1 (x) = xk1 (h1 xk2 + + hk2 x + hk1 ) , segue de (5.36) que qk (x) = x(xk1 (h1 xk2 + + hk2 x + hk1 )) hk = xk (h1 xk1 + + hk2 x2 + hk1 x + hk ) , (5.37)

JCABarata. Curso de F sica-Matemtica a

Verso de 4 de abril de 2009. a

Cap tulo 5

207/1628

como quer amos provar. Retomando, temos que pA (A)y = qk (A)rk (A)y = rk (A)qk (A)y. Sucede, porm, que qk (A)y = 0. De fato, pelo e cmputo acima, o qk (A)y = Ak y h1 Ak1 y hk2 A2 y hk1 Ay hk y ,

que igual a zero por (5.33). Logo pA (A)y = 0. Como y foi escolhido arbitrrio, segue que pA (A) = , demonstrando o e a Teorema de Hamilton-Cayley, Teorema 5.3.

O Teorema de Hamilton-Cayley fornece-nos um mtodo de calcular a inversa de matrizes no-singulares. De fato, se e a pA (x) = xn + an1 xn1 + + a1 x + a0 o polinmio caracter e o stico de uma matriz no-singular A, ento o Teorema de a a Hamilton-Cayley arma que An + an1 An1 + + a1 A + a0 = , ou seja, Isso tem por implicaao c A1 = A An1 + an1 An2 + + a2 A + a1 = a0 . (5.38)

O Teorema de Hamilton-Cayley e a inversa de matrizes

1 An1 + an1 An2 + + a2 A + a1 . a0

Vide (5.147), pgina 267, para uma expresso mais expl a a cita. Nota. Usando a deniao de polinmio caracter c o stico pA (x) = det(x A), evidente (tomando-se x = 0) que e a0 = (1)n det(A). Assim, a0 = 0 se e somente se A for no-singular. a Em muitos casos a frmula (5.38) bastante eciente para calcular A1 , pois a mesma envolve poucas operaoes o e c algbricas em comparaao com outros mtodos, o que uma vantagem para valores grandes de n. Compare, por e c e e exemplo, com a regra de Laplace, expresso (5.17), pgina 189, para o clculo de A1 , que envolve o cmputo de n2 + 1 a a a o determinantes de sub-matrizes de ordem n 1 de A. e a E. 5.17 Exerccio. Use esse mtodo para calcular a inversa das suas matrizes no-singulares favoritas. De volta ao polinmio m o nimo

O Teorema 5.2, pgina 204, e o Teorema de Hamilton-Cayley, juntos, permitem-nos precisar algo a respeito da forma a geral do polinmio m o nimo de uma matriz. Se A Mat (C, n) tem r autovalores distintos 1 , . . . , r , cada qual com multiplicidade algbrica a1 , . . . , ar , e respectivamente, ento seu polinmio caracter a o stico pA da forma e
r

pA (x) =
k=1

(x k )ak .

Pelo Teorema de Hamilton-Cayley, pA (A) = 0 e, portanto, pelo Teorema 5.2, M , o polinmio m o nimo de A, divide q. Logo, M deve ser da forma
s

M (x) =
l=1

(x kl )bl ,

(5.39)

e onde s r, {k1 , . . . , ks } {1 , . . . , r } e onde 0 < bl akl para todo 1 l s. Seja agora, porm, vm = 0 um autovetor de A com autovalor m Segue do fato que M (A) = 0 que
s s

0 = M (A)vm =
l=1 s

(A kl )bl vm =

l=1

(m kl )bl vm .

Logo, l=1 (m kl )bl = 0 e isso implica que m {k1 , . . . , ks }. Como isso vale para todo 1 m r, segue que a e {1 , . . . , r } {k1 , . . . , ks } e, portanto, {1 , . . . , r } = {k1 , . . . , ks }. Nossa concluso resumida no seguinte:

JCABarata. Curso de F sica-Matemtica a

Verso de 4 de abril de 2009. a

Cap tulo 5

208/1628

Proposio 5.14 Seja A Mat (C, n) com r autovalores distintos 1 , . . . , r C, cada qual com multiplicidade ca algbrica a1 , , . . . , ar , sendo 1 r n. Ento, M , o polinmio mnimo de A, da forma e a o e
r

M (x) =
k=1

(x k )bk ,

(5.40)

x C, onde 0 < bl al para todo 1 l r. Em particular, se A Mat (C, n) tiver exatamente n autovalores distintos, teremos que bl = al = 1 para todo 1 l n, e
n

M (x) = pA (x) =
k=1

(x k ) ,

x C.

5.4

Matrizes Diagonalizveis e o Teorema Espectral a

Vamos agora apresentar uma noao intimamente ligada ` de matriz simples introduzida acima (pgina 199), mas de c a a importncia maior. a e a vel P Denio. Uma matriz A Mat (C, n) dita ser uma matriz diagonalizvel se existir uma matriz invers ca Mat (C, n) tal que P 1 AP uma matriz diagonal, ou seja, e d1 0 . . . . P 1 AP = D = diag (d1 , . . . , dn ) = . . . . . . 0 dn a E fcil de se ver que os elementos da diagonal de D so os autovalores de A. De fato, se A diagonalizvel por P , a e a vale para seu polinmio caracter o stico p() = det( A) = det(P 1 ( A)P ) = det( P 1 AP ) = det( D) d1 . = det . . 0 .. . 0 . . . dn = ( d1 ) ( dn ) ,

Matrizes diagonalizveis a

o que mostra que os di so as ra do polinmio caracter a zes o stico de A e, portanto, seus autovalores. E. 5.18 Exerccio. Justique todas as passagens acima. Diagonalizao de matrizes ca

O prximo teorema fundamental no estudo de matrizes diagonalizveis. o e a

Teorema 5.4 Uma matriz A Mat (C, n) diagonalizvel se e somente se possuir um conjunto de n autovetores e a linearmente independentes, ou seja, se e somente se o subespao gerado pela coleao de todos os autovetores de A possuir c c dimenso n. a

Prova. Vamos primeiro provar que se A Mat (C, n) possui um conjunto de n autovetores linearmente independentes ento A diagonalizvel. Para tal vamos construir a matriz P que diagonaliza A. a e a

JCABarata. Curso de F sica-Matemtica a

Verso de 4 de abril de 2009. a

Cap tulo 5

209/1628

Seja {v 1 , . . . , v n } um conjunto de n autovetores linearmente independentes de A, cujos autovalores so {d1 , . . . , dn }, a i respectivamente. Vamos denotar as componentes de v i na base cannica por vj , j = 1, . . . , n. Seja a matriz P denida o por P = v 1 , . . . , v n , ou seja,
1 v1 . P = . .

1 vn

Como se v pela construao, a a-sima coluna de P formada pelas componentes do vetor v a . Por (5.10), segue que e c e e AP = Por (5.12) vale, porm, que e
1 v1 . = . .

.. .

n v1 . . . .
n vn

Av 1 , . . . , Av n

d1 v 1 , . . . , dn v n .

d1 v 1 , . . . , dn v n

1 vn

.. .

n v1 d1 . . . . . .
n vn

.. .

dn

0 . = PD . . .

E. 5.19 Exerccio. Verique. Portanto, AP = P D. Como, por hiptese, as colunas de P so formadas por vetores linearmente independentes, o a tem-se que det(P ) = 0 (por que?). Logo, P invers e, portanto, P 1 AP = D, como quer e vel amos demonstrar. Vamos provar agora a armaao rec c proca que se A diagonalizvel, ento possui n autovetores linearmente indee a a pendentes. Suponha que exista P tal que d1 0 . . . . P 1 AP = D = . . . . . . 0 dn 0 1 = 0 , . . . 0 0 0 . = . . 0 1

E evidente que os vetores da base cannica o 1 0 e1 = 0 , . . . 0

e2

...,

en

so autovetores de D com Dea = da ea . Logo, v a = P ea so autovetores de A, pois a a

Av a = AP ea = P Dea = P (da ea ) = da P ea = da v a . Para provar que os vetores v a so linearmente independentes, suponha que existam n meros complexos 1 , . . . , n tais a u que 1 v 1 + + n v n = 0. Multiplicando-se ` esquerda por P 1 ter a amos 1 e1 + + n en = 0. Como os ea so a obviamente linearmente independentes, segue que 1 = = n = 0. Matrizes diagonalizveis e matrizes simples a

Vamos agora discutir a relaao entre os conceitos de matriz diagonalizvel e o de matriz simples, conceito esse c a introduzido ` pgina 199. Tem-se a saber o seguinte fato: a a Proposio 5.15 Uma matriz A Mat (C, n) diagonalizvel se e somente se for simples, ou seja, se e somente se a ca e a multiplicidade algbrica de cada um dos seus autovalores coincidir com sua multiplicidade geomtrica. e e

JCABarata. Curso de F sica-Matemtica a

Verso de 4 de abril de 2009. a

Cap tulo 5

210/1628

Prova. Se A diagonalizvel existe P tal que P 1 AP = D, diagonal. Como toda matriz diagonal, D simples. e a e Escrevamos D na forma Um conjunto de n-autovetores de D linearmente independentes fornecido pelos vetores da base cannica: e o 1 0 0 0 1 0 . e1 = 0 , e2 = 0 , . . . , en = . . . . . . . 0 . . 1 0 0 c Para a matriz A, os vetores P e1 , . . . , P ea1 geram o subespao de autovetores com autovalor 1 etc. E claro que a 1 a1 a a o dimenso desse subespao a1 , pois P e , . . . , P e so linearmente independentes, j que os vetores da base cannica a c e a e mos que A simples. e e1 , . . . , ea1 o so. Como isso tambm vale para os demais autovalores conclu Resta-nos agora mostrar que se A Mat (C, n) simples ento A diagonalizvel. Como antes, sejam 1 , . . . , r , e a e a e 1 r n, seus autovalores distintos, cada qual com multiplicidade algbrica a1 , . . . , ar , respectivamente, e seja E(i ) o subespao gerado pelos autovetores com autovalor i . Como A simples, tem-se que a dimenso de E(i ) ai . J c e a e a observamos (pgina 198) que subespaos E(i ) associados a autovalores distintos tm em comum apenas o vetor nulo. a c e r Assim, se em cada E(i ) escolhermos ai vetores independentes, teremos ao todo um conjunto de i=1 ai = n autovetores (vide (5.26)) linearmente independentes de A. Pelo Teorema 5.4, A diagonalizvel, completando a prova. e a D = diag 1 , . . . , 1 , . . . , r , . . . , r , .
a1 vezes ar vezes

c Os vetores e1 , . . . , ea1 geram o subespao de autovetores com autovalor 1 de D etc.

Projetores

Uma matriz E Mat (C, n) dita ser um projetor se satiszer e E2 = E .

Discutiremos vrias propriedades importantes de projetores adiante, especialmente de uma classe especial de projea tores denominados projetores ortogonais. Por ora, vamos mostrar duas propriedades que usaremos logo abaixo quando discutirmos o teorema espectral. A primeira propriedade a armaao que se um autovalor de um projetor E ento ou igual a zero ou a um. e c e a e De fato se v um autovetor associado a um autovalor de E, tem-se que Ev = v e E 2 v = 2 v. Como E 2 = E, segue e que 2 v = v. Logo ( 1) = 0 e, portanto, = 0 ou = 1.

A segunda propriedade uma conseqncia da primeira: o trao de um projetor E Mat (C, n) um n mero inteiro e ue c e u positivo ou nulo, mas menor ou igual a n. De fato, pela deniao, o trao de um projetor E a soma de seus autovalores. c c e Como os mesmos valem zero ou um a soma um inteiro positivo ou nulo. Como h no mximo n autovalores a soma e a a no pode exceder n. Na verdade, o unico projetor cujo trao vale exatamente n a identidade e o unico projetor cujo a c e trao vale exatamente 0 a matriz nula (por que?). c e Essas observaoes tm a seguinte conseqncia que usaremos adiante. Se E1 , . . . , Er so r projetores no-nulos com c e ue a a a propriedade que
r

=
a=1

Ea

ento r n. Para ver isso, basta tomar o trao de ambos os lados dessa expresso: a c a
r

Tr() =
a=1

Tr(Ea ) .

(5.41)

O lado esquerdo vale n enquanto que o lado direito uma soma de r inteiros positivos. Obviamente isso s poss se e oe vel r n.

JCABarata. Curso de F sica-Matemtica a

Verso de 4 de abril de 2009. a

Cap tulo 5

211/1628

a e Uma outra observaao util a seguinte: se E e E so dois projetores satisfazendo EE = E E = , ento E + E c e a igualmente um projetor, como facilmente se constata. O Teorema Espectral

O chamado Teorema Espectral um dos mais importantes teoremas de toda a Algebra Linear e, em verdade, de e toda Anlise Funcional, j que o mesmo possui generalizaoes para operadores limitados e no-limitados (auto-adjuntos) a a c a agindo em espaos de Hilbert. Dessas generalizaoes trataremos na Seao 32.6.1, pgina 1569, para o caso dos chamados c c c a operadores compactos e na Seao 32.7, pgina 1575, para o caso geral de operadores limitados auto-adjuntos. Nessa verso c a a mais geral o teorema espectral de importncia fundamental para a interpretaao probabil e a c stica da F sica Quntica. Vide a discusso da Seao 32.7.5, pgina 1590. a c a Teorema 5.5 (O Teorema Espectral para Matrizes) Uma matriz A Mat (C, n) diagonalizvel se e somente e a se existirem r N, 1 r n, escalares distintos 1 , . . . , r e projetores no-nulos distintos E1 , . . . , Er Mat (C, n) a tais que
r

A =
a=1

a Ea ,
r

(5.42)

=
a=1

Ea

(5.43)

e Ei Ej = i, j Ej . Os escalares 1 , . . . , r vm a ser os autovalores distintos de A. e Adiante demonstraremos uma verso um pouco mais detalhada desse importante teorema (Teorema 5.7, abaixo). Os a projetores Ea que surgem em (5.42) so denominados projetores espectrais de A. A decomposiao (5.42) freq entemente a c e u denominada decomposiao espectral de A. Na Proposiao 5.17, pgina 213 mostraremos como os projetores espectrais c c a Ea de A podem ser expressos em termos de polinmios em A. Na Proposiao 5.18, pgina 213, provaremos a unicidade o c a da decomposiao espectral de uma matriz diagonalizvel. c a Prova do Teorema 5.5. Se A Mat (C, n) diagonalizvel existe P Mat (C, n) tal que P 1 AP = D = diag (1 , . . . , n ), e a onde 1 , . . . , n so os autovalores de A. Como pode haver autovalores repetidos, vamos denotar por {1 , . . . , r }, a 1 r n, o conjunto de autovalores distintos de A. E bem claro que podemos escrever
r

D =
a=1

a Ka ,

onde as matrizes Ka so todas matrizes diagonais, cujos elementos diagonais so ou 0 ou 1 e tais que a a
r

Ka = .
a=1

(5.44)

Por exemplo, se 2 0 D = 0 0

As matrizes Ka so simplesmente denidas de modo a terem elementos de matriz iguais a 1 nas posioes da diagonal a c ocupadas pelo autovalor a em D e zero nos demais. Formalmente, 1, se i = j e (D)ii = a 0, se i = j e (D)ii = a . (Ka )ij = 0, se i = j 0 3 0 0 0 0 2 0 0 0 0 4 1 0 D = 2 0 0 0 0 0 0 0 0 1 0 0 0 0 0 +3 0 0 0 0 0 1 0 0 0 0 0 0 0 0 0 0 +4 0 0 0 0 0 0 0 0 0 0 0 0 0 0 . 0 1

teremos

JCABarata. Curso de F sica-Matemtica a

Verso de 4 de abril de 2009. a

Cap tulo 5

212/1628

a E fcil constatar que as matrizes Ka tm a seguinte propriedade: e Ka Kb = a, b Ka . (5.45)

De fato, evidente que (Ka )2 = Ka para todo a, pois Ka diagonal com zeros ou uns na diagonal. Analogamente, se e e a = b Ka Kb = 0, pois os zeros ou uns aparecem em lugares distintos das diagonais das duas matrizes. Como A = P DP 1 , tem-se que A =
a=1 r r

a Ea ,

a onde Ea := P Ka P 1 . E fcil agora provar que =


a=1 r r

Ea e que Ei Ej = i, j Ej . De fato, por (5.44),


r

Ea =
a=1 a=1

P Ka P 1 = P
a=1

Ka P 1 = P P 1 = .

Analogamente, tem-se por (5.45), Ea Eb = P Ka P 1 P Kb P 1 = P Ka Kb P 1 = a, b P Ka P 1 = a, b Ea . Vamos agora provar a rec proca. Vamos supor que A possua a representaao (5.42), onde os Ea s satisfazem as c propriedades enunciadas. Notemos primeiramente que para todo vetor x, os vetores Ek x ou so nulos ou so autovetores de A. De fato, por a a (5.42)
r

AEk x =
j=1

j Ej Ek x = k Ek x .

Logo ou Ek x = 0 ou Ek x autovetor de A. e Como h no mximo n autovetores, o espao por eles gerado tem dimenso menor ou igual a n. Por (5.43), porm, a a c a e vale para todo vetor x que
r

x = x =
k=1

Ek x .

Para x no-nulo, alguns dos Ek x, acima, devem ser no-nulos e, portanto, autovetores de A. Assim, todo vetor x pode a a ser escrito como uma combinaao linear de autovetores de A, o que signica que o espao gerado por esses autovetores c c tem dimenso exatamente igual a n. Pelo teorema 5.4, A diagonalizvel. Isso completa a demonstraao. a e a c No Teorema 5.7, pgina 216, apresentaremos uma segunda demonstraao do Teorema Espectral para Matrizes, a qual a c lana luz sobre outras condioes de diagonalizabilidade de matrizes. Antes, exploremos algumas das conseqncias do c c ue Teorema Espectral. O Clculo Funcional para matrizes diagonalizveis a a

O Teorema Espectral tem o seguinte corolrio, muitas vezes conhecido como clculo funcional. a a

Teorema 5.6 (Clculo Funcional) Seja A Mat (C, n) uma matriz diagonalizvel e seja a a
r

A =
a=1

a Ea .

sua decomposiao espectral, de acordo com o Teorema Espectral, o Teorema 5.5. Ento, para qualquer polinmio p vale c a o
r

p(A) =
a=1

p(a )Ea .

(5.46)

JCABarata. Curso de F sica-Matemtica a

Verso de 4 de abril de 2009. a

Cap tulo 5

213/1628

Prova. Tem-se, pelas propriedades dos Ea s, A2 =


a, b=1 r

a b Ea Eb =
a, b=1

a b a, b Ea =

(a )2 Ea . Analogamente,
a=1

mostra-se que Am =

a=1

(a )m Ea , para qualquer m N. O resto da prova trivial. e

E. 5.20 Exerccio. Usando (5.46) demonstre novamente o Teorema de Hamilton-Cayley (Teorema 5.3, pgina 204), agora a apenas para o caso particular de matrizes diagonalizveis. a Por simples constataao verica-se tambm facilmente a validade do seguinte resultado, que usaremos diversas vezes: c e
r

Proposio 5.16 Seja A Mat (C, n) uma matriz diagonalizvel e inversvel e seja A = ca a espectral, de acordo com o Teorema Espectral, o Teorema 5.5. Ento, A1 = a 1 Ea . a a=1
r

a Ea sua decomposiao c
a=1

O Clculo Funcional para matrizes, Teorema 5.6, tem diversas conseqncias prticas, uma delas sendo a seguinte a ue a proposiao, que permite expressar os projetores espectrais de uma matriz A diretamente em termos de A e seus autovac lores. Proposio 5.17 Seja A Mat (C, n), no-nula e diagonalizvel, e seja A = 1 E1 + + r Er , com os k s distintos, ca a a sua representaao espectral, descrita no Teorema 5.5. Sejam os polinmios pj , j = 1, . . . , r, denidos por c o
r

Obtendo os projetores espectrais

pj (x) :=
l=1 l=j

x l j l

(5.47)

Ento, a

para todo j = 1, . . . , r.

Ej = pj (A) =

k=1 k=j

1 A l j k l=1
l=j

(5.48)

Prova. Pela deniao dos polinmios pj , evidente que pj (k ) = j, k . Logo, pelo Clculo Funcional para matrizes, c o e a
r

pj (A) =
k=1

pj (k )Ek = Ej .

O Teorema Espectral para matrizes. Unicidade Proposio 5.18 A representaao espectral de uma matriz diagonalizvel A Mat (C, n) descrita no Teorema 5.5 ca c a e unica.
r

Demonstrao. Seja A Mat (C, n) diagonalizvel e seja A = ca a

k Ek a representaao espectral de A descrita no c


k=1 r Ek uma segunda k k=1

Teorema 5.5, onde k , k = 1, . . . , r, com 1 r n so os autovalores distintos de A, Seja A = a

JCABarata. Curso de F sica-Matemtica a

Verso de 4 de abril de 2009. a

Cap tulo 5

214/1628

representaao espectral para A, onde os s so distintos e onde os Ek s so no-nulos e satisfazem Ej El = j, l El e c a a a k r

=
k=1

Ek . Por essa ultima propriedade segue que para um dado vetor x = 0 vale x =

r k=1 r

Ek x, de modo que nem todos

os vetores Ek x so nulos. Seja Ek0 x um desses vetores no-nulos. Tem-se que AEk0 x = k=1 Ek Ek0 x = 0 Ek0 x. Isso a a k k mostra que 0 um dos autovalores de A e, portanto, { , . . . , } {1 , . . . , r }. Isso, em particular ensina-nos e 1 r k que r r. Podemos sem perda de generalidade considerar que os dois conjuntos sejam ordenados de modo que = k k para todo 1 k r . Assim, r r

A =
k=1

k Ek =
k=1

k Ek .

(5.49)

Sejam agora os polinmios pj , j = 1, . . . , r, denidos em (5.47), os quais satisfazem pj (j ) = 1 e pj (k ) = 0 para o todo k = j. Pelo Clculo Funcional descrito acima, segue de (5.49) que, com 1 j r , a
r r

pj (A) =
k=1

pj (k )Ek =
k=1 =Ej

pj (k )Ek ,
=Ej

Ej = Ej .

c e (A igualdade pj (A) = k=1 pj (k )Ek segue do fato que os Ek s satisfazem as mesmas relaoes algbricas que os Ek s e, portanto, para a representaao espectral de A em termos dos Ek s vale tambm o Clculo Funcional). Lembrando que a c e a r r r

igualdade Ej =

Ej

vale para todo 1 j r , segue que =

Ek =
k=1 k=1

Ek . A ultima igualdade implica


k=r +1

Ek = .

oe vel Multiplicando por El com r + 1 l r, segue que El = para todo r + 1 l r. Isso s poss se r = r , pois os E ks so no-nulos. Isso completa a demonstraao. a a c

Algumas outras identidades decorrentes do Teorema Espectral Proposio 5.19 Seja A Mat (C, n) uma matriz diagonalizvel e invertvel, cujos autovalores distintos sejam ca a {1 , . . . , r }, para algum 1 r n. Ento, vale a identidade a
r

para cada k {1, . . . , r}.

j=1 j=k

1 k j

l=1 l=k

A l

j=1 j=k

1 1 1 k j

l=1 l=k

A1 1 , l

(5.50)

Observe-se tambm que (5.50) implica e


r r

l=1 l=k

1 1 k l

A l
r

=
l=1 l=k

k l
r

A1 1 l

(5.51)

(1)r1 r2 k
r j=1

l=1 l=k

A l

=
l=1 l=k

A1 1 . l

(5.52)

Prova da Proposio 5.19. Pelo Teorema Espectral e por (5.48) podemos escrever A em sua representaao espectral: ca c
r r

A =

k=1

j=1 j=k

1 k j

l=1 l=k

A l .

(5.53)

JCABarata. Curso de F sica-Matemtica a

Verso de 4 de abril de 2009. a

Cap tulo 5

215/1628

Se A tambm invert e e vel, a Proposiao 5.16, pgina 213, informa-nos que c a


r

A1 =

k=1

Por outro lado, se A invert e vel, A1 diagonalizvel (justique!), e seus autovalores distintos so {1 , . . . , 1 } e a a r 1 (justique!). Logo, a representaao espectral de A1 c e
r r

1 1 k j=1 k j
j=k

l=1 l=k

A l .

A1 =

k=1

1 k

j=1 j=k

1 1 k 1 j

l=1 l=k

A1 1 , l

onde as matrizes

j=1 j=k

1 1 1 j k

l=1 l=k

A1 1 so os projetores espectrais de A1 . Aplicando novamente a a l


r

Proposiao 5.16, obtemos c


r

A =
k=1

Comparando (5.53) a (5.54) e evocando a unicidade da representaao espectral de A, conclu c mos pela validade de (5.50) para cada k {1, . . . , r}. E. 5.21 Exerccio. Seja A Mat (C, n) uma matriz diagonalizvel e invert com apenas dois autovalores distintos, 1 a vel e 2 . Usando (5.50) ou (5.52) mostre que A1 = 1 1 2 1 + 2 A . (5.55)

j=1 j=k

1 1 k 1 j

l=1 l=k

A1 1 . l

(5.54)

Essa relao no geralmente vlida para matrizes no-diagonalizveis e invert ca a e a a a veis com apenas dois autovalores distintos. A 11 0 matriz 0 1 0 tem autovalores +1 e 1, invert e vel, no diagonalizvel e no satizfaz (5.55). Verique! Prove (5.55) a e a a 0 0 1 diretamente do Teorema Espectral. O Teorema Espectral para matrizes. Uma segunda visita

O Teorema Espectral, Teorema 5.5, pode ser formulado de um modo mais detalhado (Teorema 5.7). A principal utilidade dessa outra formulaao a de fornecer mais informaoes sobre os projetores espectrais Ea (vide expresso c e c a (5.58), abaixo). Obtm-se tambm nessa nova formulaao mais condioes necessrias e sucientes ` diagonalizabilidade e e c c a a e que podem ser uteis, como veremos, por exemplo, no Teorema 5.21 provado adiante (pgina 219). No teorema a seguir a e em sua demonstraao seguimos parcialmente [52]. c Teorema 5.7 (Teorema Espectral para Matrizes. Verso Detalhada) Seja A Mat (C, n). So equivalentes a a as seguintes armaoes: c 1. A possui n autovetores linearmente independentes, ou seja, o subespao gerado pelos autovetores de A tem dimenso c a n. 2. A diagonalizvel, ou seja, existe uma matriz P Mat (C, n) inversvel tal que P 1 AP uma matriz diagonal e a e diag (d1 , . . . , dn ), onde os di s so autovalores de A. a 3. Para todo vetor x Cn e todo escalar C tais que (A )2 x = 0, vale que (A )x = 0.

4. Se x um vetor no-nulo tal que (A )x = 0 para algum C ento no existe nenhum vetor y com a e a a a propriedade que (A )y = x. 5. Todas as razes do polinmio mnimo de A tm multiplicidade 1. o e

JCABarata. Curso de F sica-Matemtica a

Verso de 4 de abril de 2009. a

Cap tulo 5

216/1628

6. Existem r N, escalares distintos 1 , . . . , r e projetores distintos E1 , . . . , Er Mat (C, n), denominados projetores espectrais de A, tais que
r

A =
a=1

a Ea .

Alm disso, as matrizes Ea satisfazem e

=
a=1

Ea

(5.56)

e Ei Ej = i, j Ej . Os projetores espectrais Ek do item 6, acima, podem ser expressos em termos de polinmios da matriz A: o Ek = 1 mk (A) , mk (k ) (5.58) (5.57)

para todo k, 1 k r, onde os polinmios mk so denidos por o a M (x) = (x k )mk (x) , M sendo o polinmio mnimo de A. o

Demonstrao. A prova da equivalncia ser feita demonstrando-se sucessivamente as seguintes implicaoes: 1 2, ca e a c 2 3, 3 4, 4 5, 5 6, 6 1. Que 1 implica 2 j foi demonstrado no Teorema 5.4, pgina 208. a a 2 3. Seja D = P 1 AP diagonal. D = diag (d1 , . . . , dn ). Seja (A )2 x = 0. Segue que P 1 (A )2 P y = 0 onde y = P 1 x. Logo, ou seja, (d1 )2 y1 = . . . (dn )2 yn onde yj so as componentes de y: a = 0, 0 (D )2 y = 0 ,

Agora, evidente que se (da )2 ya = 0 ento (da )ya = 0. Logo e a (D )y = 0 . Usando-se y = P 1 x e multiplicando-se ` direita por P , conclu a mos que 0 = P (D )P 1 x = (P DP 1 )x = (A )x , que o que quer e amos provar.

y1 . y = . . . yn

JCABarata. Curso de F sica-Matemtica a

Verso de 4 de abril de 2009. a

Cap tulo 5

217/1628

3 4. A prova feita por contradiao. Vamos supor que para algum vetor x = 0 exista C tal que (A )x = 0. e c Suponhamos tambm que exista vetor y tal que (A )y = x. Ter e amos (A )2 y = (A )x = 0 . c Pelo item 3 isso implica (A )y = 0. Mas isso diz que x = 0, uma contradiao. 4 5. Seja M o polinmio m o nimo de A, ou seja, o polinmio mnico8 de menor grau tal que M (A) = 0. Vamos o o mostrar que todas as ra de M tm multiplicidade 1. Vamos, por contradiao, supor que haja uma raiz, 0 , com zes e c multiplicidade maior ou igual a 2. Ter amos, para x C, M (x) = p(x)(x 0 )2 . e c o Assim, M (A) = p(A)(A 0 )2 = 0. Como M , por deniao, o polinmio de menor grau que zera em A, segue que p(A)(A 0 ) = 0 .

Assim, existe pelo menos um vetor z tal que p(A)(A0 )z = 0. Vamos denir um vetor x por x := p(A)(A0 )z. Ento, a (A 0 )x = (A 0 )p(A)(A 0 )z = p(A)(A 0 )2 z = M (A)z = 0 , pois M (A) = 0. Agora, pela deniao, c x = (A 0 )y ,

onde y = p(A)z. Pelo item 4, porm, isso imposs e e vel. 5 6. Pela hiptese que as ra o zes de M so simples segue da expresso (5.40) da Proposiao 5.14, pgina 208, que para a a c a x C,
r

M (x) =

j=1

(x j ) ,

onde j so as ra a zes de M e que coincidem com os r autovalores distintos de A. Para k = 1, . . . , r dena-se os polinmios mk por o M (x) =: (x k )mk (x) , ou seja, mk (x) :=
j=1 j=k

(x j ) .

Vamos agora denir mais um polinmio, g, da seguinte forma: o


r

E claro que mk (j ) = 0 j = k (por que?).

g(x) = 1

k=1

1 mk (x) . mk (k )

Como os polinmios mk tm grau r 1, o polinmio g tem grau menor ou igual a r 1. Porm, observe-se que, o e o e para todos os j , j = 1, . . . , r, vale
r

g(j ) = 1

k=1

mj (j ) 1 mk (j ) = 1 = 0. mk (k ) mj (j )

Assim, g tem pelo menos r ra zes distintas! O unico polinmio de grau menor ou igual a r 1 que tem r ra o zes distintas o polinmio nulo. Logo, conclu e o mos que
r

g(x) = 1
8A

k=1

1 mk (x) 0 mk (k )

deniao de polinmio mnico est a pgina 203. c o o a` a

JCABarata. Curso de F sica-Matemtica a

Verso de 4 de abril de 2009. a

Cap tulo 5

218/1628

para todo x C. Isso signica que todos os coecientes de g so nulos. Assim, para qualquer matriz B tem-se a g(B) = 0. Para a matriz A isso diz que r 1 = mk (A) . mk (k )
k=1

Denindo-se Ek := conclu mos que

1 mk (A) , mk (k )
r

(5.59)

=
k=1

Ek .

(5.60)

Para todo k vale 0 = M (A) = (A k )mk (A), ou seja, Amk (A) = k mk (A). Pela deniao de Ek isso signica c AEk = k Ek . Assim, multiplicando-se ambos os lados de (5.60) por A, segue que
r

A =
k=1

k Ek .

Para completar a demonstraao de 6, resta-nos provar que Ei Ej = i, j Ej . c Para i = j tem-se pela deniao dos Ek s que c Ei Ej = 1 mi (A)mj (A) mi (i )mj (j ) 1 (A k ) (A l ) mi (i )mj (j ) k=1 l=1
k=i l=j

1 mi (i )mj (j ) 1 mi (i )mj (j ) 0,

k=1 k=i, k=j

(A k )

l=1

(A l )

k=1 k=i, k=j

(A k ) M (A)

2 pois M (A) = 0. Resta-nos provar que Ej = Ej para todo j. Multiplicando-se ambos os lados de (5.60) por Ej teremos r

Ej =
k=1

Ej Ek = Ej Ej ,

j que Ej Ek = 0 quando j = k. Isso completa a demonstraao do item 6. a c 6 1. Notemos primeiramente que para todo vetor x, os vetores Ek x ou so nulos ou so autovetores de A. De fato, por a a 6,
r

AEk x =
j=1

j Ej Ek x = k Ek x .

JCABarata. Curso de F sica-Matemtica a

Verso de 4 de abril de 2009. a

Cap tulo 5

219/1628

Logo, ou Ek x = 0 ou Ek x autovetor de A. O espao gerado pelos autovetores de A obviamente tem dimenso e c a menor ou igual a n. Por (5.60), porm, vale para todo vetor x que e
r

x = x =
k=1

Ek x .

Assim, todo vetor x pode ser escrito como uma combinaao linear de autovetores de A, o que signica que o espao c c gerado pelos autovetores tem dimenso exatamente igual a n. a Isso completa a demonstraao do Teorema 5.7. c Destacamos ao leitor o fato de que a expresso (5.58) permite representar os projetores espectrais diretamente em a termos da matriz diagonalizvel A. a Diagonalizabilidade de projetores

A proposiao abaixo uma aplicaao simples do Teorema 5.7 a projetores. A mesma ser usada abaixo quando c e c a falarmos de diagonalizaao simultnea de matrizes. c a Proposio 5.20 Seja E Mat (C, n) um projetor, ou seja, tal que E 2 = E. Ento, E diagonalizvel. ca a e a a e e Prova. Seja E Mat (C, n) um projetor. Denamos E1 = E e E2 = E. Ento, E2 tambm um projetor, pois (E2 )2 = ( E)2 = 2E + E 2 = 2E + E = E = E2 .

eo Tem-se tambm que E1 E2 = 0, pois E1 E2 = E( E) = E E 2 = E E = 0. Fora isso, bvio que = E1 + E2 e e que E = 1 E1 + 2 E2 , com 1 = 1 e 2 = 0. Ora, isso tudo diz que E satisfaz precisamente todas as condioes do item c 6 do Teorema 5.7. Portanto, pelo mesmo teorema, E diagonalizvel. e a

Uma condio suciente para diagonalizabilidade ca

At agora estudamos condioes necessrias e sucientes para que uma matriz seja diagonalizvel. Vimos que uma e c a a matriz A Mat (C, n) diagonalizvel se e somente se for simples ou se e somente se tiver n autovetores linearmente e a independentes ou se e somente se puder ser representada na forma espectral, como em (5.42). Nem sempre, porm, e e imediato vericar essas hipteses, de modo que util saber de condioes mais facilmente vericveis e que sejam pelo o e c a menos sucientes para garantir diagonalizabilidade. Veremos abaixo que , por exemplo, suciente que uma matriz seja e auto-adjunta ou normal para garantir que ela seja diagonalizvel. a Uma outra condiao util aquela contida na seguinte proposiao. c e c Proposio 5.21 Se A Mat (C, n) tem n autovalores distintos ento A diagonalizvel. ca a e a Prova. Isso imediato pelas Proposioes 5.8 e 5.15, das pginas 199 e 209, respectivamente. e c a Observaao. A condiao mencionada na ultima proposiao apenas suciente, pois h obviamente matrizes diagonac c c e a lizveis que no tm autovalores todos distintos. a a e

Outra forma de provar a Proposiao 5.21 a seguinte. Seja {1 , . . . , n } o conjunto dos n autovalores de A, c e todos distintos. O polinmio caracter o stico de A q(x) = (x 1 ) (x n ). Como as ra e zes de q tm, nesse caso, e multiplicidade 1, segue pela Proposiao 5.14, pgina 208, que o polinmio m c a o nimo de A, M , coincide com o polinmio o caracter stico de A: q(x) = M (x), x C. Logo, o polinmio m o nimo M de A tem tambm ra e zes com multiplicidade 1. Assim, pelo item 5 do Teorema 5.7, pgina 216, A diagonalizvel. a e a ca a a e E. 5.22 Exerccio. Demonstre a seguinte armao: se os autovalores de uma matriz A so todos iguais, ento A diagonalizvel se e somente se for um mltiplo de . Sugesto: use o Teorema Espectral ou a forma geral do polinmio a u a o m nimo (5.40).

JCABarata. Curso de F sica-Matemtica a

Verso de 4 de abril de 2009. a

Cap tulo 5

220/1628

s so diagonalizveis se todos os elementos acima da diagonal principal forem nulos, ou seja, se Aij = 0, j > i. o a a Naturalmente, a mesma armativa vlida para matrizes da forma AT , triangulares inferiores com diagonal principal e a constante.

Segue da armativa desse exerc que matrizes triangulares superiores com diagonal principal constante, ou seja, cio da forma A12 . . . A1(n1) A1n 0 . . . A2(n1) A2n . . .. . A = . , . . . 0 0 ... A(n1)n 0 0 ... 0

5.4.1

Diagonalizao Simultnea de Matrizes ca a

Uma matriz A Mat (C, n) dita ser diagonalizada por uma matriz P Mat (C, n) se P 1 AP for uma matriz diagonal. e Uma questo muito importante saber quando duas matrizes diagonalizveis podem ser diagonalizadas por uma a e a mesma matriz P . A resposta fornecida no prximo teorema. e o

Teorema 5.8 (Diagonalizao Simultnea de Matrizes) Duas matrizes diagonalizveis A e B Mat (C, n) podem ca a a ser diagonalizadas pela mesma matriz P Mat (C, n) se e somente se AB = BA, ou seja, se e somente se comutarem entre si. Prova. A parte fcil da demonstraao provar que se A e B podem ser diagonalizadas pela mesma matriz P ento A e a c e a B comutam entre si. De fato P 1 (AB BA)P = (P 1 AP )(P 1 BP ) (P 1 BP )(P 1 AP ) = 0, pois P 1 AP e P 1 BP so ambas diagonais e matrizes diagonais sempre comutam entre si (por que?). Assim, P 1 (AB BA)P = 0 e, portanto, a AB = BA. Vamos agora passar a mostrar que se AB = BA ento ambas so diagonalizveis por uma mesma matriz P . a a a Sejam 1 , . . . , r os r autovalores distintos de A e 1 , . . . , s os s autovalores distintos de B. Evocando o teorema espectral, A e B podem ser escritos de acordo com suas decomposioes espectrais como c
r s A i Ei i=1

A = onde, de acordo com (5.58),

B =
j=1

B j Ej ,

A Ei

e
B Ej

A B Como A e B comutam entre si e como Ei e Ej , dados em (5.61)-(5.62), so polinmios em A e B, respectivamente, a o A B segue que Ei e Ej tambm comutam entre si para todo i e todo j. e

1 s (j k ) = (B k ) , k=1 k=1
s
k=j k=j

1 r (i k ) = (A k ) , k=1 k=1
r
k=i k=i

i = 1, . . . , r

(5.61)

j = 1, . . . , s .

(5.62)

Com isso, vamos denir


A B B A Qi, j = Ei Ej = Ej Ei

para i = 1, . . . , r e j = 1, . . . , s. Note-se que os Qi, j s so projetores pois a


B A B A B A A B Q2 j = (Ei Ej )(Ei Ej ) = (Ei )2 (Ej )2 = Ei Ej = Qi, j . i,

JCABarata. Curso de F sica-Matemtica a

Verso de 4 de abril de 2009. a

Cap tulo 5

221/1628

Fora isso, fcil ver que, e a Qi, j Qk, l = i, k j, l Qi, j . E. 5.23 Exerccio. Mostre isso. Note-se tambm que e (5.63)

=
i=1 j=1

Qi, j ,

(5.64)

pois
r s r s A B Ei Ej = i=1 j=1 i=1 r A Ei

s j=1

Qi, j =
i=1 j=1

Armamos que podemos escrever A =

B Ej = = .

s A i, j Qi, j

(5.65)

i=1 j=1

e B =

s B i, j Qi, j ,

(5.66)

i=1 j=1 A B onde i, j = i e i, j = j . De fato, com essas denioes, c r s A i, j Qi, j = i=1 j=1 i=1 j=1 r s A B i Ei Ej = i=1 r A i Ei

s j=1

Para B a demonstraao anloga. c e a

B Ej = A = A .

Nas relaoes (5.65) e (5.66) poss fazer simplicaoes em funao do fato de que nem todos os projetores Qi, j so c e vel c c a no-nulos. Seja Q1 . . . , Qt a lista dos projetores Qi, j no-nulos, ou seja, a a {Q1 . . . , Qt } = {Qi, j | Qi, j = 0, i = 1, . . . , r e j = 1, . . . , s} . E evidente por (5.63) que os Qk s so projetores e que a Qk Ql = k, l Qk . Por (5.64), tem-se
t

=
k=1

Qk

(5.67)

e por (5.65) e (5.66)


t

A =
k=1 t

A Qk k B Qk k
k=1

(5.68)

B =

(5.69)

A B onde as constantes A e B esto relacionadas de modo bvio com i, j e i, j , respectivamente. a o k k

Em (5.68) e (5.69) vemos que A e B, por serem diagonalizveis e por comutarem entre si, tm decomposioes espectrais a e c com os mesmos projetores espectrais. Note-se tambm que, pela observaao feita no tpico Projetores, ` pgina 210 e c o a a (vide equaao (5.41)), tem-se 1 t n. c Vamos agora completar a demonstraao que A e B podem ser diagonalizados por uma mesma matriz invers P . c vel

JCABarata. Curso de F sica-Matemtica a

Verso de 4 de abril de 2009. a

Cap tulo 5

222/1628

Seja Ek o subespao dos autovetores de Qk com autovalor 1. subespaos Ek s diferentes tm em comum apenas o c c e vetor nulo. De fato, se k = l e w um vetor tal que Qk w = w e Ql w = w ento, como Qk Ql = 0 segue que e a 0 = (Qk Ql )w = Qk (Ql w) = Qk w = w . Seja dk a dimenso do subespao Ek e seja a c u1 , . . . , udk k k

um conjunto de dk vetores linearmente independentes em Ek . Notemos que dk coincide com a multiplicidade algbrica e do autovalor 1 de Qk , pois, conforme diz a Proposiao 5.20, o projetor Qk diagonalizvel e, portanto, uma matriz c e a e simples (Proposiao 5.15). c Como =
t k=1

Qk , tem-se, tomando-se o trao, que n = c

t k=1

dk . (5.70)

Pelas denioes, temos que c Ql ua = k, l ua , k k pois Qk ua k = ua k e, portanto, Ql ua k = Ql (Qk ua ) k = (Ql Qk )ua k = 0 para k = l. Armamos que o conjunto de vetores u1 , . . . , ud1 , u1 , . . . , ud2 , . . . u1 , . . . , udt t 2 t 1 2 1 formado por n vetores linearmente independentes. De fato, suponha que existam constantes ck, j tais que e
t dk

(5.71)

ck, j uj = 0 . k
k=i j=1
l Aplicando-se ` direita Ql ter a amos j=1 cl, j uj = 0, o que s poss se cl, j = 0 para todo j pois u1 , . . . , udl , foram oe vel l l l escolhidos linearmente independentes. Como l arbitrrio, conclu e a mos que cl, j = 0 para todo l e todo j, o que mostra que o conjunto de vetores em (5.71) linearmente independente. e

Seja ento a matriz P Mat (C, n) denida por a P = u1 , . . . , ud1 , u1 , . . . , ud2 , . . . u1 , . . . , udt . 1 2 t t 1 2

P invers pois o conjunto (5.71) linearmente independente (e, portanto, det(P ) = 0). e vel e Tem-se, AP = Escrevendo A =
t l=1

Au1 , . . . , Aud1 , Au1 , . . . , Aud2 , . . . , Au1 , . . . , Audt . 1 2 t t 1 2

A Ql (5.68) e usando (5.70), temos l


t

Aua = k
l=1

A Ql ua = A ua . l k k k

Assim, AP = onde A u1 , . . . , A ud1 , A u1 , . . . , A ud1 , . . . , A u1 , . . . , A udt 1 1 1 1 2 1 2 1 t t t t = P DA ,

Portanto, P

AP = DA . Analogamente, BP =
t

DA = diag A , . . . , A , A , . . . , A , . . . , A , . . . , A . 1 1 2 2 t t
d1 vezes d2 vezes dt vezes

Bu1 , . . . , Bud1 , Bu1 , . . . , Bud2 , . . . Bu1 , . . . , Budt . 1 2 t t 1 2

Escrevendo B =
l=1

B Ql (5.69) temos, l BP = B u1 , . . . , B ud1 , B u1 , . . . , B ud2 , . . . , B u1 , . . . , B udt 1 1 1 1 2 2 2 2 t t t t = P DB ,

JCABarata. Curso de F sica-Matemtica a

Verso de 4 de abril de 2009. a

Cap tulo 5

223/1628

onde

Portanto, P BP = DB . Isso provou que A e B so diagonalizveis pela mesma matriz invers P . A demonstraao a a vel c do Teorema 5.8 est completa. a

DB = diag B , . . . , B , B , . . . , B , . . . , B , . . . , B . 1 1 2 2 t t
d1 vezes d2 vezes dt vezes

5.5

Matrizes Auto-adjuntas, Normais e Unitrias a

Seja V um espao vetorial dotado de um produto escalar , e seja A : V V um operador linear. Um operador c linear A que para todos u, v V satisfaa c u, Av = A u, v dito ser o operador adjunto de A. Em espaos vetoriais gerais no bvio (e nem sempre verdadeiro!) que sempre exista e c a eo o adjunto de um operador linear A dado. H muitos casos, porm, nos quais isso pode ser garantido9. Aqui trataremos a e do caso dos espaos V = Cn com o produto escalar usual. c Sejam u = (u1 , . . . , un ) e v = (v1 , . . . , vn ) dois vetores de Cn para os quais dene-se o produto escalar usual
n

A adjunta de uma matriz

u, v =
k=1

uk vk .

Um operador linear A representado (na base cannica) por uma matriz cujos elementos de matriz so Aij , com e o a i, j {1, . . . , n}. E um exerc simples (faa!) vericar que o operador adjunto A de A representado (na base cannica) por uma cio c e o matriz cujos elementos de matriz so (A )ij = Aji , com i, j {1, . . . , n}. Ou seja, a matriz adjunta de A obtida (na a e base cannica!) transpondo-se A e tomando-se o complexo conjugado de seus elementos. o Os seguintes fatos so importantes: a Proposio 5.22 Se A e B so dois operadores lineares agindo em Cn ento ca a a (A + B) = A + B para todos , C. Fora isso, Por m, vale para todo A que (A ) = A. Deixamos a demonstraao como exerc para o leitor. c cio A operaao Mat (C, n) A A Mat (C, n) denominada operaao de adjunao de matrizes. Como vimos na c e c c Proposiao 5.22, a operaao de adjunao anti-linear e um anti-homomorsmo algbrico. c c c e e e Os espectro e a operao de adjuno ca ca (AB) = B A .

Seja A Mat (C, n). Como j vimos, o espectro de A, (A), o conjunto de ra de seu polinmio caracter a e zes o stico, denido por pA (z) = det(z A), z C. Como para toda B Mat (C, n) vale det(B ) = det(B) (por qu?), segue que e pA (z) = det(z A) = det(z A ) = pA (z), ou seja, pA (z) = pA (z). Com isso, provamos a seguinte armaao: c

Proposio 5.23 Seja A Mat (C, n). Ento, (A) se e somente se (A ), ou seja, um autovalor de A ca a e se e somente se um um autovalor de A . e
9 Tal o caso dos chamados operadores lineares limitados agindo em espaos de Hilbert, para os quais sempre poss e c e vel garantir a existncia e do adjunto.

JCABarata. Curso de F sica-Matemtica a

Verso de 4 de abril de 2009. a

Cap tulo 5

224/1628

Em s mbolos, as armaoes acima so expressas pela igualdade (A) = (A ). c a Matrizes Hermitianas, normais e unitrias a

Vamos agora a algumas denioes muito importantes. c

Denio. Um operador linear em Cn dito ser simtrico, Hermitiano ou auto-adjunto se A = A , ou seja, se para ca e e todos u, v V satiszer u, Av = Au, v . c a c e a Advertncia. Em espaos vetoriais de dimenso nita as nooes de operador simtrico, Hermitiano ou auto-adjunto so e sinnimas. Em espaos vetoriais de dimenso innita, porm, h uma distinao entre essas nooes relativa a problemas o c a e a c c com o dom nio de deniao de operadores. c Denio. Um operador linear em Cn dito ser normal se AA = A A. Ou seja, A normal se comuta com seu ca e e adjunto. e a a e Denio. Um operador linear em Cn dito ser unitrio se A A = AA = . E claro que todo operador unitrio ca n 1 normal e que um operador unitrio em C se e somente se A = A . Note que se A unitrio ento, para todos e a e a a u, v V , tem-se Au, Av = u, v . Denio. Se A um operador linear em Cn dene-se a parte real de A por ca e Re (A) = e a parte imaginria de A por a Im (A) = 1 (A + A ) 2 1 (A A ). 2i

E claro que essas denioes foram inspiradas nas relaoes anlogas para n meros complexos. Note tambm que c c a u e A = Re (A) + iIm (A) . e E. 5.24 Exerccio. Por qu? E importante notar que para qualquer operador linear A em Cn sua parte real e imaginria so ambas operadores a a Hermitianos: (Re (A)) = Re (A) e (Im (A)) = Im (A). E. 5.25 Exerccio. Mostre isso. Para operadores normais tem-se a seguinte proposiao, que ser util adiante e serve como caracterizaao alternativa c a c do conceito de operador normal. Proposio 5.24 Um operador linear agindo em Cn normal se e somente se sua parte real comuta com sua parte ca e imaginria. a Deixamos a demonstraao (elementar) como exerc para o leitor. c cio A importncia das denioes acima reside no seguinte fato, que demonstraremos adiante: matrizes Hermitianas e a c matrizes normais so diagonalizveis. Antes de tratarmos disso, vamos discutir algumas propriedades do espectro de a a matrizes Hermitianas e de matrizes unitrias. a Os autovalores de matrizes Hermitianas e de matrizes unitrias a

Os seguintes teoremas tm importncia fundamental para o estudo de propriedades de matrizes Hermitianas e de e a matrizes unitrias. a

JCABarata. Curso de F sica-Matemtica a

Verso de 4 de abril de 2009. a

Cap tulo 5

225/1628

Teorema 5.9 Os autovalores de uma matriz Hermitiana so sempre nmeros reais. a u Prova. Seja A Hermitiana, um autovalor de A e v = 0 um autovetor de A com autovalor . Como A Hermitiana e tem-se v, Av = Av, v . Como v um autovetor, o lado esquerdo vale v, v e o lado direito vale v, v . Logo, ( ) v, v = 0. Como v = 0 e isso implica = , ou seja, real. e Note-se que a rec proca desse teorema falsa. A matriz e Para matrizes unitrias temos a Teorema 5.10 Os autovalores de uma matriz unitria so sempre nmeros complexos de mdulo 1. a a u o Prova. Seja A unitria, um autovalor de A e v = 0 um autovetor de A com autovalor . Como A unitria tem-se a e a Av, Av = v, v . Como v um autovetor, o lado esquerdo vale v, v . Assim, (||2 1) v, v = 0. Como v = 0 e isso implica || = 1. Operadores simtricos e unitrios. Ortogonalidade de autovetores e a Teorema 5.11 Os autovetores associados a autovalores distintos de uma matriz simtrica so ortogonais entre si. e a Prova. Seja A simtrica e 1 , 2 dois de seus autovalores, que suporemos distintos. Seja v1 autovetor de A com autovalor e 1 e v2 autovetor de A com autovalor 2 . Temos, por A ser simtrico, v1 , Av2 = Av1 , v2 . O lado esquerdo vale e 2 v1 , v2 e o lado direito 1 v1 , v2 (lembre-se que 1 real). Assim (2 1 ) v1 , v2 = 0. Como 2 = 1 , segue que e v1 , v2 = 0, que o que se queria provar. e Teorema 5.12 Os autovetores associados a autovalores distintos de uma matriz unitria so ortogonais entre si. a a Prova. Seja U unitria e sejam 1 , 2 dois de seus autovalores, sendo que suporemos 1 = 2 . Seja v1 autovetor de U com a autovalor 1 e v2 autovetor de U com autovalor 2 . Temos, por U ser unitrio, U v1 , U v2 = v1 , U U v2 = v1 , v2 . a e u o O lado esquerdo vale 2 1 v1 , v2 = 2 v1 , v2 (lembre-se que 1 um n mero complexo de mdulo 1 e, portanto 1 1 1 = 1 ). Assim 2 1 v1 , v2 = 0 . 1 Como 2 = 1 , segue que v1 , v2 = 0, que o que se queria provar. e Projetores ortogonais 2 0 1 3 tem autovalores reais (2 e 3) mas no Hermitiana. a e

Um operador linear E agindo em Cn dito ser um projetor ortogonal se E 2 = E e se E = E. e 1 1 0 0

Projetores ortogonais so importantes na decomposiao espectral de matrizes auto-adjuntas, como veremos. a c Note-se que nem todo projetor ortogonal. Por exemplo E = e (E = E). O mesmo vale para E = 1 2 0 . 0 um projetor (E 2 = E) mas no ortogonal e a e

Um exemplo importante de projetor ortogonal representado por projetores sobre subespaos uni-dimensionais gee c v, v = 1. Denimos o projetor Pv rados por vetores. Seja v um vetor cuja norma assumiremos ser 1, ou seja, v = sobre o subespao gerado por v por c Pv u := v, u v , (5.72)

JCABarata. Curso de F sica-Matemtica a

Verso de 4 de abril de 2009. a

Cap tulo 5

226/1628

para todo vetor u. Provemos que Pv um projetor ortogonal. Por um lado, tem-se e
2 Pv u = v, u Pv v = v, u

v, v v = v, u v = Pv u ,

2 o que mostra que Pv = Pv . Por outro lado, para quaisquer vetores a e b, usando as propriedades de linearidade, anti-linearidade e conjugaao complexa do produto escalar, tem-se c

a, Pv b = a, v, b v = v, b

a, v =

a, v v, b =

v, a v, b = Pv a, b ,

provando que Pv = Pv . Isso mostra que Pv um projetor ortogonal. e

Um fato crucial sobre projetores como Pv o seguinte. Se u e v so dois vetores ortogonais, ou seja, se u, v = 0 e a ento Pu Pv = Pv Pu = 0. Para provar isso notemos que para qualquer vetor a vale a Pu (Pv a) = Pu ( v, a v) = v, a Pu v = v, a O mesmo se passa para Pv (Pu a). Matrizes auto-adjuntas e diagonalizabilidade u, v u = 0 .

Vamos aqui demonstrar a seguinte armaao importante: toda matriz auto-adjunta diagonalizvel. Uma outra c e a demonstraao (eventualmente mais simples) dessa armaao pode ser encontrada na Seao 5.8.3, pgina 251. Vide c c c a Teorema 5.27, pgina 253. a Teorema 5.13 Se A Mat (C, n) auto-adjunta, ento A possui n autovetores mutuamente ortonormais v1 , . . . , vn , e a com autovalores 1 , . . . , n , respectivamente, e pode ser representada na forma espectral A = 1 Pv1 + + n Pvn . (5.73)

Portanto, se A auto-adjunta, ento A diagonalizvel, sendo que possvel encontrar uma matriz unitria P que e a e a e a diagonaliza A, ou seja, tal que P 1 AP diagonal e P 1 = P . e Note-se que se 1 , . . . , r com 1 r n so os autovalores distintos de A, ento (5.73) pode ser reescrita como a a A = 1 P1 + + r Pr , onde cada Pk o projetor ortogonal dado pela soma dos Pvj s de mesmo autovalor k . A e Proposiao 5.18, pgina 213, garante a unicidade dessa representaao para A. c a c Prova do Teorema 5.13. A demonstraao que A diagonalizvel ser feita construindo-se a representaao espectral (5.73) c e a a c para A. Seja 1 um autovalor de A e v1 um autovetor de A com autovalor 1 normalizado de tal forma que v1 = 1. Vamos denir um operador A1 por A1 = A 1 Pv1 . a e e Como A e Pv1 so auto-adjuntos e 1 real, segue que A1 igualmente auto-adjunto. Armamos que A1 v1 = 0 e que [v1 ] um subespao invariante por A1 . De fato, e c A1 v1 = Av1 1 Pv1 v1 = 1 v1 1 v1 = 0 . Fora isso, se w [v1 ] tem-se A1 w, v1 = w, A1 v1 = 0 ,

mostrando que A1 w tambm elemento de [v1 ] . e e O operador A1 restrito a [v1 ] tambm auto-adjunto (por que?). Seja 2 um de seus autovalores com autovetor e e v2 [v1 ] , que escolhemos com norma 1. Seja A2 = A1 2 Pv2 = A 1 Pv1 2 Pv2 . Como 2 tambm real A2 igualmente auto-adjunto. Fora isso armamos que A2 anula os vetores do subespao [v1 , v2 ] e e e c e mantem [v1 , v2 ] invariante. De fato, A2 v1 = Av1 1 Pv1 v1 2 Pv2 v1 = 1 v1 1 v1 2 v2 , v1 v2 = 0 ,

JCABarata. Curso de F sica-Matemtica a

Verso de 4 de abril de 2009. a

Cap tulo 5

227/1628

pois v2 , v1 = 0. Analogamente, A2 v2 = A1 v2 2 Pv2 v2 = 2 v2 2 v2 = 0 . Por m, para quaisquer , C e w [v1 , v2 ] tem-se A2 w, (v1 + v2 ) = w, A2 (v1 + v2 ) = 0 , que o que quer e amos provar. Prosseguindo indutivamente, construiremos um conjunto de vetores v1 , . . . , vn , todos com norma 1 e com va [v1 , . . . , va1 ] e um conjunto de n meros reais 1 , . . . , n tais que u An = A 1 Pv1 n Pvn anula-se no subespao [v1 , . . . , vn ]. Ora, como estamos em um espao de dimenso n e os vetores vk so mutuamente c c a a ortogonais, segue que [v1 , . . . , vn ] deve ser o espao todo, ou seja, An = 0. Provamos ento que c a A = 1 Pv1 + + n Pvn . (5.74)

Vamos provar agora que essa a representaao espectral de A. Como os vk s so mutuamente ortogonais, evidente e c a e que Pvk Pvl = k, l Pvk . Resta-nos provar que Pv1 + + Pvn = . Como v1 , . . . , vn formam uma base, todo vetor x pode ser escrito como uma combinaao linear c x = 1 v1 + + n vn . (5.75)

Tomando-se o produto escalar com va , e usando o fato que os vk s so mutuamente ortogonais, tem-se a = va , x . a E. 5.26 Exerccio. Verique. Assim, (5.75) pode ser escrita como x = v1 , x v1 + + vn , x vn = Pv1 x + + Pvn x = (Pv1 + + Pvn ) x . Como isso vale para todo vetor x, segue que Pv1 + + Pvn = . Assim, A possui uma representaao espectral como c (5.42). Pelo Teorema Espectral 5.5, A diagonalizvel. e a Por (5.74), vemos que Ava = a va (verique!). Logo os a s so autovalores de A e os va s seus autovetores. Assim, a se A auto-adjunto, podemos encontrar n autovetores de A mutuamente ortogonais, mesmo que sejam autovetores com e o mesmo autovalor. Isso generaliza o Teorema 5.11. a e Pelo que j vimos A diagonalizada por P 1 AP , onde podemos escolher P = v 1 , . . . , v n . E fcil vericar, porm, a e que P unitria. De fato, um exerc simples (faa!) mostrar que e a e cio c v1 , v1 v1 , vn . . .. . . P P = . . . . vn , v1 vn , vn e a Como va , vb = a, b , a matriz do lado direito igual a , mostrando que P P = P P = e que, portanto, P unitria. e

Para concluir essa discusso, temos: a Proposio 5.25 Uma matriz A Mat (C, n) auto-adjunta, se e somente se for diagonalizvel por uma transformaao ca e a c de similaridade unitria e se seus autovalores forem reais. a Prova. Se A Mat (C, n) diagonalizvel por uma transformaao de similaridade unitria e seus autovalores so reais, e a c a a ou seja, existe P unitria e D diagonal real com P AP = D, ento A = P DP e A = P D P . Como D diagonal e a a e real, vale D = D e, portanto, A = P DP = A, provando que A auto-adjunta. A rec e proca j foi provada acima. a

JCABarata. Curso de F sica-Matemtica a

Verso de 4 de abril de 2009. a

Cap tulo 5

228/1628

Matrizes normais e diagonalizabilidade

O teorema que arma que toda matriz simtrica diagonalizvel tem a seguinte conseqncia: e e a ue

Teorema 5.14 Se A Mat (C, n) normal ento A diagonalizvel. e a e a Prova. J vimos que toda matriz A pode ser escrita na forma A = Re (A) + iIm (A) onde Re (A) e Im (A) so autoa a adjuntas. Vimos tambm que se A normal Re (A) e Im (A) comutam entre si (Proposiao 5.24). Pelo Teorema 5.8, e e c Re (A) e Im (A) podem ser simultaneamente diagonalizados. Observao. Como no caso auto-adjunto, o operador que faz a diagonalizaao pode ser escolhido unitrio. De fato, ca c a vale uma armativa ainda mais forte. Teorema 5.15 Uma matriz A Mat (C, n) normal se e somente se for diagonalizvel por um operador unitrio. e a a Prova. Resta provar apenas que se A diagonalizvel por um operador unitrio P ento A normal. Seja D = P AP . e a a a e Tem-se D = P A P (por que?). Assim, A A AA = P D P P DP P DP P D P = P (D D DD )P = 0 , j que D e D comutam por serem diagonais (duas matrizes diagonais quaisquer sempre comutam. Por qu?). Isso a e completa a prova que A normal. e Uma outra demonstraao (eventualmente mais simples) dessa armaao pode ser encontrada na Seao 5.8.3, pgina c c c a 251. Vide Teorema 5.28, pgina 253. a

5.5.1

Matrizes Positivas

Uma matriz A Mat (C, n) dita ser uma matriz positiva se w, Aw 0 para todo vetor w Cn . A seguinte e proposiao relevante10 : c e Proposio 5.26 Se A Mat (C, n) positiva, ento A Hermitiana e tem autovalores no-negativos. Reciprocamente, ca e a e a se A Hermitiana e tem autovalores no-negativos, ento A positiva. e a a e Prova. A expresso (u, v) := u, Av , u, v Cn , dene uma forma sesquilinear que, por hiptese, positiva, ou seja, a o e satisfaz (u, u) 0 para todo u Cn . Pelo Teorema 3.1, pgina 149, Hermitiana, ou seja, (u, v) = (v, u) , a e para todos os vetores u e v. Mas isso signica que u, Av = v, Au , ou seja, u, Av = Au, v para todos os vetores u e v e assim provou-se que A = A . Uma outra forma de demonstrar isso usa a desigualdade de polarizaao. Se A c positiva ento, para quaisquer vetores u, v Cn vale (u + in v), A(u + in v) 0 para todo n Z e, portanto, e a (u + in v), A(u + in v) um n mero real. Usando a identidade de polarizaao, eqs. (3.32)-(3.33), pgina 159, vale, para e u c a
dos resultados que seguem podem ser generalizados para operadores lineares positivos agindo em espaos de Hilbert. Vide Teorema c 32.21, pgina 1541. a
10 Vrios a

JCABarata. Curso de F sica-Matemtica a

Verso de 4 de abril de 2009. a

Cap tulo 5

229/1628

quaisquer vetores u, v Cn , Av, u = u, Av


(3.32)

1 in (u + in v), A(u + in v) 4 n=0


3

1 in (u + in v), A(u + in v) 4 n=0

1 in in in (u + in v), A(u + in v) 4 n=0 1 in in (u + in v), Ain (u + in v) 4 n=0 1 in (v + in u), A((1)n v + in u) 4 n=0 1 (1)n in (v + in u), A(v + in u) 4 n=0 1 in (v + in u), A(v + in u) 4 n=0
3 (3.33) 3 3 3

sesquilin.

v, Au .

Assim, Av, u = v, Au para todos u, v Cn , o que signica que A Hermitiana. Portanto, por (5.73), podemos e a escrever A = 1 Pv1 + + n Pvn , onde v1 , . . . , vn so autovetores mutuamente ortonormais de A com autovalores 1 , . . . , n , respectivamente. Disso segue que vj , Avj = j para todo j = 1, . . . , n. Como o lado esquerdo 0, por e hiptese, segue que j 0 para todo j = 1, . . . , n. o Se, reciprocamente, A for auto-adjunta com autovalores no-negativos, segue de (5.73) e da deniao de Pvj em (5.72) a c
n j=1

que w, Aw =

j | w, vj |2 0, para todo w Cn , provando que A positiva. e

O seguinte corolrio imediato. a e Corolrio 5.3 Uma matriz A Mat (C, n) positiva se somente se existe uma matriz positiva B (unvoca!) tal que a A = B 2 . As matrizes A e B comutam: AB = BA.

Demonstrao. Se A = B 2 com B positiva, ento, como B auto-adjunta (pela Proposiao 5.26), segue que para todo ca a e c w Cn vale w, Aw = w, B 2 w = Bw, Bw = Bw 2 0, provando que A positiva. Provemos agora a rec e proca.

Se A positiva ento, como comentamos na demonstraao da Proposiao 5.26, A auto-adjunta com representaao e a c c e c a espectral A = 1 Pv1 + + n Pvn , onde v1 , . . . , vn so autovetores mutuamente ortonormais de A com autovalores 1 , . . . , n , respectivamente, todos no-negativos. Dena-se a matriz a B := 1 Pv1 + + n Pvn . (5.76)

e a Como, pela ortonormalidade dos vj s, vale Pvj Pvk = j, k Pvj , fcil ver que B 2 = 1 Pv1 + + n Pvn = A. A unicidade de B segue da unicidade da decomposiao espectral, Proposiao 5.18, pgina 213. A igualdade (B 2 )B = B(B)2 signica c c a AB = BA, provando que A e B comutam. Denio. Se A uma matriz positiva, a ( nica!) matriz positiva B satisfazendo B 2 = A freq entemente denotada ca e u e u por A e denominada raiz quadrada da matriz A. Como vimos, A A = AA. Lema 5.2 Se A Mat (C, n) uma matriz positiva e C Mat (C, n) satisfaz CA = AC ento C A = AC. e a

JCABarata. Curso de F sica-Matemtica a

Verso de 4 de abril de 2009. a

Cap tulo 5

230/1628

Prova. Se C comuta com A, ento C comuta com qualquer polinmio em A. Vimos na Proposiao 5.17, pgina 213, que a o c a os projetores espectrais de A podem ser escritos como polinmios em A. Assim, C comuta com os projetores espectrais o de A e, portanto, com A, devido a (5.76). Uma conseqncia interessante das consideraoes acima a seguinte proposiao: ue c e c Proposio 5.27 Toda matriz Hermitiana pode ser escrita como combinaao linear de at duas matrizes unitrias. ca c e a Toda matriz pode ser escrita como combinaao linear de at quatro matrizes unitrias. c e a

Demonstrao. Seja A Mat (C, n). Se A Hermitiana (vamos supor que A = , pois de outra forma no h o que se ca e a a provar), ento, para todo w Cn , o produto escalar w A2 w um n mero real e, pela desigualdade de Cauchy-Schwarz, a e u | w A2 w | A2 w 2 n . Assim, A2 w 2 n w, A2 w A2 w 2 n Logo, a matriz A2 / A2 positiva, pois e C C C e u w, ( A2 / A2 )w = w 2 n w, A2 w / A2 w 2 n w 2 n = 0. Conseq entemente, A2 / A2 existe e C C C positiva e Hermitiana. Trivialmente, podemos escrever A = A2 2 A +i A2

A2 A2

A2 2

A i A2

A2 A2

(5.77)

Agora, as matrizes A 2 i
A

A2 A2

so unitrias. Para ver isso, notemos que a a A2 2 A A2 A2

A2 e que

+i

A A2 A i A2

A2 A2

A2 A2

A +i A2

= .

Para provar a ultima igualdade basta expandir o produto e notar que, pelo Lema 5.2, A e A e
A2 A2

A2 A2

comutam, j que a

comutam.

Assim, vemos de (5.77) que uma matriz Hermitiana A combinaao linear de at duas unitrias, provando a primeira e c e a parte da Proposiao 5.27. Para provar a segunda parte, basta notar que se M Mat (C, n) uma matriz qualquer, c e podemos escrever M + M M M M = +i . 2 2i Ambas as matrizes entre parnteses so Hermitianas e, portanto, podem cada uma ser escritas como combinaao linear e a c de at duas unitrias, totalizando at quatro unitrias para M . e a e a

5.5.2

O Teorema de Inrcia de Sylvester. Superf e cies Quadrticas a

Seja M Mat (C, n). Se P Mat (C, n) invers e vel, a transformaao M P M P dita ser uma transformaao c e c de congruncia. Uma transformaao de congruncia representa a transformaao de uma matriz por uma mudana de e c e c c base (justique essa armaao!). c Se M for auto-adjunta, P M P tambm auto-adjunta e, portanto, ambas tm auto-valores reais. Em geral, o e e e conjunto dos auto-valores de M distinto do conjunto dos auto-valores de P M P (exceto, por exemplo, se P for e unitria). Porm, um teorema devido a Sylvester, frequntemente denominado Lei de Inrcia de Sylvester, arma que a e e e uma propriedade do conjunto dos auto-valores preservada em uma transformaao de congruncia, a saber, o n mero de e c e u autovalores, positivos, de autovalores negativos e de autovalores nulos (contando-se as multiplicidades). Enunciaremos e demonstraremos esse teorema logo adiante.

Transformaoes de congruncia em Mat (C, n) c e

JCABarata. Curso de F sica-Matemtica a

Verso de 4 de abril de 2009. a

Cap tulo 5

231/1628

Dada uma matriz auto-adjunta M Mat (C, n), a tripla de n meros (m, m , m0 ), onde m o n mero de autovalores u e u positivos de M , m o n mero de autovalores negativos de M , m0 o n mero de autovalores nulos de M , (em todos os e u e u casos contando-se as multiplicidades) denominada (por razes histricas obscuras) a inrcia da matriz M . Naturalmente, e o o e vale m + m + m0 = n. A Lei de Inrcia de Sylvester arma, portanto, que a inrcia de uma matriz preservada por e e e transformaoes de congruncia. c e Dizemos que duas matrizes A e B Mat (C, n) so congruentes se existir P Mat (C, n) invers a vel tal que A = P BP . E muito fcil provar que a relaao de congruncia uma relaao de equivalncia. a c e e c e E. 5.27 Exerccio. Demonstre essa armao! ca Dessa forma, a Lei de Inrcia de Sylvester arma que a inrcia de matrizes constante nas classes de equivalncia e e e e (pela relaao de congruncia). Assim, leg c e e timo perguntar se as classes de equivalncia so univocamente determinadas e a pela inrcia de seus elementos. A resposta negativa (exceto no caso trivial n = 1), como mostra a argumentaao do e e c pargrafo que segue. a Se A Mat (C, n), com n > 1, uma matriz positiva, A da forma P P (Corolrio 5.3, pgina 229). Assim, e e a a det A = | det P |2 e conclu mos que A invers se e somente se P o for. Conclu disso que a classe de equivalncia e vel -se e (por relaoes de congruncia) que contm a matriz identidade contm todas as matrizes positivas e invers c e e e veis. Pela Proposiao 5.26, pgina 228, esse conjunto coincide com o conjunto de todas as matrizes auto-adjuntas com autovalores c a positivos, ou seja, que possuem inrcia (n, 0, 0). Entretanto, existem tambm matrizes no-auto-adjuntas com inrcia e e a e (n, 0, 0) (por exemplo, matrizes triangulares superiores11 com elementos positivos na diagonal e alguns elementos noa e nulos acima da diagonal). Como tais matrizes no podem ser equivalentes ` identidade (toda matriz da forma P P a a auto-adjunta), conclu mos que as classes de equivalncia no so determinadas univocamente pela inrcia das matrizes e a a e que as compe. o A Lei de Inrcia de Sylvester e

A Lei de Inrcia de Sylvester importante para a classicaao de formas quadrticas e sua relevncia estende-se at e e c a a e a ` classicaao de equaoes diferenciais parciais de segunda ordem. Tratemos de seu enunciado e demonstraao. c c c Teorema 5.16 (Lei de Inrcia de Sylvester) Sejam A e B Mat (C, n) duas matrizes auto-adjuntas. Denotemos e por A+ , A , A0 os subespaos gerados, respectivamente, pelos auto-vetores com autovalores positivos, negativos e nulos c de A (e analogamente para B). Suponhamos que exista P Mat (C, n), inversvel, tal que A = P BP . Ento, dim A+ = dim B+ , dim A = dim B a e dim A0 = dim B0 , onde dim C denota a dimenso de um subespao C Cn . Assim, conclumos tambm que A e B tm a c e e o mesmo nmero de autovalores positivos, o mesmo nmero de autovalores negativos e o mesmo nmero de autovalores u u u nulos (em todos os casos, contando-se as multiplicidades). Prova. Sejam 1 , . . . , a os auto-valores positivos (no necessariamente distintos) e a+1 , . . . , a+a os auto-valores a negativos (no necessariamente distintos) de A. Analogamente, sejam 1 , . . . , b os auto-valores positivos (no necesa a sariamente distintos) e b+1 , . . . , b+b os auto-valores negativos (no necessariamente distintos) de B. Naturalmente, a valem 0 a + a n e 0 b + b n. Se A e B forem nulos no h o que demonstrar, de modo que podemos supor que ambos tm pelo menos um auto-valor a a e no-nulo. Nesse caso, podemos sempre, sem perder em generalidade, supor que A tem pelo menos um autovalor positivo, a pois se tal no for verdade para A ser verdadeiro para A. a a O Teorema Espectral, Teorema 5.5, pgina 211, permite-nos escrever a
a a+a

A =
k=1

k Ak

l=a+1 b+b

|l |Al

e
b

B =
k=1
11 Para

k Bk

l=b+1

|l |Bl ,

(5.78)

a deniao, vide pgina 235 c a

JCABarata. Curso de F sica-Matemtica a

Verso de 4 de abril de 2009. a

Cap tulo 5

232/1628

onde Aj e Bj so os projetores espectrais de A e B, respectivamente. Dena-se a


a a+a

A+ :=
k=1

Ak ,

A :=
l=a+1

Al

A0 := A+ A

e, analogamente,
b b+b

B+ :=
k=1

Bk ,

B :=
l=b+1

Bl

B0 := B+ B .

A+ , A e A0 so, respectivamente, o projetor sobre o subespao de autovetores com auto-valores positivos, negativos e a c nulos de A. Analogamente para B. Esses subespaos so c a A = A Cn , A0 = A0 Cn , B = B Cn , B0 = B0 Cn .

Seja x um vetor no-nulo de A+ . Tem-se que Al x = 0 para todo l > a e Ak x = 0 para pelo menos um k = 1, . . . , a. a Logo, como k > 0 para todo k = 1, . . . , a, segue que
a a a

x, Ax

=
k=1

k x, Ak x

=
k=1

k Ak x, Ak x

=
k=1

k Ak x

>0.

(5.79)

Porm, para um tal x vale tambm e e x, Ax = x, P BP x

= P x, BP x

(5.78) C

b+b

k Bk P x
k=1

l=b+1

|k | Bk P x

Vamos agora supor que B+ < dim A+ (ou seja, que b < a). Armamos que podemos encontrar ao menos um x+ A+ , no-nulo, tal que Bk P x+ = 0 para todo k = 1, . . . , b. Se assim no fosse, no existiria x A+ no-nulo satisfazendo a a a a B+ P x = 0, ou seja, valeria B+ P x = 0 para todo x A+ com x = 0. Logo, (P A+ ) (B+ ) = {0}, o que implica que P A+ B+ . Isso, por sua vez, signica que dimenso do subespao P A+ menor ou igual ` dimenso de B+ e, como P a c e a a invers e vel, isso implica, dim A+ dim B+ , uma contradiao. c Assim, para um tal x+ ter amos
b+b 2

x+ , Ax+

l=b+1

|k | Bk P x+

0,

contradizendo (5.79). Conclu mos disso que dim B+ dim A+ . Como B = (P )1 AP 1 , um racioc anlogo trocando nio a 1 A e B e trocando P P implica que dim A+ dim B+ . Assim, dim B+ = dim A+ . Tambm de forma totalmente anloga prova-se que dim B = dim A (isso tambm pode ser visto imediatamente e a e trocando A A e B B). Isso implica ainda que dim B0 = dim A0 , completando a demonstraao. c

Para matrizes reais agindo no espao Rn valem armaoes anlogas `s obtidas acima. Seja M Mat (R, n). Se c c a a P Mat (R, n) invers e vel, a transformaao M P T M P dita ser uma transformaao de congruncia real, ou c e c e simplesmente transformaao de congruncia. Uma transformaao de congruncia representa a transformaao de uma c e c e c matriz por uma mudana de base (justique essa armaao!). Para transformaoes de congruncia reais vale tambm a Lei c c c e e de Inrcia de Sylvester: se A Mat (R, n) simtrica (ou seja, se A = AT ) sua inrcia preservada por transformaoes e e e e e c de congruncia A P T AP com P Mat (R, n) invers e vel. Como essa armaao um mero caso particular do anterior, c e omitimos a demonstraao e convidamos o estudante a complet-la. c a Classicao de matrizes simtricas em Rn ca e

Transformaoes de congruncia em Mat (R, n) c e

Matrizes simtricas em Rn podem ser classicadas de acordo com o tipo de inrcia que possuem, classicaao essa e e c invariante por transformaoes de congruncia. Uma matriz simtrica A Mat (R, n), n > 1, dita ser c e e e

JCABarata. Curso de F sica-Matemtica a

Verso de 4 de abril de 2009. a

Cap tulo 5

233/1628

1. Parablica, se ao menos um dos seus autovalores for nulo, ou seja, se sua inrcia for da forma (a, a , a0 ) com o e a0 1; 2. Elptica, se todos os seus autovalores forem positivos ou se todos forem negativos, ou seja, se sua inrcia for da e forma (a, a , 0) com a 1 e a = 0 ou com a 1 e a = 0; 3. Hiperblica, se um de seus autovalores for positivo e os demais negativos, ou o oposto: se um de seus autovalores o for negativo e os demais positivos, ou seja, se sua inrcia for da forma (1, a , 0) com a 1 (a, 1, 0) com a 1; e 4. Ultra-Hiperblica, se ao menos dois de seus autovalores forem positivos e ao menos dois forem negativos, nenhum o sendo nulo, ou seja, se sua inrcia for da forma (a, a , 0) com a 2 e a 2. Esse caso s se d se n 4. e o a Essa nomenclatura que classica as matrizes em parablicas, elpticas, hiperblicas e ultra-hiperblicas tem uma moo o o tivaao geomtrica relacionada ` classicaao de superf c e a c cies quadrticas em Rn , assunto que ilustraremos abaixo. a Superf cies quadrticas Rn a

Sejam x1 , . . . , xn so n variveis reais. A forma mais geral de um polinmio real de segundo grau nessas variveis a a o a e
n n n

p(x) =
i=1 j=1

Aij xi xj +
k=1

ck xk + d , + c, x + d, onde,

onde Aij R, ck R e d R. A expresso acima para p pode ser escrita como p(x) = x, Ax a
c1 x1

naturalmente, A a matriz cujos elementos so Aij , c = e a

cn

. . .

ex=

xn

. . .

. A matriz A pode ser sempre, sem perda

de generalidade, escolhida como simtrica. Para ver isso, notemos que, A pode sempre ser escrita como soma de uma e 1 matriz simtrica e uma anti-simtrica: A = 2 (A + AT ) + 1 (A AT ). Contudo, e e 2
n n

x, (A AT )x

=
i=1 j=1

(Aij Aji ) xi xj = 0
R,

1 como facilmente se constata. Assim, a parte anti-simtrica de A, ou seja, 2 (A AT ), no contribui em x, Ax e a 1 T a parte simtrica 2 (A + A ). Portanto, A ser doravante considerada simtrica. e a e

apenas

Estamos agora interessados em classicar as superf cies em Rn denidas por p(x) = , com constante. H primeia ramente dois casos a considerar: 1) A invers e 2) A no invers e vel a e vel. 1. Se A invers e vel, podemos escrever p(x) = x, Ax
R

+ c, x

+d =

1 1 x + A1 c , A x + A1 c 2 2

1 c, A1 c 4

+d.

1 1 e Verique! Assim, a equaao p(x) = ca x + 2 A1 c , A x + 2 A1 c R = , onde a constante + c 1 1 e c, A c R d. A matriz simtrica A pode ser diagonalizada por uma matriz ortogonal, ou seja, podemos 4 escrever A = OT DO, com D = diag (1 , . . . , n ), com k sendo os autovalores de A e O sendo ortogonal. Podemos sempre escolher O de sorte que os primeiros m autovalores 1 , . . . , m so positivos e os demais m+1 , . . . , n a so negativos (no h autovalores nulos, pois A foi suposta invers a a a vel). 1 Com isso, x + 1 A1 c , A x + 2 A1 c 2 n 2 ento, k=1 k yk = ou seja, a R m 2 k yk

= y, Dy

R, n

1 onde y = O x + 2 A1 c . A equaao p(x) = ca, c

k=1

l=m+1

2 |l | yl = .

(5.80)

Temos os seguintes sub-casos a tratar: (a) Se todos os autovalores de A so positivos e > 0, a equaao (5.80) descreve um elipside em Rn (se < 0 a c o no h soluoes e se = 0 a equaao descreve apenas o ponto y = 0 em Rn ). O mesmo vale, reciprocamente, a a c c se todos os autovalores de A forem negativos e < 0 (se > 0 no h soluoes e se = 0 a equaao descreve a a c c apenas o ponto y = 0 em Rn ).

JCABarata. Curso de F sica-Matemtica a

Verso de 4 de abril de 2009. a

Cap tulo 5

234/1628

(b) Se um dos autovalores de A positivo e os demais n 1 so negativos, ou se ocorre o oposto, ou seja, se um dos e a autovalores de A negativo e os demais n 1 so positivos, ento a equaao (5.80) descreve um hiperbolide e a a c o (n 1)-dimensional em Rn no caso = 0. Se > 0 o hiperbolide tem duas folhas (i.e., possui duas componentes conexas) e no caso < 0 apenas uma. o A Figura 5.1, pgina 235, exibe hiperbolides com uma e duas folhas em R3 . a o Devido a sua estabilidade, hiperbolides de uma folha so freq entemente encontrados em estruturas arquio a u tetnicas. A bem conhecida catedral de Braslia, de Niemeyer12 , um exemplo. A estabilidade estrutural o e desse formato decorre do fato que por qualquer ponto de um hiperbolide de uma folha passam duas linhas o retas inteiramente contidas dentro do mesmo (prove isso!). Se = 0 a equaao (5.80) descreve um cone (n 1)-dimensional em Rn . c (c) Este caso ocorre apenas se n 4. Se ao menos dois autovalores de A positivo e ao menos dois so e a positivos a equaao (5.80) descreve, no caso = 0, uma superf (n 1)-dimensional em Rn denominada c cie ultra-hiperbolide. Se = 0 a equaao (5.80) descreve uma (n 1)-dimensional em Rn denominada ultra-cone. o c

2. Se A no invers temos que proceder de modo ligeiramente diferente. Como antes, a matriz simtrica A pode a e vel e ser diagonalizada por uma matriz ortogonal, ou seja, podemos escrever A = OT DO, com D = diag (1 , . . . , n ), com k sendo os autovalores de A e O sendo ortogonal. Como A no tem inversa, alguns de seus autovalores a so nulos. Podemos sempre escolher O de sorte que os primeiros m autovalores 1 , . . . , m so positivos, os m a a a a autovalores seguintes m+1 , . . . , m+m so negativos e os demais m+m +1 , . . . , n so nulos. Naturalmente, 0 m + m < n. Podemos, ento, escrever p(x) = x, Ax R + c, x R + d = y, Dy R + Oc, y R + d onde y = Ox. a Assim, se c = 0 a equaao p(x) = ca c 1 c
m+m 2 |l | yl m

yOc = +

l=m+1

k=1

onde = ( d)/ c e yOc a projeao de y na direao do vetor Oc. Se a dimenso do subespao dos autovalores e c c a c nulos A0 for maior que 1 a equaao (5.81) descrever cilindros de diversos tipos, dependendo do n mero de c a u autovalores positivos e negativos e de Oc ter uma projeao ou no em A0 . No descreveremos os todos os detalhes c a a aqui, mas um exemplo de interesse se d em R3 , se A0 tiver dimenso 2 e Oc for um vetor no-nulo de A0 . Nesse a a a caso equaao (5.81) descreve um cilindro parablico. Vide Figura 5.3, pgina 236. c o a Para o caso em que A0 tem dimenso 1 e Oc um elemento no-nulo desse subespao, a equaao (5.81) descreve a e a c c diversos tipos de parabolides (n 1)-dimensionais. Temos os seguintes casos: o (a) a equaao (5.81) descreve um parabolide elptico (n 1)-dimensional caso todos os autovalores no-nulos de c o a A forem positivos ou se todos os autovalores no-nulos de A forem negativos. Vide Figura 5.2, pgina 5.2. a a (b) A equaao (5.81) descreve um parabolide hiperblico (n1)-dimensional caso um autovalor de A seja negativo c o o e os demais autovalores no-nulos de A sejam positivos (ou o contrrio: caso um autovalor de A seja positivo a a e os demais autovalores no-nulos de A sejam negativos). Vide Figura 5.2, pgina 5.2. a a (c) A equaao (5.81) descreve um parabolide ultra-hiperblico (n 1)-dimensional caso pelo menos dois dos c o o autovalores no-nulos de A sejam positivos e pelo menos dois dos autovalores no-nulos de A sejam negativos. a a Esse caso s pode ocorrer se n 5. o Para c = 0 diversas situaoes acima podem tambm descrever cilindros, por exemplo, se Oc encontra-se no c e subespao dos autovetores com autovalores no-nulos. c a Se c = 0 e dim A0 1, equaao p(x) = ca c
m 2 k yk k=1 m+m

2 k yk ,

(5.81)

l=m+1

2 |l | yl = ,

(5.82)

com = d. A equaao (5.82) descreve diversos tipo de cilindros (n 1)-dimensionais. c (a) Caso c = 0 a equaao (5.82) descreve um cilindro elptico (n 1)-dimensional caso todos os autovalores noc a nulos de A forem positivos ou se todos os autovalores no-nulos de A forem negativos. Vide Figura 5.3, pgina a a 236.
12 Oscar

Niemeyer Soares Filho (1907).

JCABarata. Curso de F sica-Matemtica a

Verso de 4 de abril de 2009. a

Cap tulo 5

235/1628

(b) Caso c = 0 a equaao (5.82) descreve um cilindro hiperblico (n 1)-dimensional caso um autovalor de A seja c o negativo e os demais autovalores no-nulos de A sejam positivos (ou o contrrio: caso um autovalor de A seja a a positivo e os demais autovalores no-nulos de A sejam negativos). Vide Figura 5.3, pgina 236. a a (c) Caso c = 0 a equaao (5.82) descreve um cilindro ultra-hiperblico (n1)-dimensional caso pelo menos dois dos c o autovalores no-nulos de A sejam positivos e pelo menos dois dos autovalores no-nulos de A sejam negativos. a a Esse caso s pode ocorrer se n 5 (lembrar que pelo menos um dos autovalores de A nulo). o e

Figura 5.1: Hiperbolides com uma e duas folhas em R3 . o

Figura 5.2: Um parabolide el o ptico (esquerda) e um parabolide hiperblico (direita) em R3 . o o

5.6

Matrizes Triangulares

Uma matriz S Mat (C, n) dita ser uma matriz triangular superior se forem nulos os elementos abaixo da diagonal e principal, ou seja, se Sij = 0 sempre que i > j. Note que esses no precisam ser necessariamente os unicos elementos a

JCABarata. Curso de F sica-Matemtica a

Verso de 4 de abril de 2009. a

Cap tulo 5

236/1628

Figura 5.3: Um cilindro el ptico (esquerda), um cilindro hiperblico (centro) e um cilindro parablico (direita) em R3 . o o

nulos de S. Uma matriz I Mat (C, n) dita ser uma matriz triangular inferior se forem nulos os elementos acima da diagonal e principal, ou seja, se Iij = 0 sempre que i < j. Note que esses no precisam ser necessariamente os unicos elementos a nulos de I. Proposio 5.28 Matrizes triangulares superiores possuem as seguintes propriedades: ca e 1. A matriz identidade uma matriz triangular superior. 2. O produto de duas matrizes triangulares superiores novamente uma matriz triangular superior. e 3. O determinante de uma matriz triangular superior o produto dos elementos da sua diagonal. Assim, uma matriz e triangular superior inversvel se e somente se no tiver zeros na diagonal. e a 4. Se uma matriz triangular superior inversvel, sua inversa novamente uma matriz triangular superior. e e As armaoes acima permanecem verdadeiras trocando matriz triangular superior por matriz triangular inferior. c

Prova. Os trs primeiros itens so elementares. Para provar o item 4, usa-se a regra de Laplace, expresso (5.17), pgina e a a a 189. Como fcil de se ver, Cof(S)ji = 0 se i > j. Logo, S 1 triangular superior, se existir. e a e As propriedades acima atestam que o conjunto das matrizes n n triangulares superiores invers veis forma um grupo, denominado por alguns autores Grupo de Borel13 de ordem n e denotado por GBn (C). O seguinte resultado sobre matrizes triangulares superiores ser usado diversas vezes adiante. a Lema 5.3 Uma matriz triangular superior S Mat (C, n) normal (ou seja, satisfaz SS = S S) se e somente se for e diagonal. Prova. Se S diagonal, S obviamente normal pois S tambm diagonal e matrizes diagonais sempre comutam entre e e e e si. Provaremos a rec proca, o que ser feito por induao. Para n = 1 no h o que provar. Se n = 2, S da forma a c a a e b S = ( a c ), com a, b, c C. A condiao SS = S S signica c 0 |a|2 + |b|2 cb
13 Armand

bc |c|2

|a|2 ab

ba |b|2 + |c|2

Borel (19232003).

JCABarata. Curso de F sica-Matemtica a

Verso de 4 de abril de 2009. a

Cap tulo 5

237/1628

o que implica b = 0, provando que S diagonal. Procedemos agora por induao, supondo n > 2 e que o lema seja vlido e c a para matrizes (n 1) (n 1) triangulares superiores normais. Se S Mat (C, n) triangular superior, S da forma e e b1 0 a bT . . S= , sendo a C , b = . , = . , . . C bn1 0 |a|2 + bT b bT C Cb CC = |a|2 ab abT B + C C ,

c ambas b e com n 1 linhas, sendo C uma matriz (n 1) (n 1) triangular superior. A condiao SS = S S signica

sendo B a matriz cujos elementos so Bij = bi bj . Disso extra a mos que bT b = 0, ou seja, |b1 |2 + + |bn1 |2 = 0 e, portanto, b = . Com isso, camos com CC = C C, ou seja, C normal. Como C triangular superior ento, pela e e a hiptese indutiva, C diagonal. Isso, mais o fato provado que b nulo, implica que S diagonal, provando o lema. o e e e

5.7

O Teorema de Decomposio de Jordan e a Forma Cannica ca o de Matrizes

Nas seoes anteriores demonstramos condioes que permitem diagonalizar certas matrizes. Nem todas as matrizes, porm, c c e podem ser diagonalizadas. Podemos nos perguntar, no entanto, quo prximo podemos chegar de uma matriz diagonal. a o Mostraremos nesta seao que toda matriz A pode ser levada (por uma transformaao de similaridade) a uma forma c c ` prxima ` diagonal, denominada forma cannica de Jordan14 . Resumidamente (a armaao precisa ser apresentada o a o c a mais adiante), mostraremos que existe uma matriz P tal que P 1 AP tem a seguinte forma: 1 1 0 0 0 0 0 2 2 0 0 0 0 0 3 3 0 0 .. 0 . 0 0 , 0 0 4 (5.83) . . . . . .. .. . . . . . . . . . . . . 0 0 0 0 n1 n1 0 0 0 0 0 n onde 1 , . . . , n so os autovalores de A e onde os a 1 0 0 2 0 0 0 0 . . . . . . 0 0 0 0 e a matriz supra-diagonal 0 0 0 0 . . . 0 0 1 0 0 0 . . . 0 0 i valem 1 ou 0, mas que forma que a matriz diagonal 0 0 0 0 0 0 0 0 3 0 0 0 .. . 0 0 , 0 4 . . . .. .. . . . . . . . . 0 0 n1 0 0 0 0 n 0 0 3 0 . . . 0 0 .. . . .. 0 0 0 0 .. . 0 0 0 , . . . n1 0 0 0 0

(5.84)

0 2 0 0 . . . 0 0

(5.85)

14 Marie Ennemond Camille Jordan (18381922). A forma cannica de matrizes foi originalmente descoberta por Weierstrass (Karl Theodor o Wilhelm Weierstrass (18151897)) e redescoberta por Jordan em 1870.

JCABarata. Curso de F sica-Matemtica a

Verso de 4 de abril de 2009. a

Cap tulo 5

238/1628

comutam entre si. O resultado central que provaremos, e do qual as armativas feitas acima seguiro, diz que toda matriz A pode ser a levada por uma transformaao do tipo P 1 AP a uma matriz da forma D + N , onde D diagonal e N nilpotente (ou c e e seja, tal que N q = 0 para algum q) e tais que D e N comutam: DN = N D. Essa a armativa principal do clebre e e Teorema da Decomposiao de Jordan, que demonstraremos nas pginas que seguem. c a Esse Teorema da Decomposiao de Jordan generaliza os teoremas sobre diagonalizabilidade de matrizes: para matrizes c diagonalizveis tem-se simplesmente N = 0 para um P conveniente. a Antes de nos dedicarmos ` demonstraao desses fatos precisaremos de alguma preparaao. a c c

5.7.1

Resultados Preparatrios o

Somas diretas de subespaos c

Seja V um espao vetorial e V1 e V2 dois de seus subespaos. Dizemos que V a soma direta de V1 e V2 se todo vetor c c e v de V puder ser escrito de modo unico da forma v = v1 + v2 com v1 V1 e v2 V2 . Se V a soma direta de V1 e V2 escrevemos V = V1 V2 . e

subespaos invariantes c

Se V = V1 V2 e tanto V1 quanto V2 so invariantes pela aao de A, escrevemos A = A1 A2 onde Ai A restrita a c e a Vi . Se escolhermos uma base em V da forma {v1 , . . . , vm , vm+1 , . . . , vn }, onde {v1 , . . . , vm } uma base em V1 e e {vm+1 , . . . , vn } uma base em V2 , ento nessa base A ter a forma e a a A = onde A1 Mat (C, m) e A2 Mat (C, n m). E. 5.28 Exerccio. Justique a forma (5.86). A representaao (5.86) dita ser uma representaao em blocos diagonais de A, os blocos sendo as sub-matrizes A1 e c e c A2 . Um fato relevante que decorre imediatamente de (5.86) e da Proposiao 5.1, pgina 193, e que usaremos freq entemente c a u adiante, que se A = A1 A2 ento e a det(A) = det(A1 ) det(A2 ) . Operadores nilpotentes

Um subespao E de Cn dito ser invariante pela aao de uma matriz A, se Av E para todo v E. c e c

nm, m

A1

m, nm
A2

(5.86)

Seja V um espao vetorial e N : V V um operador linear agindo em V . O operador N dito ser um operador c e nilpotente se existir um inteiro positivo q tal que N q = 0. O menor q para o qual N q = 0 dito ser o ndice de N . e Vamos a alguns exemplos. 0 E. 5.29 Exerccio. Verique que N = 0 0 0 E. 5.30 Exerccio. Verique que N = 0 0 1 0 0 1 uma matriz nilpotente de e ndice 3. 0 0 a 0 0 c b com a = 0 e b = 0 uma matriz nilpotente de e ndice 3. 0

JCABarata. Curso de F sica-Matemtica a

Verso de 4 de abril de 2009. a

Cap tulo 5

239/1628

0 E. 5.31 Exerccio. Verique que N = 0 0

0 0 0 1 0 1 e N = 0 0 0 0 0 0

0 0 so matrizes nilpotentes de a ndice 2. 0

O seguinte fato sobre os autovalores de operadores nilpotentes ser usado adiante. a Proposio 5.29 Se N Mat (C, n) nilpotente ento seus autovalores so todos nulos. Isso implica que seu polinmio ca e a a o caracterstico qN (x) = xn , x C. Se o ndice de N q ento o polinmio mnimo de N mN (x) = xq , x C. e e a o e No Corolrio 5.4, pgina 244, demonstraremos que uma matriz nilpotente se e somente se seus autovalores forem a a e todos nulos. Prova da Proposio 5.29. Se N = 0 o ca ndice q = 1 e tudo trivial. Seja N = 0 com e e ndice q > 1. Seja v = 0 um autovetor de N com autovalor : N v = v. Isso diz que 0 = N q v = q v. Logo q = 0 e, obviamente, = 0. E claro ento que qN (x) = xn . Que o polinmio m a o nimo mN (x) = xq segue do fato que mN (x) deve ser um divisor de qn (x) e (isso segue do Teorema 5.2 junto com o Teorema de Hamilton-Cayley, Teorema 5.3), pgina 204). Logo mN (x) da a e forma xk para algum k n. Mas o menor k tal que mN (N ) = N k = 0 , por deniao, igual a q. Isso completa a prova. e c Mais sobre matrizes nilpotentes ser estudado na Seao 5.7.3 onde, em particular, discutiremos a chamada forma a c cannica de matrizes nilpotentes. o O n cleo e a imagem de um operador linear u

Seja V um espao vetorial e A : V V um operador linear agindo em V . c N(A) := {x V | Ax = 0} . A imagem de A denida por e

O ncleo de A denido como o conjunto de todos os vetores que so anulados por A: u e a

R(A) := {x V | y V tal que x = Ay} . Armamos que N(A) e R(A) so dois subespaos de V . Note-se primeiramente que 0 N(A) e 0 R(A) (por que?). a c Fora isso, se x e y N(A) ento, para quaisquer escalares e , a A(x + y) = Ax + Ay = 0 , provando que combinaoes lineares x + x tambm pertencem a N(A). Analogamente se x e x R(A) ento existem c e a y e y V com x = Ay, x = Ay . Logo x + x = A(y + y ) , provando que combinaoes lineares x + y tambm pertencem a R(A). c e Para um operador A xado, e k N, vamos denir Nk = N(Ak ) e Rk = R(Ak ) . Esses subespaos Nk e Rk so invariantes por A. De fato, se x Nk , ento Ak (Ax) = A(Ak x) = A0 = 0, mostrando que c a a Ax Nk . Analogamente, se x Rk ento x = Ak y para algum vetor y. Logo, Ax = A(Ak y) = Ak (Ay), mostrando que a Ax Rk . Armamos que Nk Nk+1 Rk Rk+1 . (5.87)

e que

As demonstraoes dessas armativas so quase banais. Se x Nk ento Ak x = 0. Isso obviamente implica Ak+1 x = 0. c a a Logo x Nk+1 e, portanto, Nk Nk+1 . Analogamente, se x Rk+1 ento existe y tal que x = Ak+1 y. Logo x = Ak (Ay), a o que diz que x Rk . Portanto Rk+1 Rk .

JCABarata. Curso de F sica-Matemtica a

Verso de 4 de abril de 2009. a

Cap tulo 5

240/1628

Isso diz que os conjuntos Nk formam uma cadeia crescente de conjuntos: {0} N1 N2 Nk V , e os Rk formam uma cadeia decrescente de conjuntos: V R1 R2 Rk {0} . (5.89) (5.88)

Consideremos a cadeia crescente (5.88). Como os conjuntos Nk so subespaos de V , claro que a cadeia no pode a c e a ser estritamente crescente se V for um espao de dimenso nita, ou seja, deve haver um inteiro positivo p tal que c a Np = Np+1 . Seja p o menor n mero inteiro para o qual isso acontece. Armamos que para todo k 1 vale Np = Np+k . u

Vamos provar isso. Se x Np+k ento Ap+k x = 0, ou seja, Ap+1 (Ak1 x) = 0. Logo, Ak1 x Np+1 . Dado que a Np = Np+1 , isso diz que Ak1 x Np , ou seja, Ap (Ak1 x) = 0. Isso, por sua vez, arma que x Np+k1 . O que zemos ento foi partir de x Np+k e concluir que x Np+k1 . Se repetirmos a argumentaao k vezes concluiremos que x Np . a c Logo, Np+k Np . Por (5.87) tem-se, porm, que Np Np+k e, assim, Np+k = Np . e Assim, a cadeia (5.88) tem, no caso de V ter dimenso nita, a forma a {0} N1 N2 Np = Np+1 = = Np+k = V . (5.90)

Como dissemos, p ser daqui por diante o menor inteiro para o qual Np = Np+1 . O lema e o teorema que seguem a tm grande importncia na demonstraao do Teorema de Decomposiao de Jordan. e a c c Lema 5.4 Com as denioes acima, Np Rp = {0}, ou seja, os subespaos Np e Rp tm em comum apenas o vetor c c e nulo.

Demonstrao. Seja x tal que x Np e x Rp . Isso signica que Ap x = 0 e que existe y tal que x = Ap y. Logo, ca A2p y = Ap x = 0, ou seja, y N2p . Pela deniao de p tem-se que N2p = Np . Assim, y Np . Logo Ap y = 0. Mas, pela c prpria deniao de y valia que Ap y = x. Logo x = 0. o c Esse lema tem a seguinte conseqncia importante. ue Teorema 5.17 Com as denioes acima vale que V = Np Rp , ou seja, cada x V pode ser escrito de modo unico na c forma x = xn + xr , onde xn Np e xr Rp . Demonstrao. Seja m a dimenso de Np e seja {u1 , . . . , um } uma base em Np . Vamos estender essa base, inca a cluindo vetores {vm+1 , . . . , vn } de modo que {u1 , . . . , um , vm+1 , . . . , vn } seja uma base em V . Armamos que {Ap vm+1 , . . . , Ap vn } uma base em Rp . Seja x Rp e seja y V tal que x = Ap y. Como todo vetor de V , y pode ser e escrito como combinaao linear de elementos da base {u1 , . . . , um , vm+1 , . . . , vn }: c
m n

y =
i=1

i ui +
i=m+1

i vi .

Logo, x =

i Ap ui +
i=1 i=m+1

i Ap vi =
i=m+1

i Ap vi .

(5.91)

Os vetores {Ap vm+1 , . . . , Ap vn } so linearmente independentes. Isso se mostra com o seguinte argumento. Se existirem a
n n n

escalares m+1 , . . . , n tais que


i=m+1

i Ap vi = 0, ento ter a amos Ap


n m i=m+1

i vi

= 0, ou seja,
i=m+1

i vi Np . Isso

implica que existem constantes 1 , . . . , m tais que


i=m+1

i vi =
i=1

i ui , pois os vetores {u1 , . . . , um } so uma base a

em Np . Ora, como {u1 , . . . , um , vm+1 , . . . , vn } so linearmente independentes, segue que os i s e os j s so todos a a

JCABarata. Curso de F sica-Matemtica a

Verso de 4 de abril de 2009. a

Cap tulo 5

241/1628

nulos. Isso prova que {Ap vm+1 , . . . , Ap vn } so linearmente independentes e, portanto, por (5.91), formam uma base a em Rp . Isso incidentalmente provou que a dimenso de Rp n m. Temos, portanto, que dim (Np ) + dim (Rp ) = dim (V ). a e Para i = m + 1, . . . , n dena-se ui = Ap vi . Armamos que o conjunto de vetores {u1 , . . . , um , um+1 , . . . , un } = {u1 , . . . , um , Ap vm+1 , . . . , Ap vn } tambm linearmente independente e, portanto, forma uma base em V . Suponhamos que haja constantes escalares e e 1 , . . . , n tais que
n m n

0 =
i=1

i ui =
i=1

i ui + Ap
i=m+1 n

i vi

Isso implica, obviamente,


m i=1

i ui = Ap

i vi
i=m+1

O lado esquerdo dessa igualdade um elemento de Np (pois u1 , . . . , um so uma base em Np ), enquanto que o lado e a esquerdo obviamente um elemento da imagem de Ap , ou seja, de Rp . Contudo, j vimos (Lema 5.4) que o unico vetor e a que Np e Rp tm em comum o vetor nulo. Logo, e e
m

i ui = 0
i=1

(5.92)

i Ap vi = 0 .
i=m+1

(5.93)

A relaao (5.92) implica 1 = = m = 0, pois {u1 , . . . , um } uma base em Np . A relaao (5.93) implica c e c m+1 = = n = 0, pois {Ap v1 , . . . , Ap vm } uma base em Rp . Assim, todos os i s so nulos, provando que e a {u1 , . . . , um , um+1 , . . . , un } = {u1 , . . . , um , Ap vm+1 , . . . , Ap vn } um conjunto de n vetores linearmente e independentes. Conseq entemente, todo x V pode ser escrito na forma u
n m n

x =
i=1

i ui =
i=1

i ui + Ap
i=m+1 xr Rp xn Np

i vi

Provar a unicidade dessa decomposiao ca como exerc c cio. Isso completa a demonstraao. c Uma das coisas que o teorema que acabamos de demonstrar diz que, dado um operador A, o espao V pode ser e c decomposto em uma soma direta de dois subespaos, invariantes por A: um onde A nilpotente, Np , e outro onde A c e e invers vel, Rp . A nilpotente em Np pois Ap x = 0 para todo elemento x de Np . A invers em Rp pois se x Rp e e vel e tal que Ax = 0 isso implica x N1 Np . Mas x s pode pertencer a Np e a Rp se for nulo. Logo, em Rp , Ax = 0 se o e somente se x = 0, provando que A invers 15. Para referncia futura formulemos essa armativa na forma de um e vel e teorema: Teorema 5.18 Se A um operador linear no-nulo agindo em um espao vetorial V = Cn ento possvel decompor e a c a e V em dois subespaos invariantes por A, V = S T, de forma que A restrito a S nilpotente, enquanto que A restrito a c e T inversvel. e

Esse ser o teorema bsico do qual extrairemos a demonstraao do Teorema de Decomposiao de Jordan. a a c c
15 Lembre-se

que esse argumento s funciona em espaos vetoriais V que tenham dimenso nita, o que estamos supondo aqui. o c a

JCABarata. Curso de F sica-Matemtica a

Verso de 4 de abril de 2009. a

Cap tulo 5

242/1628

5.7.2

O Teorema da Decomposio de Jordan ca

Chegamos agora ao resultado mais importante desta seao, o Teorema da Decomposiao de Jordan16, um importante c c teorema estrutural sobre matrizes de importncia em vrios campos, por exemplo na teoria das equaoes diferenciais a a c ordinrias. Para tais aplicaoes, vide Cap a c tulo 9, pgina 340. a O Teorema da Decomposiao de Jordan tambm tem certa relevncia na Teoria de Grupos, e o usaremos para provar c e a que toda matriz n n complexa invers (ou seja, todo elemento do grupo GL(C, n)) pode ser escrita como exponencial vel de outra matriz (Proposiao 6.11, pgina 283). No Cap c a tulo 6 usaremos o Teorema da Decomposiao de Jordan para c provar a identidade util det(eA ) = eTr(A) , vlida para qualquer matriz n n real ou complexa (Proposiao 6.7, pgina a c a 281). Enunciado e demonstrao do Teorema da Decomposio de Jordan ca ca Teorema 5.19 (Teorema da Decomposio de Jordan) Seja A um operador linear agindo no espao V = Cn e ca c seja {1 , . . . , r } o conjunto de seus autovalores distintos. Ento, existem r subespaos S1 , . . . , Sr tais que V = a c S1 . . . Sr e tais que cada Si invariante por A. Ou seja, A = A1 . . . Ar , onde Ai A restrita a Si . Fora isso, e e cada Ai , da forma Ai = i i + Ni , onde i a matriz identidade em Si e onde Ni nilpotente. Por m, a dimenso e e e a si de cada subespao Si igual a multiplicidade algbrica do autovalor i . c e ` e

Demonstrao. Seja {1 , . . . , r } o conjunto dos autovalores distintos de A e seja ni a multiplicidade algbrica do ca e autovalor i . Seja A1 = A 1 . Pelo Teorema 5.18, pgina 241, V pode ser escrito como V = S1 T1 , onde S1 e T1 a so invariantes por A1 , sendo A1 nilpotente em S1 e invers em T1 . Assim, A1 da forma A1 = N1 M1 com N1 a vel e nilpotente e M1 invers vel. Logo A = 1 + A1 = (1 S1 + N1 ) (1 T1 + M1 ) , (5.94)

onde S1 a matriz identidade em S1 etc. Vamos mostrar que a dimenso de S1 igual ` multiplicidade algbrica de 1 . e a e a e Por (5.94) o polinmio caracter o stico de A e qA () = det( A) = det(( 1 )S1 N1 ) det(( 1 )T1 M1 ) . o stico de N1 , tem-se Se qN1 denota o polinmio caracter det(( 1 )S1 N1 ) = qN1 ( 1 ) = ( 1 )s1 , onde, na ultima igualdade, usamos a Proposiao 5.29, pgina 239, sobre a forma do polinmio caracter c a o stico de uma o stico de M1 . Como matriz nilpotente. Da segue que qA () = ( 1 )s1 qM1 ( 1 ), sendo qM1 o polinmio caracter , e a M1 invers e vel, M1 no tem o zero como autovalor. Logo, qM1 (0) = 0. Portanto s1 igual ` multiplicidade de 1 como a raiz de qA , ou seja, igual a n1 , a multiplicidade algbrica de 1 . e e A idia agora prosseguir decompondo agora o operador 1 T1 + M1 que aparece em (5.94) da mesma maneira como e e zermos acima com A. Evocando novamente o Teorema 5.18, pgina 241, T1 pode ser escrito como T1 = S2 T2 , onde S2 e T2 so invariantes a a por A2 , sendo A2 nilpotente em S2 e invers em T2 . Assim, V = S1 S2 T2 . Agindo em T1 = S2 T2 , A2 da forma vel e A2 = N2 M2 com N2 nilpotente e M2 invers vel. Logo A = 2 T1 + A2 = (2 S2 + N2 ) (2 T2 + M2 ) . Vamos, como acima, mostrar que a dimenso de S2 igual ` multiplicidade algbrica de 2 . a e a e Pela deniao, c Logo,
16 Marie Ennemond

Seja A = 1 T1 + M1 e que age em T1 , que um espao de dimenso n n1 . Denimos A2 = A 2 T1 . e c a

(5.95)

A = (1 S1 + N1 ) A = (1 S1 + N1 ) (2 S2 + N2 ) (2 T2 + M2 ) . qA () = det (( 1 )S1 N1 ) det (( 2 )S2 N2 ) det (( 2 )T2 M2 ) .

Camille Jordan (18381922). A forma cannica de matrizes (que ser discutida mais adiante) foi originalmente descoberta o a por Weierstrass (Karl Theodor Wilhelm Weierstrass (18151897)) e redescoberta por Jordan em 1870.

JCABarata. Curso de F sica-Matemtica a

Verso de 4 de abril de 2009. a

Cap tulo 5

243/1628

Portanto, pelos mesmos argumentos usados acima, qA () = ( 1 )n1 ( 2 )s2 qM2 ( 2 ) . e c Como M2 invers e vel, M2 no tem autovalor zero e, assim, qM2 (0) = 0. Logo, s2 = n2 . T2 assim um subespao de a dimenso n n1 n2 . a

Prosseguindo nas mesmas linhas, aps r passos chegaremos a um subespao Tr de dimenso n n1 nr = 0 o c a (por (5.26), pgina 195). A teremos V = S1 Sr , onde cada Si tem dimenso ni e a , a A = (1 S1 + N1 ) (r Sr + Nr ) , onde os Ni s so todos nilpotentes. Isso completa a demonstraao. a c Um corolrio importante do Teorema de Decomposiao de Jordan o seguinte: a c e Teorema 5.20 Para toda matriz A Mat (C, n) existe uma matriz inversvel P Mat (C, n) tal que P 1 AP = D + N , onde D uma matriz diagonal formada pelos autovalores de A e N uma matriz nilpotente e de tal forma que D e N e e comutam: DN = N D. Conseqentemente, toda matriz A Mat (C, n) pode ser escrita na forma A = Ad + An com Ad An = An Ad , sendo u Ad diagonalizvel e An nilpotente, a saber, Ad = P DP 1 e An = P N P 1 , com D e N dados acima. a

Demonstrao do Teorema 5.20. O Teorema 5.19 est dizendo que, numa base conveniente, A tem a forma de blocos ca a diagonais 0 0 1 s1 + N1 A1 0 0 0 2 s2 + N2 0 0 A2 0 (5.96) A = . , = . . .. . . . . . . .. . . . . . . . . . . . 0 0 Ar 0 0 r sr + Nr ou seja, A = D+N , onde 1 s1 0 D = . . . 0 2 s2 . . . 0 0 .. . 0 0 . . . = diag 1 , . . . , 1 , . . . , r , . . . , r s1 vezes sr vezes 0 N2 . . . 0 .. . 0 0 . . . . (5.97)

Acima si a dimenso do subespao Si . e a c fcil de se ver que N uma matriz nilpotente, pois se o ki o E a e e ndice de Ni (ou seja, ki o menor inteiro positivo e para o qual Niki = 0), ento para k := max (k1 , . . . , kr ) tem-se a (N1 )k 0 0 0 (N2 )k 0 k N = . . . = 0. .. . . . . . . . 0 0 (Nr )k Em verdade, k = max (k1 , . . . , kr ) o e ndice de N (por que?). c Por m, como cada Ni comuta com i si , ca claro que D e N comutam. Isso completa a demonstraao.

r sr N1 0 N = . . . 0

Nr

JCABarata. Curso de F sica-Matemtica a

Verso de 4 de abril de 2009. a

Cap tulo 5

244/1628

Corolrio 5.4 Uma matriz M Mat (C, n) nilpotente se e somente se todos os seus autovalores forem nulos. a e Prova. A Proposiao 5.29, pgina 239, arma que se M nilpotente todos os seus autovalores so nulos. O Teorema c a e a 5.20, pgina 243, arma que se os autovalores de M so nulos, ento existe P tal que P 1 M P = N , nilpotente. Isso a a a implica que M nilpotente. e

5.7.3

Matrizes Nilpotentes e sua Representao Cannica ca o

Os teoremas que estudamos acima nesta seao revelam a importncia de matrizes nilpotentes. Um fato relevante que c a e elas podem ser representadas de uma forma especial, denominada forma cannica, da qual traremos logo abaixo. Antes, o alguma preparaao se faz necessria. c a Seja N Mat (C, n) uma matriz nilpotente de ndice q, ou seja, N q = 0, mas N q1 = 0. Para uso futuro, provemos o seguinte lema: Lema 5.5 Seja N uma matriz nilpotente de ndice q. Esto existe um vetor v = 0 tal que os q vetores a v, N v, N 2 v, ..., N q1 v , (5.98)

so linearmente independentes. Fora isso, o subespao q-dimensional Jv, q := v, N v, N 2 v, . . . , N q1 v de V gerado a c por esses q vetores invariante por N . e

Prova. Se q = 1, ento N = 0 e no h nada a provar, pois a armaao trivialmente verdadeira para qualquer v = 0. a a a c e Seja ento q > 1 (em cujo caso N = 0, trivialmente). Sabemos, por hiptese, que a matriz N q1 no-nula. Isso a o e a signica que existe pelo menos um vetor v = 0 tal que N q1 v = 0. Fixemos um tal vetor. E imediato que os vetores N v, N 2 v, . . . , N q1 v so todos no-nulos pois, se tivssemos N j v = 0 para algum 1 j < q 1, ento, aplicando-se a a e a N q1j ` esquerda, ter a amos N q1 v = 0, uma contradiao. c Sejam agora 1 , . . . , q escalares tais que 1 v + 2 N v + 3 N 2 v + + q N q1 v = 0 . (5.99)

Aplicando-se N q1 nessa igualdade e lembrando que N q = 0, conclu mos que 1 N q1 v = 0. Como N q1 v = 0, segue que 1 = 0 e, com isso, (5.99) ca 2 N v + 3 N 2 v + + q N q1 v = 0 . (5.100) Aplicando agora N q2 nessa igualdade conclu mos que 2 = 0. Prosseguindo, conclu mos depois de q passos que todos os escalares j so nulos. Isso prova que os q vetores de (5.98) so linearmente independentes. a a Que o subespao Jv, q denido acima invariante por N evidente pois, para quaisquer escalares 1 , . . . , q , tem-se c e e N 1 v + 2 N v + + q N q1 v = 1 N v + 2 N 2 v + + q1 N q1 v Jv, q .

O seguinte teorema central para o que segue. e Teorema 5.21 Se N uma matriz nilpotente de ndice q agindo em V e v um vetor com a propriedade que N q1 v = 0, e ento existe um subespao K de V tal que Jv, q K = {0}, tal que V = Jv, q K e tal que K tambm invariante por a c e e N. Prova.17 A prova feita por induao em q. Note-se que se q = 1, ento N = 0 e a armativa trivial, pois podemos e c a e tomar como v qualquer vetor no-nulo, Jv, q seria o subespao gerado por esse v e K o subespao complementar a v, que a c c trivialmente invariante por N , pois N = 0. e
17 Extra da,

com modicaoes, de [70]. c

JCABarata. Curso de F sica-Matemtica a

Verso de 4 de abril de 2009. a

Cap tulo 5

245/1628

Seja V0 = R(N ) o conjunto imagem de N . Sabemos que V0 um subespao de V e que invariante por N . Fora isso, e c e N nilpotente de e ndice q 1 agindo em V0 (por que?) Seja v0 = N v V0 . E claro que N q2 v0 = N q1 v = 0. Assim, pelo Lema 5.5, o subespao (q 1)-dimensional c Jv0 , q1 = v0 , N v0 , . . . , N q2 v0 = N v, N 2 v, . . . , N q1 v = JN v, q1 , que um subespao de V0 , invariante por N e, da hiptese indutiva, conclu e c e o mos que existe um subespao K0 de V0 que c invariante por N tal que JN v, q1 K0 = {0} e tal que V0 = JN v, q1 K0 . e c Seja agora K1 := {x V | N x K0 }. Vamos provar a seguinte armaao: I. Todo vetor x de V pode ser escrito na forma x = y + z onde y Jv, q e z K1 .

Vamos supor ento que a armaao seja vlida para matrizes nilpotentes de a c a ndice q 1 e provar que a mesma vlida e a para matrizes nilpotentes de ndice q. O que desejamos construir um subespao K com as propriedades desejadas, ou e c seja, tal que V = Jv, q K, sendo K invariante por N .

Para provar isso, notemos que para qualquer x V vale certamente que N x V0 . Portanto, como pela hiptese o indutiva V0 = JN v, q1 K0 , podemos escrever N x = y + z , com y JN v, q1 e z K0 . Como y JN v, q1 , y e da forma de uma combinaao linear y = 1 N v + + q1 N q1 v = N y, onde y := 1 v + 2 N v + + q1 N q2 v c um elemento de Jv, q . Logo, z = N (x y). Como z K0 , segue que z := x y K1 . Assim, x = y + z, com e y Jv, q e z K1 . Isso provou I.

Note que a armaao feita em I no signica que V = Jv, q K1 , pois os subespaos Jv, q e K1 podem ter uma c a c intersecao no-trivial. Tem-se, porm, o seguinte: c a e II. Jv, q K0 = {0}.

Provemos essa armaao. Seja x Jv, q K0 . Como x Jv, q , x da forma x = 1 v + 2 N v + + q N q1 v. c e Logo N x = 1 N v + 2 N 2 v + + q1 N q1 v JN v, q1 . Agora, como x K0 e, por hiptese, K0 invariante o e por N , segue que N x K0 . Logo, N x JN v, q1 K0 . Todavia, mencionamos acima que JN v, q1 K0 = {0}. Logo, N x = 0, ou seja, 0 = N x = 1 N v + 2 N 2 v + + q1 N q1 v. Como os vetores N v, . . . , N q1 v so a linearmente independentes, conclu mos que 1 = q1 = 0. Logo, x = q N q1 v. Isso signica que x JN v, q1 . Demonstramos, ento, que se x Jv, q K0 ento x JN v, q1 K0 mas, como JN v, q1 K0 = {0}, segue que a a x = 0. Isso conclui a prova de II.

III. K0 e Jv, q K1 , so dois subespaos disjuntos de K1 . a c A demonstraao muito simples. E evidente que Jv, q K1 subespao de K1 . Como K0 invariante pela aao de c e e c e c N , segue que se x K0 ento N x K0 . Pela deniao, isso diz que x K1 e conclu a c mos que K0 um subespao e c e K1 . Que K0 e Jv, q K1 so subespaos disjuntos, segue do fato que a c K0 (Jv, q K1 ) = K1 (Jv, q K0 ) = K1 {0} = {0} .
A armaao III implica que K1 = (Jv, q K1 ) K0 K0 para algum subespao K0 de K1 (no necessariamente c c a unico). Seja agora K := K0 K0 . Note que K1 = (Jv, q K1 ) K e, portanto, II

(Jv, q K1 ) K = {0} .

(5.101)

Provaremos que esse K possui as propriedades desejadas, ou seja, que V = Jv, q K, sendo K invariante por N . Isso e feito em trs passos. e 1. Jv, q e K so subespaos disjuntos, ou seja, Jv, q K = {0}, pois, como K K1 , segue que K = K K1 e, portanto, a c Jv, q K = Jv, q (K K1 ) = (Jv, q K1 ) K
(5.101)

{0} .

2. Jv, q K contm os vetores de Jv, q e de (Jv, q K1 ) K = K1 . Por I, isso implica que Jv, q K = V . e

JCABarata. Curso de F sica-Matemtica a

Verso de 4 de abril de 2009. a

Cap tulo 5

246/1628

3. K invariante por N , pois o fato que K K1 , implica, pela deniao de K1 , que N K N K1 K0 K. e c A prova do Teorema 5.21 est completa a A principal conseqncia do Teorema 5.21 a seguinte. ue e Proposio 5.30 Seja N Mat (C, n) uma matriz nilpotente de ndice q. Ento, existem ca a 1. um inteiro positivo r, com 1 r n, 2. r nmeros inteiros positivos n q1 q2 qr 1, com q1 + + qr = n, u 3. r vetores v1 , . . . , vr satisfazendo N qj vj = 0 mas N qj 1 vj = 0, j = 1, . . . , r, tais que V = Jv1 , q1 Jvr , qr .

Prova. Se q = 1 ento N = 0. Basta tomar r = n e escolher v1 , . . . , vn uma base qualquer em V . Os qj s so todos a a iguais a 1. Consideremos ento q > 1 com N = 0. Tomemos q1 = q. Pelo Teorema 5.21, existem um vetor v1 = 0 e um subespao a c K 1 , invariante por N tais que V = Jv1 , q1 K 1 . Como K 1 invariante por N , podemos tambm dizer que a matriz N nilpotente quando restrita a K 1 (j que e e e a e nilpotente em todo V ). Denotemos por q2 o ndice de N quando restrita a K 1 . E claro que q2 q = q1 .

Assim, podemos aplicar o Teorema 5.21 para a matriz N restrita a K 1 e concluir que existe v2 = 0 em K 1 e um subespao K 2 de K 1 , invariante por N , tais que K 1 = Jv2 , q2 K 2 . Note que N q2 v2 = 0, pois v2 K 1 . c Com isso, temos V = Jv1 , q1 Jv2 , q2 K 2 .

Novamente K 2 invariante por N e, como K 2 um subespao de K 1 . O e e c ndice de N em K 2 ser q3 q2 q1 . a

O espao V tem dimenso nita. Assim, a prova se conclu repetindo o procedimento acima um n mero nito r de c a u vezes. Note que N qj vj = 0, pois N q1 v1 = 0, e vj K j1 para todo j = 2, . . . , r. Pela construao acima, claro que q1 + + qr = n, a dimenso de V , e que os n vetores c e a v1 , N v1 , . . . , N q1 1 v1 , v2 , N v2 , . . . , N q2 1 v2 , . . . , vr , N vr , . . . , N qr 1 vr

so linearmente independentes e formam uma base em V . Vamos denot-los (na ordem em que aparecem acima) por a a b 1 , . . . , bn . Note agora que, pela construao, N bj = bj+1 , para j em cada um dos conjuntos c {1, . . . , q1 1}, {1 + q1 , . . . , q1 + q2 1}, {1 + q1 + q2 , . . . , q1 + q2 + q3 1} , ... {1 + q1 + + qr1 , . . . , q1 + + qr 1} , (5.102)

com l = 0, . . . , r 1, sendo que N bj = 0 para todo j na forma q1 + + ql , l = 1, . . . , r. u c E. 5.32 Exerccio importante para compreender o que segue. Justique as ltimas armaoes.

JCABarata. Curso de F sica-Matemtica a

Verso de 4 de abril de 2009. a

Cap tulo 5

247/1628

(q 1) vezes
1

}
N =

(q 1) vezes
2

}
0

0
1 0 0
(q 1) vezes
r

0 0 1

Figura 5.4: Forma cannica t o pica de uma matriz nilpotente N . Os elementos da primeira supra-diagonal podem valer 0 ou 1. Todos os demais elementos de matriz so nulos. a

Isso signica que na base b1 , . . . , bn os elementos de matriz de N so todos nulos exceto aqueles na forma Nj, j+1 a com j em algum dos conjuntos listados em (5.102), em cujo caso Nj, j+1 = 1. Pictoriamente, isso diz-nos que na base b1 , . . . , bn a matriz N assume uma forma genericamente ilustrada na Figura 5.4. Essa a denominada forma cannica e o da matriz nilpotente N ou representaao cannica da matriz nilpotente N , que descrevemos mais detalhadamente no que c o segue. Os elementos da diagonal principal so todos nulos. Os unicos elementos no-nulos da matriz podem estar localizados a a apenas na diagonal imediatamente acima da principal, ou seja, aquela diagonal formada por elementos de matriz do tipo Nj, j+1 com j = 1, . . . , n 1. Chamaremos essa diagonal de primeira supra-diagonal. Os elementos da primeira supra-diagonal podem ser 0 ou 1, da forma seguinte: a primeira supra-diagonal possuir r leiras. As primeiras r 1 a leiras so formadas por qj elementos, j = 1, . . . , n 1, sendo os primeiros qj 1 elementos iguais a 1 e o ultimo igual a a 0. A ultima leira ter qr 1 elementos iguais a 1. Assim, se qr = 1, o ultimo elemento da primeira supra-diagonal a ser nulo, proveniente da (r 1)-sima leira (essa a unica forma de aparecer um zero no ultimo elemento da primeira a e e supra-diagonal). Note que zeros consecutivos podem ocorrer, se tivermos alguns qj s iguais a 1. Note tambm que os elementos da e primeira supra-diagonal podem ser todos nulos (o que valer se r = n, em cujo caso q1 = = rn = 1. Isso s pode a o ocorrer se N = 0 e, nesse caso, q = 1) ou todos iguais a 1 (o que valer se r = 1, em cujo caso q1 = n). a

JCABarata. Curso de F sica-Matemtica a

Verso de 4 de abril de 2009. a

Cap tulo 5

248/1628

5.7.4

A Forma Cannica de Matrizes o

Finalizamos esta seao e nossa discusso sobre o Teorema da Decomposiao de Jordan e suas conseqncias reunindo o c a c ue que descobrimos at aqui. e Se A Mat (C, n) o Teorema 5.19, pgina 242 ensinou-nos que numa base conveniente (ou seja, por uma transa 1 formaao de similaridade P0 AP0 ), toda matriz A tem a forma de blocos diagonais: c 1 n1 + N1 0 0 A1 0 0 0 0 2 n2 + N2 0 A2 0 1 P0 AP0 = . (5.103) , . = . .. . . . . . . . .. . . . . . . . . . . 0 0 Ar 0 0 r nr + Nr

sendo 1 , . . . , r os autovalores distintos de A. O j-simo bloco de tamanho nj nj , sendo que nj a multiplicidade e e e algbrica do autovalor j . As matrizes Nj so nilpotentes. e a

c Cada matriz Nj pode ser levada ` sua forma cannica Nj (tal como explicado na Figura 5.4, pgina 247, e no que se a o a 1 lhe segue) em uma base conveniente, ou seja, por uma transformaao de similaridade Pj Nj Pj . Assim, denindo c P1 0 0 0 P2 0 (5.104) P = . . , . .. . . . . . . . 0 0 Pr 1 vemos que P 1 (P0 AP0 )P = (P0 P )1 A(P0 P ), sendo que, por (5.103), 1 P1 (1 n1 + N1 ) P1 0 1 0 P2 (2 n2 + N2 ) P1 1 P 1 (P0 AP0 )P = . . . . . . 0 0

.. .

0 0 . . .
1 Pr (r nr + Nr ) Pr

c 1 n1 + N1 0 . . . 0

0
c 2 n2 + N2

.. .

0 0 . . .
c r nr + Nr

. . . 0

(5.105)

E. 5.33 Exerccio. Complete os detalhes. A matriz nal de (5.105) denominada forma cannica da matriz A, ou forma cannica de Jordan da matriz A. e o o Como dissemos, toda matriz A assume essa forma numa certa base. Devido ao fato de todos as sub-matrizes nilpotentes c Nj terem a forma cannica, os unicos elementos no-nulos da forma cannica da matriz A podem estar ou na diagonal o a o principal (sendo estes os autovalores de A, cada um aparecendo em uma leira de nj elementos), ou na primeira supradiagonal, sendo que estes valem apenas 0 ou 1 e seguem as regras descritas acima. Isso ilustrado na Figura 5.5, e A Figura 5.5, mostra a forma cannica de uma matriz que possui 4 autovalores distintos 1 , 2 , 3 e 4 . A primeira o supra-diagonal formada pela seqncia de n meros e ue u
1 a 1 b 1 c 1 d 1 , . . . , 1 , 0, 1 , . . . , 1 , 0, 1 , . . . , 1 , 0, 1 , . . . , 1 ,

(5.106)

JCABarata. Curso de F sica-Matemtica a

Verso de 4 de abril de 2009. a

Cap tulo 5

249/1628

j sendo que os i assumem apenas os valores 0 ou 1, de acordo com as regras explicadas acima quando discutimos a forma cannica de matrizes nilpotentes. Todos os elementos fora da diagonal principal e da primeira supradiagonal so nulos. o a O primeiro bloco de dimenso (a + 1) (a + 1), o segundo bloco de dimenso (b + 1) (b + 1) etc., sendo a + 1 a e a e a multiplicidade algbrica de 1 , b + 1 a multiplicidade algbrica de 2 etc. e e E interessante notar que na primeira supra-diagonal, sempre ocorrem zeros nos pontos localizados fora dos blocos, ou seja, nos pontos onde ocorrem transioes entre dois autovalores distintos (indicados por setas na Figura 5.5). Esses so c a os zeros que ocorrem explicitamente na lista (5.106).

Por m, comentamos que a forma cannica no exatamente unica, pois poss ainda fazer transformaoes de o a e e vel c similaridade que permutem os blocos de Jordan da matriz. Alm disso, dentro de cada subespao invariante (onde cada e c bloco age) poss fazer certas permutaoes dos elementos da base, de modo a preservar a diagonal e permutar os i s e vel c da primeira supradiagonal.

5.8

Algumas Representaes Especiais de Matrizes co

Nas seoes anteriores apresentamos algumas formas especiais de representar matrizes com determinadas caracter c sticas, como aquelas expressas no Teorema Espectral e no Teorema de Jordan. Nesta seao apresentaremos outras representaes, c co relevantes em certos contextos, como a decomposiao polar. c

5.8.1

A Decomposio Polar de Matrizes ca

E bem conhecido o fato de que todo n mero complexo z pode ser escrito na forma polar z = |z|ei , onde |z| 0 e u a c a a [, ). Tem-se que |z| = zz e ei = z|z|1 . H uma armaao anloga vlida para matrizes A Mat (C, n), a qual muito util, e da qual trataremos nesta seao. Antes de enunciarmos esse resultado de forma mais precisa (o e c Teorema da Decomposiao Polar, Teorema 5.22, abaixo), faamos algumas observaoes preliminares. c c c Seja A Mat (C, n) e seja a matriz A A. Notemos primeiramente que (A A) = A A = A A, ou seja, A A e auto-adjunta. Pelo Teorema 5.13, pgina 226, poss a e vel encontrar um conjunto ortonormal {vk , k = 1, . . . , n} de autovetores de A A, com autovalores dk , k = 1, . . . , n, respectivamente, sendo que a matriz P := v1 , . . . , vn (5.107)

Com esses fatos ` mo, vamos denir uma matriz diagonal, que denotaremos sugestivamente por D1/2 , por D1/2 := a a 2 diag ( d1 , . . . , dn ). Tem-se que D1/2 = D, uma propriedade bvia18 . Note-se tambm que D1/2 = D1/2 , pois o e e u a u cada dk real. Os n meros positivos d1 , . . . , dn so freq entemente denominados valores singulares de A. Denamos agora a matriz A A, por A A := P D1/2 P . (5.108) 2 Essa matriz A A auto-adjunta, pois e A A = P D1/2 P = P D1/2 P = A A. Observemos que A A = P (D1/2 )2 P = P DP = A A. Disso segue que det A A
2

(para a notaao, vide (5.7)) unitria e diagonaliza A A, ou seja, P (A A)P = D, sendo D a matriz diagonal D := c e a diag (d1 , . . . , dn ), cujos elementos da diagonal so os autovalores de A A. Os autovalores dk so todos maiores ou iguais a a a zero. De fato, se vk = 0 um autovetor de A A com autovalor dk , teremos dk vk 2 = dk vk , vk C = vk , Bvk C = e vk , A Avk C = Avk , Avk C = Avk 2 . Logo, dk = Avk 2 / vk 2 0.

= det

A A

= det(A A) = det(A ) det(A) = det(A) det(A) = | det(A)|2 .

A A = | det(A)| e, portanto, A A invers se e somente se A o for. e vel o u Alguns autores denotam a matriz A A por |A|, por analogia com o mdulo de um n mero complexo. Podemos agora formular e demonstrar o resultado que procuramos: Provamos assim que det
no a unica matriz com essa propriedades, pois qualquer matriz do tipo diag ( d1 , . . . , dn ), com os sinais escolhidos a e independentemente uns dos outros, tambm tem como quadrado a matriz D. e
18 Essa

JCABarata. Curso de F sica-Matemtica a

Verso de 4 de abril de 2009. a

Cap tulo 5

250/1628

Teorema 5.22 (Teorema da Decomposio Polar) Seja A Mat (C, n). Ento, existe uma matriz unitria U ca a a Mat (C, n) tal que A = U A A . (5.109) Se A inversvel, ento U univocamente determinada. A representaao (5.109) denominada representaao polar de e a e c e c A. Prova. Sejam, como acima, dk , k = 1, . . . , n os autovalores de A A com autovetores respectivos vk , k = 1, . . . , n. Sabemos pelo Teorema 5.13, pgina 226 que podemos escolher os vk s de forma que vk , vl C = k l . a Como vimos acima, os autovalores dk satisfazem dk 0. Sem perda de generalidade, vamos sup-los ordenados de o forma que dk > 0 para todo k = 1, . . . , r e dk = 0 para todo k = r + 1, . . . , n. Com essa escolha, tem-se que Avk = 0 para todo k = r + 1, . . . , n , pois de A Avk = 0, segue que 0 = vk , A Avk
C

(5.110)

= Avk , Avk

= Avk

Para k = 1, . . . , r, sejam wk os vetores denidos da seguinte forma: 1 wk := Avk , dk a E fcil ver que wk , wl
C

k = 1, . . . , r .

(5.111)

1 = Avk , Avl dk dl

1 = A Avk , vl dk dl

dk = vk , vl dk dl

dk = k l = k l , dk dl

para todos k, l = 1, . . . , r. Assim, o conjunto de vetores {wk , k = 1, . . . , r} forma um conjunto ortonormal. A eles podemos acrescentar um novo conjunto {wk , k = r + 1, . . . , n}, escolhido arbitrariamente, de vetores ortonormais pertencentes ao complemento ortogonal do subespao gerado por {wk , k = 1, . . . , r} e construir assim, um conjunto c ortonormal {wk , k = 1, . . . , n}. Sejam agora a matriz P , denida em (5.107) e as seguintes matrizes de Mat (C, n): Q := w1 , . . . , wn , U := QP

(para a notaao, vide (5.7)). Como {vk , k = 1, . . . , n} e {wk , k = 1, . . . , n} so dois conjuntos ortonormais, segue que c a P e Q so matrizes unitrias (por qu?) e, portanto, U tambm unitria. a a e e e a a ver E fcil que AP = QD1/2 , onde D1/2 def idiag d1 , . . . , dn , De fato, AP
(5.107)

A v1 , . . . , vn

(5.10)

Av1 , . . . , Avn Av1 , . . . , Avr 0, . . . , 0 d1 w1 , . . . , dr wr 0, . . . , 0

(5.110)

(5.111)

(5.12)

w1 , . . . , wn D1/2 = QD1/2 .
(5.108)

Agora, de AP = QD1/2 , segue que A = QD1/2 P = U P D1/2 P

Para mostrar que U univocamente determinado se A for invers e vel, suponhamos que exista U tal que A = U A A = U A A. Como comentamos acima, A A invers se e somente se A o for. Logo, se A invers e vel e vel, a igualdade U A A = U A A implica U = U , estabelecendo a unicidade. Caso A no seja invers a arbitrariedade de U reside a vel na escolha dos vetores ortogonais {wk , k = r + 1, . . . , n}. O seguinte corolrio elementar: a e

e amos provar. U A A, que o que quer

JCABarata. Curso de F sica-Matemtica a

Verso de 4 de abril de 2009. a

Cap tulo 5

251/1628

Teorema 5.23 Seja A Mat (C, n). Ento, existe uma matriz unitria V Mat (C, n) tal que a a A = AA V . Se A inversvel, ento V univocamente determinada. e a e

(5.112)

Prova. Para a matriz A , (5.109) diz-nos que A = U0 (A ) A = U0 AA para alguma matriz unitria U0 . Como a AA auto-adjunta, segue que A = AA U0 . Identicando V = U0 , obtemos o que desejamos. e O Teorema da Decomposiao Polar pode ser generalizado para abranger operadores limitados agindo em espaos c c de Hilbert (vide Teorema 32.22, pgina 1544) e mesmo para abranger operadores no-limitados agindo em espaos de a a c Hilbert (vide [142]).

5.8.2

A Decomposio em Valores Singulares ca

O Teorema da Decomposiao Polar, Teorema 5.22, pgina 250, tem um corolrio de particular interesse. c a a Teorema 5.24 (Teorema da Decomposio em Valores Singulares) Seja A Mat (C, n). Ento, existem maca a trizes unitrias V e W Mat (C, n) tais que a A = V SW , (5.113) onde S Mat n) uma matriz diagonal cujos elementos diagonais so os valores singulares de A, ou seja, os (C, e a autovalores de A A. Prova. A armaao segue imediatamente de (5.109) e de (5.108) tomando V = U P , W = P e S = D1/2 . c O Teorema 5.24 pode ser generalizado para matrizes retangulares. No que segue, m, n N e usaremos as denioes c (5.1), (5.5) e a relaao (5.6) (vide pgina 187) que permitem mapear injetivamente matrizes retangulares em certas c a matrizes quadradas. Teorema 5.25 (Teorema da Decomposio em Valores Singulares. Geral) Seja A Mat (C, m, n). Ento, ca a existem matrizes unitrias V e W Mat (C, m + n) tais que a A = Im, m+n V SW Jm+n, n , (5.114) onde S Mat (C, m + n) uma matriz diagonal cujos elementos diagonais so os valores singulares de A (denida em e a (5.5)), ou seja, os autovalores de (A ) A . Prova. A matriz A Mat (C, m + n) uma matriz quadrada e, pelo Teorema 5.24, possui uma decomposiao em valores e c singulares A = V SW com V e W Mat (C, m + n), unitrias, e S Mat (C, m + n) sendo uma matriz diagonal cujos a elementos diagonais so os valores singulares de A . Com isso, (5.114) segue de (5.6). a Na Seao 5.9, pgina 256, estudaremos uma aplicaao do Teorema da Decomposiao em Valores Singulares, a saber, c a c c ao estudo da chamada Pseudo-Inversa de Moore-Penrose e suas aplicaoes em problemas de optimizaao linear. c c

5.8.3

O Teorema da Triangularizao de Schur ca

O teorema que apresentamos abaixo, devido a Schur19 , semelhante, mas no idntico, ao Teorema de Jordan: toda e a e matriz de Mat (C, n) pode ser levada por uma transformaao de similaridade induzida por uma matriz unitria a uma c a matriz triangular superior (para a deniao, vide Seao 5.6, pgina 235). Esse teorema alternativamente denominado c c a e Teorema da Triangularizaao de Schur ou Teorema da Decomposiao de Schur. Como veremos, esse teorema pode ser c c usado para fornecer uma outra demonstraao (eventualmente mais simples) da diagonalizabilidade de matrizes autoc adjuntas e de matrizes normais por matrizes unitrias. a
19 Issai

Schur (18751941).

JCABarata. Curso de F sica-Matemtica a

Verso de 4 de abril de 2009. a

Cap tulo 5

252/1628

Teorema 5.26 (Teorema da Decomposio de Schur) Seja A Mat (C, n). Ento, existe U Mat (C, n), ca a unitria, e S Mat (C, n), triangular superior, tais que A = U SU . Os elementos da diagonal de S so os autoa a valores de A. Antes de provarmos esse teorema, mencionemos um corolrio evidente: a Corolrio 5.5 Seja A Mat (C, n). Ento, existe V Mat (C, n), unitria, e I Mat (C, n), triangular inferior, tais a a a que A = V IV . Os elementos da diagonal de I so os autovalores de A. a Prova do Corolrio 5.5. Pelo Teorema 5.26, a matriz A pode ser escrita da forma A = V SV , com V unitria e S a a triangular superior. Logo, A = V S V . Porm, S I triangular inferior. e e

Tambm pelo Teorema 5.26, os autovalores de A so os elementos diagonais de S, que so o complexo conjugado e a a dos elementos diagonais de S I. Mas os autovalores de A so o complexo conjugado dos autovalores de A (pela a Proposiao 5.23, pgina 223) e, portanto, so os elementos diagonais de I. c a a

Prova do Teorema 5.26. Comecemos observando que se A = U SU com U unitrio, ento A e S tm o mesmo polinmio a a e o caracter stico e, portanto, os mesmos autovalores, incluindo a multiplicidade (vide a discusso em torno de (5.28), pgina a a e 195). Mas o polinmio caracter o stico de S pS (x) = det(x S) = n (x Skk ), pois S triangular superior e, e k=1 portanto, os autovalores de S so os elementos de sua diagonal. Passemos ` demonstrao da armativa principal, ou a a ca seja, que A = U SU com U unitrio e S triangular superior. a
(1) u1 ,

Seja n 2 e v1 um autovetor de A com autovalor 1 e v1 = 1. Seja U (1) uma matriz unitria da forma U (1) = a . . . , un
(1)

com u1 = v1 , ou seja, cuja primeira coluna o vetor v1 . Ento, e a

(1)

AU

(1) (5.10)

(1) Au1 ,

...,

Au(1) n

(1) 1 u1 ,

(1) Au2 ,

...,

Au(1) n

para certos bk e akl , k, l = 1, . . . , n 1, onde


n1

(1)

(1)

1 0 = U (1) . . . 0

b1 (1) a11 . . . a(n1)1


(1)

(1)

.. .

a1(n1) . . . a(n1)(n1)
(1)

(1)

bn1

(1)

Auk

(1)

= bk u 1 +
l=1

(1) (1)

alk ul+1 ,

(1) (1)

k = 2, . . . , n .

(5.115)

(n1 tendo n 1 linhas) e escrevemos a identidade (5.115) como U (1) AU (1) = Para n = 2 isso demonstra o teorema, pois arma que U (1) AU (1) =

Para simplicar a notaao, denimos c (1) 0 b1 . . (1) n1 = . , b = . , . . (1) 0 bn1

A(1)

a11 . = . . (1) a(n1)1 b(1) A(1)


T

(1)

.. .

a(n1)(n1)

(1)

(1) a1(n1) . , . .

n1

(5.116)

1 0

b1 (1) a11

(1)

sendo o lado direito uma matriz triangular superior. Para n > 2 procedemos por induao. Supondo a armaao vlida c c a para matrizes (n 1) (n 1), ento existe uma matriz unitria V Mat (C, n 1) tal que V A(1) V = S (1) , sendo a a

JCABarata. Curso de F sica-Matemtica a

Verso de 4 de abril de 2009. a

Cap tulo 5

253/1628

S (1) triangular superior. Assim, denindo a matriz unitria U (2) Mat (C, n) por U (2) := a (5.116), U (1) U (2) AU (1) U (2)

n1

T n1
V

, teremos por

= =

U (2) U (1) AU (1) U (2) 1

n1
1

T n1
V

n1
T

b(1) A(1)

n1

T n1
V

n1
1

V T b(1) V A(1) V V T b(1) S (1)

n1

que triangular superior, pois S (1) o . Como U (1) U (2) unitria (pois U (1) e U (2) o so), o teorema est provado. e e e a a a Comentrio. Toda matriz triangular superior S pode ser escrita na forma D + N , sendo D a matriz diagonal formada a pela diagonal de S (ou seja, Dii = Sii para todo i = 1, . . . , n) e N nilpotente (pois triangular superior, mas com e e diagonal nula). Assim, o Teorema 5.26 arma que toda matriz A pode ser levada ` forma D + N por uma transformaao a c de similaridade unitria. Porm, o Teorema 5.26 no garante (nem verdade, em geral) que D e N comutem. Assim, o a e a e Teorema 5.26 distinto do Teorema de Jordan, Teorema 5.20, pgina 243. e a O Teorema 5.26 tem por corolrio o seguinte teorema, j provado anteriormente por outros meios (Teorema 5.13, a a pgina 226, e Proposiao 5.25, pgina 227). a c a Teorema 5.27 Uma matriz A Mat (C, n) auto-adjunta, se e somente se for diagonalizvel por uma transformaao e a c de similaridade unitria e se seus autovalores forem reais. a Prova. Pelo Teorema 5.26, existe uma matriz unitria U tal que U AU = S, sendo S triangular superior cujos elementos a diagonais so os autovalores de A. Assim, se A = A , segue que S = (U AU ) = U A U = U AU = S. Mas para uma a matriz triangular superior S, a igualdade S = S implica que S diagonal e os elementos da diagonal so reais. e a Reciprocamente, se A Mat (C, n) diagonalizvel por uma transformaao de similaridade unitria e seus autovalores e a c a so reais, ou seja, existe U unitria e D diagonal real com U AU = D, ento A = U DU e A = U D U . Como D a a a e diagonal e real, vale D = D e, portanto, A = U DU = A, provando que A auto-adjunta. e Pelo Teorema 5.26, se A Mat (C, n) uma matriz normal e U AU = S, com U unitria e S triangular superior, e a ento S normal (justique!). Assim, junto com o Lema 5.3, pgina 236, provamos o seguinte: a e a Teorema 5.28 Uma matriz A Mat (C, n) normal se e somente se for diagonalizvel por uma transformaao de e a c similaridade unitria. a Essas armaoes foram demonstradas por outros meios no Teorema 5.15, pgina 228. c a

5.8.4

A Decomposio QR e a Decomposio de Iwasawa (KAN) ca ca

O propsito desta seao apresentar a chamada decomposiao de Iwasawa20 , ou decomposiao KAN 21 , de matrizes o c e c c invers veis, Teorema 5.30. Esse teorema tem relaao com a teoria dos grupos de Lie, como discutiremos brevemente c
Iwasawa (19171998). no h uniformidade na literatura quanto a denominaao dessa decomposiao. Vamos cham-la de decomposiao de Iwasawa a a ` c c a c pois a mesma um caso particular (para o grupo GL(C, n) das matrizes complexas n n invers e veis) de um teorema mais geral da teoria dos grupos de Lie, denominado Teorema da Decomposiao de Iwasawa, que arma que todo elemento g de um grupo de Lie semi-simples pode c ser escrito como produto de um elemento k de um subgrupo compacto maximal, por um elemento a de um subgrupo Abeliano (real) e por
21 Infelizmente 20 Kenkichi

JCABarata. Curso de F sica-Matemtica a

Verso de 4 de abril de 2009. a

Cap tulo 5

254/1628

ao nal. Os dois primeiros resultados preparatrios abaixo, Proposiao 5.31 e Teorema 5.29 (Decomposiao QR), tm o c c e interesse por si s. o Proposio 5.31 Seja R Mat (C, n) uma matriz triangular superior cujos elementos diagonais so no-nulos (i.e., ca a a R inversvel). Ento, podemos escrever R = AN , onde A Mat (C, n) a matriz diagonal formada com a diagonal de e a e R: A = diag (R11 , . . . , Rnn ), e N Mat (C, n) uma matriz triangular superior cujos elementos diagonais so iguais e a a 1. a Prova. E fcil constatar que (abaixo m n 1) R = R11 0 . . . 0 0 R12 R22 .. . .. . .. . .. . .. . R1n 0 R2n . = . . . . . 0 Rmn Rnn 0 R11 0 R22 .. . .. . .. . .. .
A

.. .

0 0 . . .

Rmm 0

Rmm 0

0 . . . 0 0 Rnn 0

R12 R11

1 .. .

.. . .. . .. .
N

.. 1 0 .

R1n R11 R2n R22

Rmm

. . . . Rmn 1

O estudante deve comparar as armaoes do teorema a seguir com o Teorema da Decomposiao Polar, Teorema 5.22, c c pgina 250, e com o Teorema da Decomposiao de Schur, Teorema 5.26, pgina 252. a c a Teorema 5.29 (Teorema da Decomposio QR) Seja M Mat (C, n) uma matriz inversvel. Ento, M pode ser ca a escrita na forma M = QR, onde Q Mat (C, n) unitria e R Mat (C, n) triangular superior, sendo que os e a e elementos diagonais de R so estritamente positivos. a e vel, os vetores mk , k = 1, . . . , n, so linearmente a Prova do Teorema 5.29. Seja M = m1 , . . . , mn . Como M invers independentes, ou seja, formam uma base em Cn . Podemos, portanto, usar o procedimento de ortogonalizaao de Gramc Schmidt (vide Seao 3.3, pgina 161) e construir uma nova base ortonormal de vetores qj , j = 1, . . . , n, a partir dos c a vetores ml , l = 1, . . . , n. Tais vetores so denidos por a
j1

q1 =

m1 , m1

qj =

mj mj

ql , mj
l=1 j1

ql , j = 2, . . . , n . ql

ql , mj
l=1

Como fcil vericar, tem-se qi , qj e a m1 = q1 m1 ,

= i j para todos i, j = 1, . . . , n. As relaoes acima implicam trivialmente c


j1 j1

mj = qj mj

ql , mj
l=1

ql +
l=1

ql ql , mj

j = 2, . . . , n ,

um elemento n de um subgrupo nilpotente (ou seja, cuja algebra de Lie nilpotente): g = kan. Em Alemo, as palavras compacto, Abeliano e a e nilpotente so Kompakt, Abelsch e Nilpotent, da a denominaao decomposiao KAN para essa decomposiao, denominaao essa a c c c c encontrada em alguns textos.

JCABarata. Curso de F sica-Matemtica a

Verso de 4 de abril de 2009. a

Cap tulo 5

255/1628

com

relaoes estas que podem ser escritas em forma matricial como c R11 q1 , m2 0 R22 . .. m1 , . . . , mn = q1 , . . . , qn R , onde R := . . . 0 0
j1

.. .. .. . . .

.. .

q1 , mn

R(n1)(n1) 0

ql ,

q2 , mn C . . , (5.117) . qn1 , mn C Rnn

R11 =

m1 ,

Rjj =

mj

ql , mj
l=1

j = 2, . . . , n .

c ca E. 5.34 Exerccio. Convena-se da validade da relao (5.117). c e Denindo Q := q1 , . . . , qn , a relaao (5.117) diz-nos que M = QR, sendo R triangular superior (como se v) e Q unitria (pois os vetores ql , l = 1, . . . , n, so ortonormais). Isso completa a prova do Teorema 5.29. a a Chegamos assim ao importante Teorema da Decomposiao de Iwasawa para matrizes invers c veis: Teorema 5.30 (Teorema da Decomposio de Iwasawa, ou Decomposio KAN ) Seja M Mat (C, n) uma ca ca e matriz inversvel. Ento, M pode ser escrita de modo unico na forma M = KAN , onde K Mat (C, n) uma matriz a unitria, A Mat (C, n) a uma matriz diagonal, tendo elementos diagonais estritamente positivos, e N Mat (C, n) a e uma matriz triangular superior cujos elementos diagonais so iguais a 1. e a Prova. A armaao que M pode ser escrita na forma M = KAN , com K, A e N com as propriedades acima segue c imediatamente da Proposiao 5.31 e do Teorema 5.29, dispensando demonstraao. O unico ponto a se demonstrar a c c e unicidade dessa decomposiao. c Vamos ento supor que para algum M Mat (C, n) existam K, K0 Mat (C, n), matrizes unitrias, A, A0 a a Mat (C, n), matrizes diagonais, tendo elementos diagonais estritamente positivos, e N, N0 Mat (C, n) matrizes triangulares superiores cujos elementos diagonais so iguais a 1, tais que M = KAN = K0 A0 N0 . a
1 Segue imediatamente disso que K0 K = A0 N0 N 1 A1 . O lado esquerdo dessa igualdade uma matriz unitria e a e, portanto, normal. O lado direito uma matriz triangular superior (pela Proposiao 5.28, pgina 236). Pelo Lema e c a 1 5.3, pgina 236, A0 N0 N 1 A1 deve ser uma matriz diagonal D. Assim, temos que K0 K = D e A0 N0 N 1 A1 = D. a A primeira dessas relaoes diz-nos que D unitria. A segunda diz-nos que N0 N 1 = A1 DA, ou seja, N0 = D0 N , c e a 0 e e a onde D0 := A1 DA diagonal (por ser o produto de trs matrizes diagonais). Agora, N e N0 so matrizes triangulares 0 superiores cujos elementos diagonais so iguais a 1. Portanto, a relaao N0 = D0 N com D0 diagonal s poss se a c o e vel D0 = (de outra forma haveria elementos na diagonal de N ou de N0 diferentes de 1), estabelecendo que N = N0 .

Provamos, assim, que A1 DA = , ou seja, D = A0 A1 . Agora, A e A0 so diagonais, tendo na diagonal n meros a u 0 reais positivos. Logo, D tambm diagonal e tem na diagonal n meros reais positivos e, portanto, D = D . Como e e u D unitria (como observado linhas acima), segue que D2 = . Logo, os elementos Dkk da diagonal de D satisfazem e a Dkk = 1, para todo k = 1, . . . , n (os sinais podendo ser distintos para ks distintos). Agora, como A0 = DA e como A e A0 tm na diagonal n meros reais positivos, no podemos ter Dkk = 1 para algum k e, portanto, D = . e u a Conseq entemente, K = K0 e A = A0 , estabelecendo a unicidade desejada. u Note o leitor que o conjunto das matrizes unitrias de Mat (C, n) forma um subgrupo de GL(C, n) (o grupo das a matrizes complexas n n invers veis). O conjunto das matrizes diagonais de Mat (C, n) tendo elementos diagonais estritamente positivos igualmente um subgrupo de GL(C, n). Por m, o conjunto das matrizes triangulares superiores e de Mat (C, n) cujos elementos diagonais so iguais a 1 tambm um subgrupo de GL(C, n). Assim, o Teorema 5.30 a e e

JCABarata. Curso de F sica-Matemtica a

Verso de 4 de abril de 2009. a

Cap tulo 5

256/1628

arma que cada elemento de GL(C, n) pode ser escrito de modo unico como produto de elementos de cada um desses trs subgrupos. Esse um caso particular de um teorema da teoria dos grupos de Lie conhecido como Teorema da e e Decomposiao de Iwasawa. c

5.9

A Pseudo-Inversa de Moore-Penrose. Optimizao Lica near

Na presente seao introduziremos uma generalizaao especial da noao de inversa de matrizes, a qual aplica-se mesmo a c c c matrizes no-quadradas. O conceito que descreveremos, a chamada pseudo-inversa de Moore-Penrose, particularmente a e util no tratamento de problemas de optimizaao linear, como discutiremos adiante (Seao 5.9.2, pgina 264), ou seja, c c a em problemas onde procura-se soluoes optimalmente aproximadas de sistemas de equaoes lineares como Ax = y, onde c c A uma matriz m n dada, y um vetor-coluna, dado, com m componentes e x, a incgnita do problema, um um e o e vetor-coluna com n componentes. Em tais problemas procura-se vetores x tais que a norma de Ax y seja a menor poss e que representem, portanto, no necessariamente a soluao exata do sistema Ax = y (que pode no existir), vel a c a mas a melhor aproximaao em termos de m c nimos quadrados ao que seria a soluao. c Inversas generalizadas, ou pseudo-inversas

Sejam m, n N e seja uma matriz (no necessariamente quadrada) A Mat (C, m , n). Uma matriz B a Mat (C, n, m) dita ser uma inversa generalizada, ou pseudo-inversa, de A, se satiszer as seguintes condioes: e c 1. ABA = A, 2. BAB = B. O leitor h de notar que se A Mat (C, n) uma matriz quadrada invers a e vel, sua inversa A1 satisfaz trivialmente as propriedades denidoras da inversa generalizada. Provaremos mais adiante que toda matriz A Mat (C, n) possui ao menos uma inversa generalizada, a saber, a pseudo-inversa de Moore-Penrose. Com a generalidade da deniao acima, c porm, no se pode garantir a unicidade da inversa generalizada de A. e a Com a amplitude da deniao acima, a noao inversa generalizada no muito util, mas certos tipos mais espec c c a e cos de inversas generalizadas so de interesse em certos tipos de problemas. No que segue discutiremos a chamada pseudoa inversa de Moore-Penrose e seu emprego em problemas de optimizaao linear. c Denio da pseudo-inversa de Moore-Penrose de uma matriz ca

Sejam m, n N e seja uma matriz (no necessariamente quadrada) A Mat (C, m , n). Uma matriz A+ a Mat (C, n, m) dita ser uma pseudo-inversa de Moore-Penrose de A se satiszer as seguintes condioes: e c 1. AA+ A = A, 2. A+ AA+ = A+ , 3. AA+ Mat (C, m) e A+ A Mat (C, n) so auto-adjuntas. a O leitor h de notar que se A Mat (C, n) uma matriz quadrada invers a e vel, sua inversa A1 satisfaz trivialmente as propriedades denidoras da pseudo-inversa de Moore-Penrose. A noao de pseudo-inversa descrita acima foi introduzida por E. H. Moore22 em 1920 e redescoberta por R. Penrose23 c em 1955. O conceito de pseudo-inversa de Moore-Penrose util para a resoluao de problemas de optimizaao lineares, e c c ou seja, ` determinaao da melhor aproximaao em termos de m a c c nimos quadrados ` soluao de sistemas lineares. a c Trataremos desses aspectos mais adiante (vide Teorema 5.33, pgina 264), aps demonstrarmos resultados sobre existncia a o e e unicidade. Outros desenvolvimentos da teoria das pseudo-inversas de Moore-Penrose e suas aplicaoes, podem ser c encontrados em [14]. Vide tambm as referncias originais: E. H. Moore, On the reciprocal of the general algebraic e e
22 Eliakim 23 Sir

Hastings Moore (18621932). Roger Penrose (1931).

JCABarata. Curso de F sica-Matemtica a

Verso de 4 de abril de 2009. a

Cap tulo 5

257/1628

matrix. Bulletin of the American Mathematical Society 26, 394395 (1920); R. Penrose, A generalized inverse for matrices, Proceedings of the Cambridge Philosophical Society 51, 406413 (1955) e R. Penrose, On best approximate solution of linear matrix equations, Proceedings of the Cambridge Philosophical Society 52, 1719 (1956). Nas pginas que seguem demonstraremos que toda a matriz A Mat (C, m, n) possui uma pseudo-inversa de Moorea Penrose, a qual unica. Comeamos com a questo da unicidade para em seguida tratarmos de propriedades gerais e, e c a posteriormente, da questo da existncia. As aplicaoes em problemas de optimizaao so discutidas na Seao 5.9.2, a e c c a c pgina 264. a A unicidade da pseudo-inversa de Moore-Penrose

Seja A+ Mat (C, n, m) uma pseudo-inversa de Moore-Penrose de A Mat (C, m, n) e seja B Mat (C, n, m) uma outra pseudo-inversa de Moore-Penrose de A, ou seja, tal que ABA = A, BAB = B com AB e BA auto-adjuntas. o e c Seja M1 := AB AA+ = A(B A+ ) Mat (C, m). Pelas hipteses M1 auto-adjunta (por ser a diferena de duas matrizes auto-adjuntas) e (M1 )2 = (AB AA+ )A(B A+ ) = (ABA AA+ A)(B A+ ) = (A A)(B A+ ) = 0. Como M1 auto-adjunta, o fato que (M1 )2 = 0 implica M1 = 0 (lembrar que, por ser auto-adjunta, M1 diagonalizvel). e e a Isso provou que AB = AA+ . Analogamente, prova-se que BA = A+ A (para tal, considere-se a matriz auto-adjunta M2 := BA A+ A Mat (C, n) e proceda-se como acima). Agora, tudo isso implica que A+ = A+ AA+ = A+ (AA+ ) = A+ AB = (A+ A)B = BAB = B, provando a unicidade. Como j comentamos, se A Mat (C, n) uma matriz quadrada invers a e vel, sua inversa A1 satisfaz trivialmente as propriedades denidoras da pseudo-inversa de Moore-Penrose e, portanto, tem-se nesse caso A+ = A1 , univocamente. E tambm evidente pela deniao que para mn , a matriz m n identicamente nula, vale (mn )+ = nm . e c Apresentaremos no que seguir duas demonstraoes da existncia da pseudo-inversa de Moore-Penrose de matrizes a c e arbitrrias de Mat (C, m, n). Ambas as demonstraoes permitem produzir algoritmos para sua determinaao expl a c c cita da pseudo-inversa de Moore-Penrose. Uma primeira demonstraao ser apresentada na Seao 5.9.1.1, pgina 260, (vide c a c a o Teorema 5.31, pgina 262, e o Teorema 5.32, pgina 263) e decorrer de diversos resultados que estabeleceremos a a a a seguir. Destacamos particularmente as expresses (5.139) e (5.140), as quais permitem calcular a pseudo-inversa de o Moore-Penrose A+ de uma matriz A Mat (C, m, n) diretamente em termos de A, A e dos autovalores de AA ou de A A (ou seja, dos valores singulares de A). Uma segunda demonstraao ser apresentada na Seao 5.9.3, pgina 265, e para a mesma faremos uso da decomposiao c a c a c em valores singulares apresentada no Teorema 5.24, pgina 251. A essa segunda demonstraao da Seao 5.9.3 o leitor a c c interessado poder passar sem perdas neste ponto. Os resultados da Seao 5.9.3, porm, no sero usados no que segue. a c e a a Essa segunda demonstraao a mais freq entemente apresentada na literatura, mas cremos que as expresses (5.139) e c e u o (5.140) fornecem um mtodo algoritmicamente mais simples para a determinaao da pseudo-inversa de Moore-Penrose e c de uma matriz geral. Calculando a pseudo-inversa de Moore-Penrose em casos particulares A existncia da pseudo-inversa de Moore-Penrose e

Demonstremos a unicidade da pseudo-inversa de Moore-Penrose de uma matriz A Mat (C, m, n), caso exista.

Se A Mat (C, m, n), ento A Mat (C, n, m) denida como a matriz cujos elementos (A )ij so dados por Aji a e a para todos 0 i n e 0 j m. Futuramente obteremos as expresses (5.139) e (5.140), as quais permitem calcular a o pseudo-inversa de Moore-Penrose A+ Mat (C, n, m) de uma matriz A Mat (C, m, n) diretamente em termos de A, A e dos autovalores de AA ou de A A. Nos exerc cios que seguem indicaremos situaoes especiais mas uteis nas quais c a pseudo-inversa de Moore-Penrose pode ser calculada de modo relativamente simples.
a1

E. 5.35 Exerccio. Constate que se A Mat (C, m, 1), A =


1 A
2 C

am

. . .

, um vetor-coluna no-nulo, ento A+ = a a

1 A

2 C

A =

( a1 , ..., am ), onde A

|a1 |2 + + |am |2 .

Observe-se que se z C, podemos considerar z como uma matriz complexa 1 1, ou seja, como elemento de 0, z = 0 Mat (C, 1, 1) e, com isso, obtemos do exposto acima (z)+ = . 1 z, z = 0

JCABarata. Curso de F sica-Matemtica a

Verso de 4 de abril de 2009. a

Cap tulo 5

258/1628

O resultado do Exerc E. 5.35 pode ser generalizado. cio E. 5.36 Exerccio. Seja A Mat (C, m, n). Mostre que se (AA )1 existe, ento A+ = A (AA )1 . Mostre que se a (A A)1 existe, ento A+ = (A A)1 A . Sugesto: em ambos os casos, verique que o lado direito satisfaz as propriedades a a denidoras da pseudo-inversa de Moore-Penrose e use a unicidade. Os resultados do Exerc E. 5.36 podem ser generalizados para situaoes em que AA ou A A no so invers cio c a a veis + + + pois, como veremos na Proposiao 5.33, pgina 259 valem sempre as relaoes A = A AA c a c = A A A . Tambm e o Teorema 5.31, pgina 262, apresentar uma generalizaao dos resultados do Exerc E. 5.36, mostrando uma outra a a c cio forma de proceder quando AA ou A A no forem invers a veis. Os exerc cios que seguem contm aplicaoes dos resultados do Exerc E. 5.36. e c cio
i E. 5.37 Exerccio. Seja A = ( 2 0 1 ), com A = 0 i 2 0 0 i 1 1

. Mostre que AA possui inversa, mas que A A no possui. a


1 9 4 2i 1 5i i 4

Usando o Exerc E. 5.36, calcule a pseudo-inversa de Moore-Penrose A+ de A, obtendo A+ = cio essa A satisfaz de fato as propriedades denidoras da pseudo-inversa de Moore-Penrose. E. 5.38 Exerccio. Seja A =
12 0 i 03 +

. Verique que

, com A =

1 0 0 2 i 3

. Mostre que AA no possui inversa, mas que A A possui. a


1 10 10 2i 6 0 i 3

Usando o Exerc E. 5.36, calcule a pseudo-inversa de Moore-Penrose A+ de A, obtendo A+ = cio essa A+ satisfaz de fato as propriedades denidoras da pseudo-inversa de Moore-Penrose.

. Verique que

5.9.1

Outras Propriedades da Pseudo-Inversa de Moore-Penrose

As seguintes propriedades da pseudo-inversa de Moore-Penrose seguem das denioes e da unicidade. Suas demonstraoes c c so elementares e so deixadas como exerc a a cio: para A Mat (C, m, n) valem 1. (A+ )+ = A, 2. (A+ ) = AT
T + +

, A+ = A

e, conseq entemente, (A+ ) = (A )+ , u

3. (zA)+ = z 1 A+ para todo z C no-nulo. a E de se observar, porm, que se A Mat (C, m, n) e B Mat (C, n, p), nem sempre (AB)+ dada por B + A+ , e e ao contrrio do que ocorre com a inversa usual (para o caso m = n = p). Uma exceao relevante ser encontrada na a c a Proposiao 5.33, pgina 259. c a A seguinte proposiao lista mais algumas propriedades importantes, algumas das quais usaremos logo adiante: c Proposio 5.32 A pseudo-inversa de Moore-Penrose satisfaz as seguintes relaoes ca c A+ = A+ (A+ ) A , A A (A+ ) , A A A+ , A (A+ ) A+ , (A+ ) A A , A+ A A , (5.118) (5.119) (5.120) (5.121) (5.122) (5.123)

A = A A+ = =

A = A vlidas para toda A Mat (C, m, n). a =

JCABarata. Curso de F sica-Matemtica a

Verso de 4 de abril de 2009. a

Cap tulo 5

259/1628

Prova. Por AA+ ser auto-adjunta, vale AA+ = (AA+ ) = (A+ ) A . Multiplicando-se ` esquerda por A+ obtemos a A+ = A+ (A+ ) A , provando (5.118). Substituindo-se A A+ e usando o fato que A = (A+ )+ , obtm-se de (5.118) e que A = AA (A+ ) , que a relaao (5.119). Substituindo-se A A e usando o fato que (A )+ = (A+ ) , obtm-se de e c e (5.119) que A = A AA+ que a relaao (5.120). e c As relaoes (5.121)-(5.123) podem ser obtidas analogamente a partir do fato de A+ A ser tambm auto-adjunta, mas c e mais fcil obt-las substituindo-se A A em (5.118)-(5.120) e tomando-se o adjunto das expresses resultantes. e a e o Da Proposiao 5.32 podem ser obtidos vrios resultados de interesse, alguns dos quais encontram-se reunidos na c a proposiao que segue. c Proposio 5.33 Para a pseudo-inversa de Moore-Penrose vale ca AA para todo A Mat (C, m, n). Disso obtm-se que e A+ = A AA tambm para todo A Mat (C, m, n). e A expresso (5.125) generaliza os resultados do Exerc E. 5.36, pgina 258 e pode ser empregada para calcular A+ a cio a + + desde que AA ou A A sejam previamente conhecidas. Prova da Proposio 5.33. Seja B = A ca AA
(5.119) + +

A+

(5.124)

A A

A ,

(5.125)

A+ . Tem-se
(5.123)

A A (A+ ) A

A A (A+ ) A+ A A = (AA )B(AA ) ,

onde usamos tambm que A e B = A


+

= A+ . Tem-se tambm que e = (A+ ) A+ A A+ A A B =


(5.121)

A+

(5.118)

(A+ ) A+ A A (A+ ) A+ = B A A B .
(5.120)

Observe-se tambm que e A A (A+ ) A+ = AA+ que auto-adjunto, por deniao. Analogamente, e c B A A que tambm auto-adjunto, por deniao. e e c Os fatos expostos nas linhas acima provaram que B a pseudo-inversa de Moore-Penrose de AA , provando (5.124). e Substituindo-se A A em (5.124) obtm-se tambm e e A A Observe-se agora que A AA e que A A provando (5.125). A pseudo-inversa de Moore-Penrose, o n cleo e a imagem de uma matriz u
+ + (5.124) + +

= (A+ ) A+ A A

(5.122)

(A )+ A

= A+ A A A A+ A
+

.
(5.121)

(5.126) = A+ A+ ,

A+

(5.126)

(5.118)

Denimos o n cleo e a imagem (range) de uma matriz A Mat (C, m, n) por Ker (A) := {u Cn | Au = 0} e u Ran (A) := {Au, u Cn }, respectivamente. E evidente que Ker (A) um subespao linear de Cn e que Ran (A) um e c e subespao linear de Cm . c A seguinte proposiao ser usada logo adiante, mas de interesse por si s. c a e o

JCABarata. Curso de F sica-Matemtica a

Verso de 4 de abril de 2009. a

Cap tulo 5

260/1628

Proposio 5.34 Seja A Mat (C, m, n) e sejam denidos P1 := n A+ A Mat (C, n) e P2 := m AA+ ca Mat (C, n). Ento valem as seguintes armaoes: a c
1. P1 e P2 so projetores ortogonais, ou seja, satisfazem (Pk )2 = Pk e Pk = Pk , k = 1, 2. a

2. Ker (A) = Ran (P1 ), Ran (A) = Ker (P2 ), Ker (A+ ) = Ran (P2 ) e Ran (A+ ) = Ker (P1 ). 3. Ran (A) = Ker (A+ ) e Ran (A+ ) = Ker (A) . 4. Ker (A) Ran (A+ ) = Cn e Ker (A+ ) Ran (A) = Cm , ambas somas diretas de subespaos ortogonais. c Prova. Que P1 e P2 so auto-adjuntos segue do fato de AA+ e A+ A o serem. Tem-se tambm que (P1 )2 = 2A+ A + a e A+ AA+ A = 2A+ A + A+ A = A+ A = P1 e analogamente para P2 . Isso provou o item 1.

Seja x Ker (A). Como Ran (P1 ) um subespao linear fechado de Cn , o Teorema do Melhor Aproximante e o e c Teorema da Decomposiao Ortogonal (que neste texto so apresentados com toda generalidade no contexto de espaos c a c de Hilbert, como Cm na forma do Teorema 31.1, pgina 1468, e do Teorema 31.2, pgina 1470, respectivamente) a a garantem-nos a existncia de um unico z0 Ran (P1 ) tal que x z0 Cm m e e nimo. Alm disso, x z0 ortogonal a e e Ran (P1 ). Assim, existe ao menos um y0 Cm tal que x P1 y0 ortogonal a todo elemento da forma P1 y, ou seja, e x P1 y0 , P1 y C = 0 para todo y Cm , o que implica P1 (x P1 y0 ), y C = 0 para todo y Cm , o que por sua vez implica P1 (x P1 y0 ) = 0. Isso, porm, arma que P1 x = P1 y0 . Como x Ker (A) vale P1 x = x (pela deniao de P1 ). e c Provamos portanto que se x Ker (A) ento x Ran (P1 ), estabelecendo que Ker (A) Ran (P1 ). Por outro lado, o a fato que AP1 = A( A+ A) = A A = 0 implica que Ran (P1 ) Ker (A), provando que Ran (P1 ) = Ker (A). Se z Ker (P1 ), ento z = A+ Az, provando que z Ran (A+ ). Isso provou que Ker (P1 ) Ran (A+ ). Por outro a lado, se u Ran (A+ ) ento existe v Cm tal que u = A+ v. Logo, P1 u = (n A+ A)A+ v = (A+ A+ AA+ )v = 0, a provando que u Ker (P1 ) e que Ran (A+ ) Ker (P1 ). Isso estabeleceu que Ker (P1 ) = Ran (A+ ). P2 obtida de P1 com a substituiao A A+ (lembrando-se que (A+ )+ = A). Logo, os resultados de acima implicam e c que Ran (P2 ) = Ker (A+ ) e que Ker (P2 ) = Ran (A). Isso provou o item 2.

Se M Mat (C, p) (com p N, arbitrrio) auto-adjunta, ento y, M x C = M y, x C para todos x, y Cp . Essa a e a relaao torna evidente que Ker (M ) = Ran (M ) (justique!). Com isso o item 3 segue do item 2 tomando-se M = P1 e c M = P2 . O item 4 evidente pelo item 3. e E. 5.39 Exerccio. Calcule P1 e P2 para o exemplo do Exerc E. 5.37, pgina 258, e para o exemplo do Exerc E. 5.38, cio a cio pgina 258. a

5.9.1.1

A Regularizao de Tikhonov. Existncia ca e

Pelo dito acima, podemos substituir as matrizes AA ou A A, caso sejam singulares, pelas matrizes invers veis AA + c c o ou A A + com = 0 com || pequeno. Esse procedimento de regularizaao (que envolve a substituiao provisria de uma expresso singular por outra regular) denominado regularizaao de Tikhonov24 , em honra ao matemtico que a e c a desenvolveu essas idias no contexto de equaoes integrais25. e c
24 Andrei Nikolaevich Tikhonov (19061993). O sobrenome russo Tikhonov por vezes transliterado como Tykhonov, Tichonov ou e ainda Tychono. 25 Para uma referncia geral, vide [176]. Para os trabalhos originais, vide: Tikhonov, A. N., 1943, On the stability of inverse problems, e Dokl. Akad. Nauk. USSR, 39, No. 5, 195198 (1943); Tikhonov, A. N., Solution of incorrectly formulated problems and the regularization method, Soviet Math. Dokl. 4, 10351038 (1963), traduao para o ingls de Dokl. Akad. Nauk. USSR 151, 501504 (1963). c e

No Exerc E. 5.36, pgina 258, vimos que se (AA )1 existe, ento A+ = A (AA )1 e que se (A A)1 existe, ento cio a a a A+ = (A A)1 A . No caso de essas inversas no existirem h um procedimento alternativo que tambm permite obter a a e a vel A+ . Sabemos da Proposiao 5.4, pgina 196, que mesmo se (AA )1 no existir, a matriz AA + ser invert c a a para todo C no-nulo com || pequeno o suciente. Isso permite conjecturar que as expresses A (AA + )1 a o e (A A + )1 A , que esto bem denidas para = 0 com || pequeno, convergem a A+ quando tomamos o limite a 0. Como veremos no que segue, essa conjectura correta. e

JCABarata. Curso de F sica-Matemtica a

Verso de 4 de abril de 2009. a

Cap tulo 5

261/1628

Nosso primeiro resultado consiste em provar que os limites descritos acima de fato existem e so iguais, o que ser a a feito nos dois lemas que seguem. Lema 5.6 Seja A Mat (C, m, n) e seja C tal que AA + m e A A + n sejam inversveis (i.e., (AA ) (A A), um conjunto nito). Ento, A (AA + m )1 = (A A + n )1 A . a Prova. Sejam B := A (AA + m )1 e C := (A A + n )1 A . Temos que A AB = A AA (AA +m )1 = A AA +m m (AA +m )1 = A m (AA +m )1 = A B . Logo, (A A + n )B = A , o que implica B = (A A + n )1 A = C . Lema 5.7 Para toda A Mat (C, m, n) os limites lim A (AA + m )1 e lim (A A + n )1 A existem e so iguais a
0 0

(pelo Lema 5.6), denindo um elemento de Mat (C, n, m). Prova do Lema 5.7. Notemos primeiramente que A uma matriz identicamente nula se e somente se AA ou A A o forem. e De fato, se, por exemplo, A A = 0, valer para todo vetor x que 0 = x, A Ax C = Ax, Ax C = Ax 2 , provando que a A = 0. Como a armaao a ser provada evidente se A for nula, suporemos no que segue que AA e A A no so nulas. c e a a A matriz AA Mat (C, m) , evidentemente, auto-adjunta. Sejam 1 , . . . , r seus autovalores distintos. Pelo e Teorema Espectral para operadores auto-adjuntos (vide Teorema 5.5, pgina 211 e Teorema 5.13, pgina 226) podemos a a escrever
r

AA =
a=1

a Ea ,
r a=1

(5.127) Ea = m . Logo,

onde Ea so os projetores espectrais de AA e satisfazem Ea Eb = ab Ea , Ea = Ea e a r

AA + m =
a=1

(a + )Ea

e, portanto, para {1 , . . . , r }, vale pela Proposiao 5.16, pgina 213, c a AA + m


1

1 Ea a + a=1

A AA + m

1 A Ea . a + a=1

H dois casos a se considerar 1. AA no tem auto-valor nulo e 2. AA tem auto-valor nulo. a a No caso em que AA no tem auto-valor nulo claro pela ultima expresso que o limite lim A (AA + m )1 existe a e a
0

e vale
0

lim A (AA + m )1 =

1 A Ea . a a=1

(5.128)

No caso em que AA tem auto-valor nulo, digamos, 1 = 0, o projetor E1 projeta sobre o n cleo de AA : Ker (AA ) := u {u Cn | AA u = 0}. Se x Ker (AA ), ento A x = 0, pois 0 = x, AA x C = A x, A x C = A x . Portanto, a A E1 = 0 e, assim, podemos escrever, A (AA + m )1 = donde obtm-se e
0

(5.129)

1 A Ea , a + a=2 1 A Ea . a a=2
r

lim A (AA + m )1 =

(5.130)

Isso provou que lim A (AA + m )1 sempre existe.


0

JCABarata. Curso de F sica-Matemtica a

Verso de 4 de abril de 2009. a

Cap tulo 5

262/1628

Pelo Lema 5.6, pgina 261, o limite lim (A A + n )1 A tambm existe e coincide com lim A (AA + m )1 . a e
0 0

A principal conseqncia o seguinte resultado: ue e Teorema 5.31 (Regularizao de Tikhonov) Para toda A Mat (C, m, n) valem ca A+ = lim A AA + m
0 1

(5.131)

A+ = lim A A + n
0

A .

(5.132)

Como a existncia dos limites acima foi estabelecida para matrizes arbitrrias no Lema 5.7, pgina 261, o Teorema e a a 5.31 contm uma prova geral de existncia da pseudo-inversa de Moore-Penrose. e e Prova do Teorema 5.31. As armaoes a serem provadas so evidentes caso A = mn pois, como j vimos (mn )+ = nm . c a a Assim, assumiremos no que segue que A no nula, o que equivale, pelo exposto no in da prova do Lema 5.7, a supor e a cio que AA e A A no so nulas. a a Pelos Lemas 5.6 e 5.7 suciente demonstrar (5.131). H dois casos a se considerar 1. AA no tem auto-valor nulo e a a e 2. AA tem auto-valor nulo. No caso 1., vimos em (5.128), na prova do Lema 5.7 (e com a notaao l estabelecida), c a que r 1 1 A Ea =: B . = lim A AA + m 0 a=1 a Note-se agora que AB = 1 AA Ea = a a=1
r

1 a=1 a

b Eb
b=1

Ea =
a=1 b=1

1 b ab Ea = a

Ea = m ,
a=1

(5.133)

que auto-adjunta, e que e BA =

1 A Ea A , a a=1

(5.134)

que tambm auto-adjunta, pois a R para todo a (por serem autovalores de uma matriz auto-adjunta) e pelo fato e e de (A Ea A) = A Ea A para todo a, j que Ea = Ea . a De (5.133) segue que ABA = A. De (5.134) segue que BAB = 1 A Ea A a a=1
r r

b=1

1 A Eb b

=
a=1 b=1

1 A Ea (AA )Eb . a b

Agora, pela decomposiao espectral (5.127) de AA , segue que (AA )Eb = b Eb . Logo, c
r r

BAB =
a=1 b=1

1 A Ea Eb = a

1 A Ea a a=1

Eb
b=1

= B.

Isso provou que A = A+ no caso em que AA no tem autovalor nulo. a Vamos agora supor que AA no autovalor nulo, a saber, 1 . Vimos em (5.130), na prova do Lema 5.7, que a lim A AA + m
1

1 A Ea =: B . a=2 a

JCABarata. Curso de F sica-Matemtica a

Verso de 4 de abril de 2009. a

Cap tulo 5

263/1628

Usando o fato que (AA )Ea = a Ea , o qual segue da decomposiao espectral (5.127) de AA , obtm-se c e AB = 1 AA Ea = a a=2
r

1 a Ea = a a=2
r

r a=2

Ea = m E1 ,

(5.135)

que auto-adjunta, pois E1 o . Tem-se tambm e e e BA = 1 A Ea A , a a=2 (5.136)

que tambm auto-adjunta, pelos argumentos j expostos. e e a De (5.135) segue que ABA = A E1 A. Note-se agora que (E1 A) = A E1 = 0, por (5.129). Isso demonstrou que E1 A = 0 e que ABA = A. De (5.136) segue que BAB =

1 A Ea A a=2 a
r

r b=2

1 A Eb b

=
a=2 b=2

1 A Ea (AA )Eb . a b

Usando novamente que (AA )Eb = b Eb , obtemos


r r

BAB =
a=2 b=2

1 A Ea Eb = a

1 A Ea a a=2

Eb
b=2

= B

1 A Ea E1 = B , a a=2

m E1

pois Ea E1 = 0 para a = 1. Isso demonstrou que BAB = B. Assim, estabelecemos que A = A+ tambm no caso em que e AA tem autovalor nulo, completando a prova de (5.131).

5.9.1.2

A Pseudo-Inversa de Moore-Penrose e o Teorema Espectral


r

Durante a demonstraao do Teorema 5.31 estabelecemos tambm o seguinte resultado de interesse: c e Teorema 5.32 Seja A Mat (C, m, n) no-nula e seja AA = a=1 a Ea a representaao espectral de AA , onde a c {1 , . . . , r } R o conjunto dos autovalores distintos de AA e Ea so os correspondentes projetores espectrais e a auto-adjuntos. Ento, vale a r 1 A Ea . (5.137) A+ = a a=1
a =0

Analogamente, seja A A = b=1 b Fb a representaao espectral de A A, onde {1 , . . . , s } R o conjunto dos c e autovalores distintos de A A e Fb os correspondentes projetores espectrais auto-adjuntos. Ento, vale tambm a e
s

A+ =
b=1 b =0

1 Fb A . b

(5.138)

(Vale mencionar aqui que, pelo Exerccio E. 5.6, pgina 197, o conjunto de autovalores no-nulos de AA coincide com a a o conjunto de autovalores no-nulos de A A: {1 , . . . , r } \ {0} = {1 , . . . , s } \ {0}). a De (5.137) e (5.138) segue que para A no-nula valem a
r

A+

=
a=1 a =0

1
r

a
l=1 l=a

(a l )

A
s

l=1 l=a

AA l m ,

(5.139)

A+

=
b=1 b =0

1
r

b
l=1 l=b

(b l )

l=1 l=b

A A l n A .

(5.140)

JCABarata. Curso de F sica-Matemtica a

Verso de 4 de abril de 2009. a

Cap tulo 5

264/1628

As expresses (5.139) e (5.140) fornecem mais um algoritmo geral para o cmputo da pseudo-inversa de Mooreo o Penrose, o qual pode ser de implementaao simples, pois requer apenas a determinaao dos autovalores de AA ou de c c A A. Prova do Teorema 5.32. A igualdade (5.137) foi provada durante a demonstraao do Teorema 5.31 (vide (5.128) e (5.130)). c A relaao (5.138) pode ser provada analogamente, mas segue mais facilmente do truque j mencionado de usar (5.137), c a trocando A A e tomando-se o adjunto da expresso obtida. As relaoes (5.139) e (5.140) seguem da Proposiao 5.17, a c c pgina 213, particularmente de (5.48). a E. 5.40 Exerccio. Usando (5.139) ou (5.140) reobtenha as matrizes A+ dos Exerc cios E. 5.35, E. 5.37 e E. 5.38.

5.9.2

A Pseudo-Inversa de Moore-Penrose e Problemas de Optimizao ca Linear

Tratemos agora de uma das principais aplicaoes da noao de pseudo-inversa de Moore-Penrose, a saber, no tratamento c c de problemas de optimizaao linear, que motivaremos e deniremos a seguir. c Sejam A Mat (C, m, n) e y Cm dados e considere-se o problema de determinar x Cn que satisfaa a equaao c c linear Ax = y . (5.141) No caso em que m = n e A tem inversa, a soluao ( nica) , naturalmente, x = A1 y. Nos demais casos uma soluao c u e c pode no estar presente ou no ser unica. Podemos considerar o problema alternativo de saber para quais x Cn a a e vel. Tais vetores x Cn seriam, no sentido da norma Euclidiana a norma Euclidiana Ax y Cm a menor poss m , ou seja, em termos de m nimos quadrados, os melhores aproximantes ao que seria a soluao de (5.141). Um tal c C problema por vezes dito ser um problema de optimizaao linear. Esse problema pode ser tratado com o uso da noao e c c de pseudo-inversa de Moore-Penrose, a qual permite caracterizar precisamente o conjunto dos vetores x que minimizam Ax y Cm . A isso dedicaremos as linhas que seguem, sendo o principal resultado condensado no seguinte teorema: Teorema 5.33 (Optimizao Linear) Sejam A Mat (C, m, n) e y Cm dados. Ento a coleao de todos vetores ca a c de Cn para os quais a aplicaao Cn x Ax y Cm [0, ) assume um mnimo absoluto coincide com o conjunto c A+ y + Ker (A) = A+ y + n A+ A z, z Cn . (5.142)

Esse conjunto dito ser o conjunto minimizante do problema de optimizaao linear em questo. E interessante observar e c a + + + que pela Proposiao 5.34, pgina 260, tem-se tambm A y + Ker (A) = A y + Ran (A ) . c a e Como se v do enunciado acima, a pseudo-inversa de Moore-Penrose fornece a melhor aproximaao em termos de e c m nimos quadrados ` soluao de sistemas lineares. Observe-se que para os elementos x do conjunto minimizante (5.142) a c vale Ax y Cm = (AA+ m )y Cm = P2 y Cm , que nulo se e somente se y Ker (P2 ) = Ran (A) (pela Proposiao e c 5.34, pgina 260), um fato um tanto bvio. a o Prova do Teorema 5.33. A imagem de A, Ran (A), um subespao linear fechado de Cm . O Teorema do Melhor e c Aproximante e o Teorema da Decomposiao Ortogonal (que neste texto so apresentados com toda generalidade no c a contexto de espaos de Hilbert, como Cm na forma do Teorema 31.1, pgina 1468, e do Teorema 31.2, pgina 1470, c a a e nimo, sendo que esse y0 respectivamente) garantem-nos a existncia de um unico y0 Ran (A) tal que y0 y Cm m e satisfaz a propriedade de que y0 y ortogonal a Ran (A). e

Assim, existe ao menos um x0 Cn tal que Ax0 y Cm m e nimo. Tal x0 no necessariamente unico e, como a e e fcil ver, x1 Cn tem as mesmas propriedades se e somente se x0 x1 Ker (A) (j que Ax0 = y0 e Ax1 = y0 , pela a a unicidade de y0 ). Como observamos, Ax0 y ortogonal a Ran (A), ou seja, (Ax0 y), Au C = 0 para todo u Cn . e Isso signica que (A Ax0 A y), u C = 0 para todo u Cn e, portanto, x0 satisfaz A Ax0 = A y . (5.143)

JCABarata. Curso de F sica-Matemtica a

Verso de 4 de abril de 2009. a

Cap tulo 5

265/1628

Agora, a relaao (5.120) mostra-nos que x0 = A+ y satisfaz (5.143), pois A AA+ y = A y. Assim, conclu c mos que nimo composto por todos os vetores da e o conjunto de todos x Cn que satisfazem a condiao de Ax y Cm ser m c forma A+ y + x1 com x1 Ker (A). Pela Proposiao 5.34, pgina 260, x1 da forma x1 = (n A+ A)z para algum c a e z Cn , completando a prova. Os exerc cios que seguem ilustram a aplicaao da pseudo-inversa de Moore-Penrose no tratamento de problemas de c optimizaao linear. c E. 5.41 Exerccio. Usando o Exerc E. 5.37, pgina 258, determine o conjunto dos melhores aproximantes x C3 ` cio a a 1 i soluo da equao linear Ax = y com A = ( 2 0 1 ) e y = 2i . Para tais vetores minimizantes x, calcule Ax y C . ca ca 0 i O exerc que segue envolve uma situaao menos trivial que a do exerc anterior, pois trata de um sistema linear cio c cio sub-determinado e que no tem soluao. a c cio a E. 5.42 Exerccio. Usando o Exerc E. 5.38, pgina 258, determine o conjunto dos melhores aproximantes x C2 1 1 2 ` soluo da equao linear Ax = y com A = 0 i e y = 3 . Para tais vetores minimizantes x, calcule Ax y C . a ca ca 0 3 2 Observe que nesse caso y Ran (A) e, portanto, o sistema Ax = y no tem soluo. a ca

(5.120)

5.9.3

Existncia e Decomposio em Valores Singulares e ca

Passemos agora a uma segunda demonstraao da existncia da pseudo-inversa de Moore-Penrose de uma matriz A c e Mat (C, m, n) geral, fazendo uso aqui do Teorema da Decomposiao em Valores Singulares, Teorema 5.24, pgina 251. c a Trataremos primeiramente de matrizes quadradas para depois passarmos ao caso de matrizes no-quadradas. a Determinando a pseudo-inversa de Moore-Penrose para matrizes quadradas

Comearemos pelas matrizes diagonais. Se D Mat (C, n) uma matriz diagonal, a pseudo-inversa de Moore-Penrose c e de D dada pela matriz diagonal D+ Mat (C, n) cujos elementos diagonais so denidos para todo i = 1, . . . , n por e a (D+ )ii =
1 Dii

0,

, se Dii = 0 , se Dii = 0 .

E elementar vericar que DD+ D = D, D+ DD+ = D+ , e que DD+ e D+ D so auto-adjuntas. Em verdade, DD+ = a D+ D, uma matriz diagonal com elementos diagonais iguais a 0 ou a 1: (DD+ )ii = (D+ D)ii = 1 , se Dii = 0 , 0 , se Dii = 0 .

Passemos agora ` questo da existncia da pseudo-inversa de Moore-Penrose de uma matriz quadrada geral. Se a a e A Mat (C, n) tem uma decomposiao em valores singulares A = V SW (vide Teorema 5.24, pgina 251), ento a c a a pseudo-inversa de Moore-Penrose A+ de A dada por e A+ = W S + V . De fato, AA+ A = (V SW )(W S + V )(V SW ) = V SS + SW + = V SW = A, A+ AA+ = (W S + V )(V SW )(W S + V ) = W S + SS + V = W S + V = A+ . Alm disso, AA+ = (V SW )(W S + V ) = V (SS + )V auto-adjunta, pois SS + uma e e e matriz diagonal com elementos diagonais iguais a 0 ou a 1. Analogamente, A+ A = (W S + V )(V SW ) = W (S + S)W e auto-adjunta. Determinando a pseudo-inversa de Moore-Penrose para matrizes retangulares

Seja A Mat (C, m + n) a matriz quadrada (m + n) (m + n) denida em (5.5), pgina 187. Como A uma matriz a e quadrada, estabelecemos acima que ela possui uma pseudo-inversa de Moore-Penrose (A )+ , unica, satisfazendo

Consideraremos agora matrizes gerais (no necessariamente quadradas) A Mat (C, m, n). a

JCABarata. Curso de F sica-Matemtica a

Verso de 4 de abril de 2009. a

Cap tulo 5

266/1628

1. A (A )+ A = A , 2. (A )+ A (A )+ = (A )+ , 3. A (A )+ e (A )+ A so auto-adjuntas. a No que segue demonstraremos que A+ Mat (C, n, m), a pseudo-inversa de Moore-Penrose de A Mat (C, m, n), dada, seguindo as denioes (5.1)-(5.2), por e c A+ := (Jm+n, n )T (A )+ (Im, m+n )T , ou seja, A+ = (Jm+n, n )T (Im, m+n )T A(Jm+n, n )T
+

(5.144)

(Im, m+n )T .

(5.145)

O ponto de partida a existncia da pseudo-inversa de A . A relaao A (A )+ A = A signica, usando a deniao e e c c (5.5), (Im, m+n )T A (Jm+n, n )T (A )+ (Im, m+n )T A(Jm+n, n )T = (Im, m+n )T A(Jm+n, n )T e das relaoes (5.3)-(5.4) segue, multiplicando-se a esquerda por Im, m+n e ` direita por Jm+n, n que AA+ A = A, uma c ` a das relaoes que desejamos provar. c A relaao (A )+ A (A )+ = (A )+ signica, usando a deniao (5.5), c c (A )+ (Im, m+n )T A(Jm+n, n )T (A )+ = (A )+ . Multiplicando ` esquerda por (Jm+n, n )T e ` direita por (Im, m+n )T , isso estabelece a validade de A+ AA+ = A+ . a a Se A (A )+ auto-adjunta, segue da deniao a deniao (5.5), que (Im, m+n )T A(Jm+n, n )T (A )+ auto-adjunta, ou e c c e seja, (Im, m+n )T A(Jm+n, n )T (A )+ = A(Jm+n, n )T (A )+ Im, m+n . Logo, multiplicando-se ` esquerda por Im, m+n e ` direita por (Im, m+n )T , segue de (5.3) que a a A(Jm+n, n )T (A )+ (Im, m+n )T = Im, m+n A(Jm+n, n )T (A )+ provando que AA+ auto-adjunta. e Por m, se (A )+ A auto-adjunta, segue da deniao a deniao (5.5), que (A )+ (Im, m+n )T A(Jm+n, n )T autoe c c e adjunta, ou seja, (A )+ (Im, m+n )T A(Jm+n, n )T = Jm+n, n (A )+ (Im, m+n )T A . Logo, multiplicando-se ` esquerda por (Jm+n, n )T e ` direita por Jm+n, n , segue de (5.4) que a a (Jm+n, n )T (A )+ (Im, m+n )T A = estabelecendo que A+ A auto-adjunta. e Com isso estabelecemos que A+ dada em (5.144) a pseudo-inversa de Moore-Penrose de A. e (A )+ (Im, m+n )T A

T A(Jm+n, n )T (A )+ Im, m+n

Jm+n, n =

T Jm+n, n (A )+ (Im, m+n )T A

5.10
5.10.1

Propriedades Especiais de Determinantes


Expanso do Polinmio Caracter a o stico
n

m Seja A Mat (C, n) e seja pA () = det( A) = o stico. Desejamos m=0 cm , C, seu polinmio caracter obter uma frmula explicita para os coecientes cm em termos de determinantes de sub-matrizes de A (vide abaixo). o Vamos designar por ak a k-sima coluna de A, de sorte que, pela notaao introduzida em (5.7), pgina 188, valha e c a A = a1 , . . . , an . Recordando a deniao de base cannica fornecida em (5.8) e (5.9), pgina 188, ca claro que c o a

JCABarata. Curso de F sica-Matemtica a

Verso de 4 de abril de 2009. a

Cap tulo 5

267/1628

pA () = det e1 a1 , . . . , en an . Usando a propriedade de multilinearidade do determinante (linearidade em relaao a cada coluna), segue que c
n

pA () =

m=1

e onde, para 1 j1 < < jm n, a1 , . . . , ej1 . . . , ejm . . . , an a matriz obtida a partir da matriz A substituindo sua jl -sima coluna por ejl para cada l = 1, . . . , m. Note que no caso m = n, tem-se forosamente jl = l para cada e c l = 1, . . . , n e a1 , . . . , ej1 . . . , ejm . . . , an = e1 , . . . , en = . Com isso, escrevemos
n1

(1)nm m

1j1 <<jm n

det a1 , . . . , ej1 . . . , ejm . . . , an + (1)n det(A) ,

pA () = n +

m=1

(1)nm m

1j1 <<jm n

det a1 , . . . , ej1 . . . , ejm . . . , an + (1)n det(A) .

Como cada vetor-coluna ejl contm 1 na jl -sima linha, as demais linhas sendo nulas, as bem-conhecidas regras de e e clculo de determinantes ensinam-nos que, para todo m = 1, . . . , n 1, a det a1 , . . . , ej1 . . . , ejm . . . , an = det Aj1 , ..., jm ,

e Aj1 , ..., jm sendo a matriz de Mat (C, n m) (ou seja (n m) (n m)) obtida a partir de A eliminando-lhe as jl -simas linhas e colunas para todo l = 1, . . . , m. Assim, obtemos
n1

pA () = n +

m=1

onde poss reconhecer os coecientes de pA (). e vel

(1)nm m

1j1 <<jm n

det Aj1 , ..., jm + (1)n det(A) ,

(5.146)

Pelo Teorema de Hamilton-Cayley, Teorema 5.3, pgina 204, pA (A) = e, portanto, a


n1

An +

m=1

Como comentamos em (5.38), pgina 207, se A for invers a vel, obtm-se disso e n1 1 (1)nm An1 + A1 = det Aj1 , ..., jm Am1 . (1)n+1 det(A) m=1
1j1 <<jm n

(1)nm

1j1 <<jm n

det Aj1 , ..., jm Am + (1)n det(A) = .

(5.147)

5.10.2

A Desigualdade de Hadamard

Vamos nesta seao demonstrar uma desigualdade para determinantes de matrizes, a qual muito util, a chamada c e desigualdade de Hadamard26 . Teorema 5.34 (Teorema do Determinante de Hadamard) Seja A Mat (C, n). Ento, a
n n

| det(A)|2

j=1 i=1

|Aij |2 ,

(5.148)

sendo Aij o elemento ij da matriz A. Segue disso que para toda matriz A Mat (C, n) vale
n

| det(A)| nn/2

max |Aij |
ij

(5.149)

26 Jacques Salomon Hadamard (18651963). A referncia ao trabalho de Hadamard : J. Hadamard, Rsolution dune question relativ aux e e e dterminants, Bull. Sci. Math. 28, 240-246 (1893). e

JCABarata. Curso de F sica-Matemtica a

Verso de 4 de abril de 2009. a

Cap tulo 5

268/1628

O importante na estimativa (5.149) o tipo de dependncia em n que se tem do lado direito. Ela ser usada, por e e a exemplo, em estimativas de convergncia da srie de determinantes de Fredholm na Seao 14.2, pgina 664. e e c a Prova do Teorema 5.34. A prova de (5.149) elementar, por (5.148). Passemos ` prova de (5.148). e a Seja A Mat (C, n). Se A no tem inversa, ento det(A) = 0 e a desigualdade (5.148) trivialmente satisfeita, no a a e a havendo o que se provar. Vamos ento supor que A tenha inversa. a Seja A o conjunto de todas as matrizes M de Mat (C, n) com a propriedade que
n i=1 n

|Mij |2 =

i=1

|Aij |2

para todo j = 1, . . . , n. Claro est que A A. E tambm claro que A um subconjunto compacto de Mat (C, n) (visto a e e 2 aqui como Cn ). A funao | det(M )| cont c e nua como funao de M e, portanto, assume ao menos um mximo absoluto c a (no necessariamente unico) em A, por este ser compacto (teorema de Weierstrass). Seja T A um desses mximos. a a Note-se que | det(T )| | det(A)| > 0 e, portanto, T tem inversa.
n

Para todo i = 1, . . . , n vale por (5.18), pgina 189, que det(T ) = a


j=1

Tij Cof(T )ij , onde Cof(T ), chamada de

matriz dos cofatores de T , foi denida no enunciado do Teorema 5.1, pgina 189. Seja xo esse i. Pela desigualdade de a Cauchy-Schwarz, vale
n n n n

A ultima igualdade sendo devida ao fato que T A.

| det(T )|2

j=1

|Tij |2

j=1

|Cof(T )ij |2 =
n k=1

j=1

|Aij |2

j=1

|Cof(T )ij |2 .

(5.150)

Como bem sabido, para o produto escalar a, b := e

ak bk , a desigualdade de Cauchy-Schwarz | a, b | a

b e

uma igualdade se e somente se os vetores a e b forem proporcionais. Assim, tem-se a igualdade em (5.150) se e somente e a e se existir i C tal que Tij = i Cof(T )ij para todo j, ou seja, se a i-sima linha de T for proporcional ` i-sima linha de Cof(T ). O ponto importante agora e notar que se se tivermos a desigualdade estrita
n n

ento T no pode maximizar o mdulo de determinante entre as matrizes de A. De fato, considere a matriz T que a a o e igual ` matriz T , exceto sua i-sima linha, que dada por a e e
n j=1 n j=1

| det(T )|2 <

j=1

|Aij |2

j=1

|Cof(T )ij |2 ,

(5.151)

Tij

:=

2 |Cof(T )ij |
n

|Aij |

1/2

Cof(T )ij ,

j = 1, . . . , n. E claro que

n j=1 |Tij |2 =

j=1

|Aij |2 ,

JCABarata. Curso de F sica-Matemtica a

Verso de 4 de abril de 2009. a

Cap tulo 5

269/1628

o que mostra que T A (para as demais linhas T coincide com T e no h o que provar, pois T A). Fora isso, a a
n

det(T ) =
j=1

Tij Cof(T )ij , pois Cof(T )ij = Cof(T )ij , j que T e T s diferem na i-sima linha. Assim, a o e

e conclu mos por (5.151) que ter amos | det(T )| < det(T ), contrariando a hiptese que | det(T )| mximo. Assim, o e a devemos ter a igualdade em (5.150) e, pelos comentrios de acima, isso implica que existe i C tal que Tij = i Cof(T )ij a e a e e a para todo j, ou seja, a i-sima linha de T proporcional ` i-sima linha de Cof(T ). Como i arbitrrio, isso vale para e todo i. Agora, como as linhas de T so proporcionais as de Cof(T ), segue que a `
n

det(T ) =

n j=1

j=1

|Cof(T )ij |2

|Aij |2

1/2

n j=1

|Cof(T )ij |2 =

n j=1

|Aij |2

1/2

n j=1

|Cof(T )ij |2

1/2

det(T ) =
j=1

Tij Cof(T )ij =

1 i

n j=1

|Tij |2 , =

1 i

n j=1

|Aij |2

e pela multilinearidade do determinante, que det(T ) = det(T ) = 1 n det(Cof(T )) . Dessas duas relaoes extra c mos det(T )n = 1 1 n
n n n n

i=1 j=1

|Aij |2 =

det(Cof(T )) det(T )

i=1 j=1

|Aij |2 .

Como a relaao (5.23) vale para qualquer matriz invers c vel, tem-se det(Cof(T )) = det(T )n1 e, portanto, | det(T )|2 =
n

i=1 j=1

|Aij |2 . Por construao, T maximiza | det(T )| em A. Como A A, segue que c


n n

| det(A)|2 Isso prova o teorema.

i=1 j=1

|Aij |2 .

(5.152)

JCABarata. Curso de F sica-Matemtica a

Verso de 4 de abril de 2009. a

Cap tulo 5

270/1628

5.11

Exerc cios Adicionais

E. 5.43 Exerccio. a) Determine o polinmio caracter o stico da matriz 5 2 7 A = 0 2 3i 5i . 0 0 1 4i c) Usando o Teorema de Hamilton-Cayley calcule A1 .

b) Verique explicitamente a validade do Teorema de Hamilton-Cayley para a matriz A.

cio E. 5.44 Exerccio. Repita o exerc anterior para as matrizes 2 1 3 5 0 0 i , 8 0 . A1 = 0 4 + i A2 = 8 0 0 2 7i 3i 1 9i 4 5i E. 5.45 Exerccio. Considere em Cn o seguinte produto escalar
n

u, v

=
a=1

ua va pa ,

onde pa > 0 para a = 1, . . . , n. Seja uma matriz A, com elementos de matriz Aij . Mostre que, com o produto escalar , p e o elemento de matriz (Ap )ij da adjunta Ap da matriz A dado por pj Aji . (5.153) (Ap )ij = pi e (Lembre-se que Ap denida de sorte que u, Av
p

= Ap u, v

Mostre que para quaisquer u, v Cn vale u, v p = u, P v C , onde u, v C = n ua va o produto escalar usual em e a=1 e ca Cn e P = diag (p1 , . . . , pn ). Conclua disso que Ap = P 1 A P , onde A a adjunta usual de A em relao ao produto escalar , C : (A )ij = Aji . 4 i/2 . Essa matriz no auto-adjunta em relao ao a e ca 2i 5 produto escalar usual em C2 , mas possui autovalores reais. Justique esse fato mostrando, pelos exerc cios anteriores, que A auto-adjunta em relao ao produto escalar u, v p = 2u1 v1 + u2 v2 /2. Mostre a adjunta Ap em relao a esse produto e ca ca 4 i/2 escalar Ap = e = A e constate explicitamente que u, Av p = Au, v p para todos u, v C2 . Determine os 2i 5 autovetores de A e constate que os mesmos so ortogonais em relao ao produto escalar , p . a ca E. 5.46 Exerccio. Determine os autovalores da matriz A = O exerc que segue generaliza o Exerc E. 5.45. cio cio ca a u E. 5.47 Exerccio. Seja um produto escalar em Cn . Pela Proposio 3.5, pgina 166, existe uma nica matriz M Mat (C, n) auto-adjunta e de autovalores positivos (e, portanto, invers vel) tal que (x, y) = x, M y C para todos x, y Cn . ca Seja A Mat (C, n) e seja A Mat (C, n) sua adjunta em relao ao produto escalar : (x, Ay) = (A x, y) 1 e ca para todos x, y Cn . Mostre que A = M A M , onde A a adjunta usual de A em relao ao produto escalar , C . Mostre que para quaisquer matrizes A, B Mat (C, n) valem (A ) = A e (AB) = B A . Calcule .

Para a matriz adjunta denida em (5.153), verique a validade das regras (Ap )p = A e (AB)p = B p Ap , para quaisquer matrizes A, B Mat (C, n). Calcule p .

para todos u, v Cn ).

JCABarata. Curso de F sica-Matemtica a

Verso de 4 de abril de 2009. a

Cap tulo 5

271/1628

1 1

0
1
a

0 2 2
1

0
0

0
3

0
3
c

0
4

0
4
d

Figura 5.5: Forma cannica de uma matriz com 4 autovalores distintos 1 , 2 , 3 e 4 . Os s assumem apenas os o valores 0 ou 1, de acordo com as regras explicadas acima. Todos os elementos fora da diagonal principal e da primeira supradiagonal so nulos. As setas indicam zeros que ocorrem na primeira supradiagonal nos pontos onde ocorre transiao a c entre os blocos, conseqncia do fato de esses elementos estarem fora dos blocos. ue

Potrebbero piacerti anche